SlideShare a Scribd company logo
1 of 82
ĐẠI HỌC THÁI NGUYÊN
TRƢỜNG ĐẠI HỌC KHOA HỌC
---------------------------
Tải tài liệu tại sividoc.com
Viết đề tài giá sinh viên – ZALO:0973.287.149-TEAMLUANVAN.COM
ĐẶNG THỊ THU HÀ
MỘT SỐ PHƢƠNG PHÁP
GIẢI CÁC ĐỀ THI OLYMPIC
VỀ PHƢƠNG TRÌNH DIOPHANT
LUẬN VĂN THẠC SĨ TOÁN HỌC
THÁI NGUYÊN - 2019
ĐẠI HỌC THÁI NGUYÊN
TRƢỜNG ĐẠI HỌC KHOA HỌC
---------------------------
Tải tài liệu tại sividoc.com
Viết đề tài giá sinh viên – ZALO:0973.287.149-TEAMLUANVAN.COM
ĐẶNG THỊ THU HÀ
MỘT SỐ PHƢƠNG PHÁP
GIẢI CÁC ĐỀ THI OLYMPIC
VỀ PHƢƠNG TRÌNH DIOPHANT
Chuyên ngành: Phƣơng pháp Toán sơ cấp
Mã số: 8 46 01 13
LUẬN VĂN THẠC SĨ TOÁN HỌC
NGƯỜI HƯỚNG DẪN KHOA HỌC
GS.TSKH. Nguyễn Văn Mậu
THÁI NGUYÊN - 2019
i
Viết đề tài giá sinh viên – ZALO:0973.287.149-TEAMLUANVAN.COM
L i cảm ơn
Lu n văn này được hoàn thành tại trường Đại hoc Khoa hoc - Đại hoc Thái
Nguyên. Tác giả xin bày tỏ lòng biet ơn sâu sac đoi với GS.TSKH Nguyen Văn
M u (Trường ĐH Khoa hoc Tự nhiên, ĐHQGHN), thay đã trực tiep hướng dan
t n tình và đ®ng viên tác giả trong suot thời gian nghiên cáu vàa qua.
Xin chân thành cảm ơn tới các quý thay, cô giáo đã trực tiep giảng dạy lớp cao
hoc Toán K11, các bạn hoc viên, và các bạn đong nghi p đã tạo đieu ki n thu n
lợi, đ®ng viên giúp đơ tác giả trong quá trình hoc t p và nghiên cáu tại trường.
Tác giả cũng xin bày tỏ lòng biet ơn sâu sac tới gia đình và người thân luôn khuyen
khích đ®ng viên tác giả trong suot quá trình hoc cao hoc và viet lu n văn này.
M c dù có nhieu co gang nhưng lu n văn khó tránh khỏi nhǎng thieu sót và
hạn che. Tác giả mong nh n được nhǎng ý kien đóng góp của các thay cô và các
bạn đoc đe lu n văn được hoàn thi n hơn.
Xin chân thành cảm ơn!
Thái Nguyên, tháng 11 năm 2019
Tác giả
Đ ng Thị Thu Hà
ii
Viết đề tài giá sinh viên – ZALO:0973.287.149-TEAMLUANVAN.COM
Mnc lnc
M ĐAU 1
Chương 1. Phương trình Diophant và h Diophant cơ bản 2
1.1 Phương trình Diophant tuyen tính . . . . . . . . . . . . . . . . . . 2
1.1.1 Nghi m riêng . . . . . . . . . . . . . . . . . . . . . . . . . 3
1.1.2 Nghi m nguyên dương . . . . . . . . . . . . . . . . . . . . . 9
1.2 Nghi m nguyên dương của h phương trình Diophant tuyen tính cơ
bản.................................................................................................................. 10
Chương 2. Các phương pháp giải phương trình Diophant 19
2.1 Phương pháp phân tích thành nhân tả...................................................... 19
2.2 Phương pháp đong dư.................................................................................. 24
2.3 Phương pháp đánh giá ................................................................................. 25
2.4 Phương pháp tham so hóa........................................................................... 27
2.5 Phương pháp quy nạp toán hoc.................................................................. 30
2.6 Phương pháp xuong thang........................................................................... 33
2.7 M®t so phương pháp khác ........................................................................... 40
Chương 3. Các dạng toán liên quan đen phương trình và h phương
trình Diophant 47
3.1 M®t so dạng toán ve đa thác nguyên......................................................... 47
3.2 M®t so dạng toán lượng giác liên quan ...................................................... 50
3.3 M®t so dạng toán thi Olympic liên quan ............................................... 66
KET LU N 77
TÀI LI U THAM KHẢO 78
1
Viết đề tài giá sinh viên – ZALO:0973.287.149-TEAMLUANVAN.COM
M đau
Trong các kì thi hoc sinh giỏi toán các cap, Olympic Toán sinh viên, các bài
toán liên quan tới phương trình Diophant (dạng tuyen tính và phi tuyen) thường
xuyên được đe c p. Nhǎng dạng toán này thường được xem là thu®c loại khó vì
phan kien thác ve phương trình Diophant tőng quát không nam trong chương trình
chính thác của giáo trình So hoc và Đại so b c trung hoc phő thông.
Đe đáp áng nhu cau boi dương giáo viên và boi dương hoc sinh giỏi ve chuyên
đe phương trình Diophant, tôi chon đe tài lu n văn "M®t so phương pháp giải các
đe thi Olympic ve phương trình Diophant".
Tiep theo, khảo sát m®t so lớp h phương trình Diophant liên quan.
Cau trúc lu n văn gom 3 chương:
Chương 1. Các kien thác bő túc ve so hoc và phương trình Diophant cơ bản.
Chương 2. Các phương pháp giải phương trình Diophant.
Chương 3. Các dạng toán liên quan đen h phương trình Diophant.
Tiep theo, cuoi các chương đeu trình bày các bài t p áp dụng và giải các đe thi
HSG quoc gia và Olympic liên quan.
2
Viết đề tài giá sinh viên – ZALO:0973.287.149-TEAMLUANVAN.COM
Chương 1. Phương trình Diophant và
h Diophant cơ bản
1.1 Phương trình Diophant tuyen tính
Ta nhac lại thu t toán Euclid và liên phân so đã được trình bày tương đoi chi
tiet trong chương trình toán b c THCS.
Định nghĩa 1.1. So nguyên c được goi là m®t ước so chung của hai so nguyên a
và b (không đong thời bang không) neu c chia het a và c chia het b (hay a và b
đeu chia het cho c).
Định nghĩa 1.2 (xem [3,5, 7]). M®t ước so chung d của hai so nguyên a và b
(không đong thời bang không) được goi là ước so chung lớn nhat của a và b neu
moi ước so chung c của a và b đeu là ước của d.
Nh n xét 1.1. Neu d là ước so chung lớn nhat của a và b thì −d cũng là ước so
chung lớn nhat của a và b. Vì v y, ta quy ước rang ước so chung lớn nhat của a
và b là so nguyên dương.
Ước so chung lớn nhat của hai so a và b được ký hi u là (a, b) hay gcd(a,b)
(greatest common divisor). Như v y d = (a, b) hay d = gcd(a, b).
Ví dn 1.1. (25,30) = 5, (25,-72) = 1.
Định nghĩa 1.3 (xem [3,5,7]). M®t so nguyên c được goi là m®t ước so chung của
b® n so nguyên a1, a2, a3, . . . , an (không đong thời bang không) neu c là ước của
moi so đó.
Định nghĩa 1.4 (xem [5,7]). M®t ước so chung d của b® n so nguyên a1, a2, a3, . . . , an
(không đong thời bang không) được goi là ước so chung lớn nhat của a1, a2, a3, . . . , an
neu moi ước so chung c của a1, a2, a3, . . . , an đeu là ước của d.
Tương tự, ta cũng quy ước rang ước so chung lớn nhat của n so nguyên
a1, a2, a3, . . . , an là so nguyên dương.
3
Viết đề tài giá sinh viên – ZALO:0973.287.149-TEAMLUANVAN.COM
n−1
x = (
−
1
)
.c.q
0
n−1
x0 = (−1)n−1
.c.qn−1
Bài toán 1.1. Giải phương trình Diophant tuyen tính
Ước so chung lớn nhat của a1, a2, a3, . . . , an ký hi u là
(a1, a2, a3, . . . , an) hay gcd(a1, a2, a3, . . . , an).
Như v y d = (a1, a2, a3, . . . , an) hay d = gcd(a1, a2, a3, . . . , an).
Định lý 1.1 (Ước so chung lớn nhat của nhieu so). Cho các so nguyên a1, a2, a3, . . . , an
không đong thời bang không. Khi đó ton tại ước so chung lớn nhat của a1, a2, a3, . . . , an.
Tính chat 1.1. Cho a, b, q, r là các so nguyên (a2
+ b2
/= 0). Neu a = bq + r và
0 ≤ r < |b| thì (a, b) = (b, r).
1.1.1 Nghi m riêng
Trong mục này, ta trình bày hai thu t toán tìm nghi m riêng của phương trình
Diophant, đó là thu t toán giản phân và thu t toán Euclid.
Xét phương trình Diophant tuyen tính
Ax + By = C. (1.1)
Đe tìm nghi m riêng dựa vào giản phân, ta tien hành thực hi n theo các bước như
sau:
- Bước 1. Tìm d = (A, B) đe đưa phương trình (1.1) ve phương trình (1.2) với
(a, b) = 1. phương trình Diophant tuyen tính
- Bước 2. Viet
a
|b|
ax + by = c. (1.2)
= [a0; a1, a2, . . . , an].
p
- Bước 3. Tính giản phân Cn−1 = [a0; a1, . . . , an−1] =
qn−1.
n−1
. Suy ra p và
qn−1
n−1
- Bước 4. Suy ra m®t nghi m riêng (x0, y0) của phương trình (1.2).
Neu b > 0 thì
Neu b < 0 thì
y0 = (−1)n
.c.pn−1.
y0 = (−1)n−1
.c.pn−1.
342x − 123y = 15. (1.3)
4
Viết đề tài giá sinh viên – ZALO:0973.287.149-TEAMLUANVAN.COM
0
d
Cm
Lài giai. Phương trình đã cho tương đương với phương trình
114x − 41y = 5. (1.4)
Ta có
114
41
= [2; 1, 3, 1, 1, 4], với n = 5.
C =
p4
4
q4
25
= [2; 1, 3, 1, 1] =
9 nên p4
= 25
(p4, q4) = 1
Do b = −41 < 0 nên m®t nghi m riêng của (1.4) là
x0 = (−1)5−1
.5.9 = 45
y0 = (−1)5−1
.5.25 = 125.
q4 = 9.
x = 45 + 41t
y = 125 + 114t
, t ∈ Z.
Đe tìm nghi m riêng dựa vào thu t toán Euclid, ta tien hành thực hi n theo
các bước như sau:
- Bước 1. Xác định d = (|A| , |B|) theo thu t toán Euclid mở r®ng.
- Bước 2. Bieu thị d như m®t tő hợp tuyen tính của A và B, chȁng hạn
d = nA + mB (n, m ∈ Z) .
C
- Bước 3. Nhân hai ve đȁng thác trên với
d
ta thu được
Cn Cm
A + B
d d
= C.
- Bước 4. Suy ra m®t nghi m riêng (x0, y0) của phương trình (1.1) là
Cn
x0 =
d
y = .
Bài toán 1.2. Giải phương trình Diophant tuyen tính
342x − 123y = 15. (1.5)
V y nghi m của phương trình (1.4), tác phương trình (1.14) là
5
Viết đề tài giá sinh viên – ZALO:0973.287.149-TEAMLUANVAN.COM
3
, t ∈ Z
Lài giai. Vì (342, −123) = 3 |15 nên phương trình đã cho có nghi m. Ta có
342 = 123.2 + 96,
123 = 96.1 + 27,
96 = 27.3 + 15,
27 = 15.1 + 12,
15 = 12.1 + 3,
12 = 3.4 + 0.
Suy ra
3 = 15 − 12.1 = 15 − (27 − 15.1).1 = 15.2 − 27.1
= (96 − 27.3).2 − 27.1 = 96.2 − 27.7 = 96.2 − (123 − 96.1).7
= 96.9 − 123.7 = (342 − 123.2).9 − 123.7 =
= 342.9 − 123.25.
Suy ra
342.45 − 123.125 = 15.
Tà đó, phương trình (1.14) có m®t nghi m riêng là
(x0; y0) = (45; 125).
Suy ra nghi m tőng quát của phương trình (1.14) là
x = 45 +
123
t
hay
342
y = 125 +
3
t
x = 45 + 41t
y = 125 + 114t
, t ∈ Z.
Định lj 1.2. Xét phương trình Diophant tuyen tính
a1x1 + a2x2 + . . . + anxn = c. (1.6)
1. Phương trình (1.6) có nghi m khi và chỉ khi d = (a1, a2, . . . , an) |c.
2. Neu phương trình (1.6) có nghi m thì nó sě có vô so nghi m.
6
Viết đề tài giá sinh viên – ZALO:0973.287.149-TEAMLUANVAN.COM
Σ
.Σ
.
Σ
Σ
Σ
Σ
k
ChGng minh.
1) ⇒). Giả sả (x1, x2, . . . , xn) là m®t nghi m của phương trình (1.6), tác là
n
aixi = c.
i=1
n
Ta có d = (a1, a2, . . . , an), suy ra d
i=1
aixi , suy ra d |c.
⇐) Ta cháng minh khȁng định bang phương pháp quy nạp theo n.
Với n = 2, khȁng định là đúng.
Giả sả khȁng định đúng với n = k (k ≥ 2).
Với n = k + 1, xét d = (a1, a2, . . . , ak+1) |c, đ t h = (a1, a2, . . . , ak). Khi đó, ta
có
Suy ra, ton tại t, xk+1 ∈ Z đe
d = (h, ak+1) |c
ht + ak+1xk+1 = c.
Vì h |ht nên theo giả thiet quy nạp sě ton tại x1, x2, . . . , xk ∈ Z đe
k
aixi = ht.
i=1
Do đó
k+1
aixi = c.
i=1
V y nên phương trình a1x1+a2x2+. . .+ak+1xk+1 = c có nghi m (x1, x2, . . . , xk+1).
2). Ta cháng minh khȁng định bang phương pháp quy nạp theo n.
Với n = 2: khȁng định là đúng. Giả sả khȁng định đúng với n = k (k ≥ 2), tác là
phương trình aixi = c neu có nghi m thì sě có vô so nghi m. Với n = k + 1, ta
i=1
k
Σ
+1
sě cháng tỏ phương trình aixi = c neu có nghi m thì sě có vô so nghi m.
i=1
k
Σ
+1
Th t v y, goi (t1, t2, . . . , tk+1) là m®t nghi m của phương trình
là
k+1
aiti = c.
i=1
i=1
aixi = c, tác
7
Viết đề tài giá sinh viên – ZALO:0973.287.149-TEAMLUANVAN.COM
Σ
Σ
Σ
Σ
Σ
Khi đó
Xét phương trình
k
aiti = c − ak+1tk+1.
i=1
k
aixi = c − ak+1tk+1 (1.7)
i=1
có ve phải là hang so và có m®t nghi m là (t1, t2, . . . , tk) nên theo giả thiet quy
nạp thì phương trình này có vô so nghi m.
Úng với moi nghi m (x1, x2, . . . , xk) của (6) thì phương trình
k
Σ
+1
k+1
i=1
aixi = c có
Ta de dàng cháng minh được nghi m tőng quát của phương trình (1.6) có dạng
x1 = x1
n−1
+
i=1
n
Σ
−1
Li1ti ,
x2 = x2 +
. . .
xn = xn +
i=1
n−1
i=1
Li2ti ,
Linti.
trong đó (x1, x2, . . . , xn) là m®t nghi m riêng của (1.6), ti ∈ Z, ∀i = 1, 2, . . . , n.
Tuy nhiên, các h so Lij (i, j = 1, 2, . . . , n) không có công thác tính tường minh.
Bài toán 1.3. Giải phương trình Diophant tuyen tính
6x + 15y + 10z = 3. (1.8)
Lài giai. Phương trình đã cho tương đương với
6(x + z) + 15y + 4z = 3.
Đ t u = x + z, ta thu được phương trình
15y + 4z = 3 − 6u. (1.9)
Ta nh n thay phương trình
15y + 4z = 1
i=1
nghi m là (x1, x2, . . . , xk, tk+1). Cháng tỏ phương trình
nghi m.
aixi = c có vô so
8
Viết đề tài giá sinh viên – ZALO:0973.287.149-TEAMLUANVAN.COM
.
x = −12 + 25u + 15t
có m®t nghi m riêng là (y1; z1) = (−1; 4) nên phương trình (7a) có m®t nghi m
riêng là (y0; z0) = (−3 + 6u; 12 − 24u).
Do đó nghi m tőng quát của phương trình (1.9) là
y = −3 + 6u + 4t
z = 12 − 24u − 15t
, u, t ∈ Z.
M t khác, u = x + z suy ra x = u − z = −12 + 25u + 15t.
V y nên phương trình (1.8) có nghi m tőng quát là
y = −3 + 6u + 4t
z = 12 − 24u − 15t
, u, t ∈ Z.
Nh n xét 1.2. Có the tóm lược cách giải phương trình Diophant tuyen tính nhieu
ȁn trên như sau:
Tà phương trình Diophant tuyen tính n ȁn, ta đưa ve phương trình Diophant
tuyen tính n − 1 ȁn (giảm ȁn), tiep tục như v y, sau hǎu hạn bước, ta nh n được
phương trình Diophant tuyen tính 2 ȁn.
Moi lan giảm so ȁn như v y ta lại giải phương trình Diophant tuyen tính 2 ȁn
(cháa tham so). Vì lě đó, ta thu được h nghi m phụ thu®c vào n − 1 tham so.
Ví dn 1.2. Xét phương trình (1.6) với n ≥ 3, ai 0, ∀i = 1, 2, . . . , n.
Goi d = (an−1, an). Khi đó an−1 = d.bn−1, an = d.bn, (bn−1, bn) = 1. Do đó
(1.6) trở thành
a1x1 + a2x2 + . . . + an−2xn−2 + d (bn−1xn−1 + bnxn) = c. (1.10)
Đưa vào ȁn mới t bang h thác
bn−1xn−1 + bnxn = t. (1.11)
Khi đó (1.6) trở thành
a1x1 + a2x2 + . . . + an−2xn−2 + dt = c. (1.12)
Giả sả x1, x2, . . . , xn−2, t là m®t nghi m nguyên của (1.12).
Úng với so t xác định, xét phương trình
bn−1xn−1 + bnxn = t. (1.13)
Do (bn−1, bn) = 1 t nên (1.13) nhat định có nghi m nguyên, chȁng hạn (xn−1, xn).
Khi đó, rõ ràng (x1, x2, . . . , xn−2, xn−1, xn) là nghi m nguyên của (1.6).
De thay moi nghi m nguyên của (1.13) chính là nghi m của (1.12) với đieu ki n
(1.11).
9
Viết đề tài giá sinh viên – ZALO:0973.287.149-TEAMLUANVAN.COM
ai
x = 25 thì z = 2.
Hơn nǎa, tà giả thiet bài ra, suy ra x . 5 và y . 2.
1.1.2 Nghi m nguyên dương
Xét phương trình Dipophant tuyen tính
a1x1 + a2x2 + . . . + anxn = c (1.14)
với các h so ai, c ∈ Z+
, các bien so xi ∈ Z+
, ∀i = 1, 2, . . . , n. Khi đó phương
trình (1.14) luôn có hǎu hạn nghi m nguyên dương x = (x1, x2, . . . , xn).
Tà giả thiet bài ra, ta có the hạn che đieu ki n của các bien so bởi
1 ≤ xi ≤
(c + ai) − (a1 + a2 + . . . + an)
, ∀i = 1, 2, . . . , n.
Khi đó, cách đơn giản nhat đe tìm nghi m nguyên dương x = (x1, x2, . . . , xn)
của phương trình (1.14) là ta cho m®t bien so xi nào đó lan lượt chạy qua các giá
trị có the có của nó và tìm các bien so còn lại tà phương trình đã cho.
Bài toán 1.4. Tìm các nghi m nguyên dương của phương trình Diophant tuyen
tính
6x + 15y + 10z = 200. (1.15)
Lài giai. Theo giả thiet, ta có
1 ≤ x ≤ 29
1 ≤ y ≤ 12
1 ≤ z ≤ 17.
- Với y = 2, ta có 3x + 5z = 85 nên
x = 5 thì z = 14;
x = 10 thì z = 11;
x = 15 thì z = 8;
x = 20 thì z = 5;
- Với y = 4, ta có 3x + 5z = 70 nên
x = 5 thì z = 11;
x = 10 thì z = 8;
x = 15 thì z = 5;
x = 20 thì z = 2.
10
Viết đề tài giá sinh viên – ZALO:0973.287.149-TEAMLUANVAN.COM
x = 10 thì z = 2.
- Với y = 6, ta có 3x + 5z = 55 nên
x = 5 thì z = 8;
x = 10 thì z = 5;
x = 15 thì z = 2.
- Với y = 8, ta có 3x + 5z = 40 nên
x = 5 thì z = 5;
- Với y = 10, ta có 3x + 5z = 25 nên
x = 5 thì z = 2.
- Với y = 12, ta có 3x + 5z = 10. Phương trình này không có nghi m nguyên
dương.
Như v y, phương trình (1.15) có cả thảy 15 nghi m nguyên dương (x, y, z) bao
gom
(5, 2, 14) , (5, 4, 11) , (5, 6, 8) , (5, 8, 5) , (5, 10, 2) ,
(10, 2, 11) , (10, 4, 8) , (10, 6, 5) , (10, 8, 2) , (15, 2, 8) ,
(15, 4, 5) , (15, 6, 2) , (20, 2, 5) , (20, 4, 2) , (25, 2, 2) .
1.2 Nghi m nguyên dương của h phương trình Diophant
tuyen tính cơ bản
Phan này đe c p tới h phương trình Diophant tuyen tính, trình bày ve h hai
phương trình ba ȁn với nguy n nguyên dương giải bang phương pháp ”chìa khóa”
(xem [1],[3] và [5]).
Bài toán tong quát 1.1. Tìm nghi m nguyên dương của h phương trình
Phương pháp giải:
a1x + b1y + c1z = s1
a2x + b2y + c2z = s2.
(1.16)
Ta goi ”chìa khóa” của h (1) là b® so (x0, y0, z0) thỏa mãn đieu ki n: x0, y0, z0 ∈
Z và
11
Viết đề tài giá sinh viên – ZALO:0973.287.149-TEAMLUANVAN.COM
a x + b y + c z = 0.20
2
0 2 0
4x + y + z = 322
3
2 1
1
1 1
a1x0 + b1y0 + c1z0 = 0
Neu (x1, y1, z1) là m®t nghi m của h (1.16) thì với
x2 = x1 + mx0 ; y2 = y1 + my0 ; z2 = z1 + mz0,
ta cũng có
a1x2 + b1y2 + c1z2 = s1 ; a2x2 + b2y2 + c2z2 = s2,
tác là (x2, y2, z2) cũng là m®t nghi m của h (1). Neu các nghi m này nguyên
dương thì ta có nghi m nguyên dương. Ta đe ý rang trong h (1) neu x xác định
thì y và z cũng xác định. Vì v y khi ta cho x chạy qua tat cả các giá trị nguyên
dương có the có của nó, ta sě tìm được các giá trị tương áng của y và z. Trong
các trường hợp này, có bao nhiêu trường hợp các giá trị tương áng của y và z đeu
nguyên dương thì h có bay nhiêu nghi m nguyên dương.
Bài toán 1.5. Tìm nghi m nguyên dương của h phương trình
x + y + z = 134
(1.17)
Lài giai. Ta có
2x +
2
y +
3
z = 161.
(1.17) ⇔
x + y + z = 134
Suy ra z = 3, x = 63, y = 68.
x + y + z = 134
3x −
3
z = 188.
Suy ra h (1.17) có m®t nghi m (x1, y1, z1) = (63, 68, 3) .
Tìm chìa khóa (x0, y0, z0) thỏa mãn đieu ki n
x0 + y0 + z0 = 0
Suy ra
2x0 +
2
y0 +
3
z0 = 0.
x0 + y0 + z0 = 0
3x0 −
3
z0 = 0.
Suy ra x0 = 1, z0 = 9, y0 = −10 hay (x0, y0, z0) = (1, −10, 9). Khi đó
1
1
⇔
12
Viết đề tài giá sinh viên – ZALO:0973.287.149-TEAMLUANVAN.COM
2
⇔
1
suy ra
x2 = x1 + mx0
y2 = y1 + my0
z2 = z1 + mz0,
x2 = 63 + m.1
y2 = 68 + m. (−10)
z2 = 3 + m.9.
• Với m = 1, h có nghi m (64, 58, 12);
• Với m = 2, h có nghi m (65, 48, 21);
• Với m = 3, h có nghi m (66, 38, 30);
• Với m = 4, h có nghi m (67, 28, 39);
• Với m = 5, h có nghi m (68, 18, 48);
• Với m = 6, h có nghi m (69, 8, 57);
• m ∈ Z−
, neu
x = 63 + m.1 thì y = 68 + m.(−10) ; z = 3 + m.9 < 0.
• m ∈ Z+
, neu x = 69 + m.1 thì y = 8 + m.(−10) < 0.
V y bài toán có 7 nghi m nguyên dương.
Bài toán 1.6. Tìm nghi m nguyên dương của h phương trình
x + y + z = 134
(2)
2x +
2
y +
3
z = 40.
Lài giai. (2) ⇔
−x − y − z = −134
4x + y +
3
z = 80
3x −
3
z = −54
−x − y − z = −134
nên x = 1, z = 171, y = −38.
Suy ra h (2) có m®t nghi m (x1, y1, z1) = (1, −38, 171) .
Tìm chìa khóa (x0, y0, z0) thỏa mãn đieu ki n:
1
1
13
Viết đề tài giá sinh viên – ZALO:0973.287.149-TEAMLUANVAN.COM
1
15x + 9y + z = 300
1
1 1
x0 + y0 + z0 = 0
2x0 +
2
y0 +
3
z0 = 0
”Chìa khóa” van là (x0, y0, z0) = (1, −10, 9) .
Khi đó
suy ra
x2 = x1 + mx0
y2 = y1 + my0
z2 = z1 + mz0
x2 = 1 + m.1
y2 = −38 + m. (−10)
z2 = 171 + m.9.
• Với m ∈ Z+
, neu x = 1 + m.1 suy ra y = −38 + m.(−10) < 0.
V y bài toán không có nghi m nguyên dương.
Bài toán 1.7. Tìm nghi m nguyên dương của h phương trình
x + y + z = 100
5x + 3y +
3
z = 100
(1.18)
Lài giai. Ta có
(1.18) ⇔
x + y + z = 100
Suy ra x . 4 nên x = 4, y = 18, z = 78.
x + y + z = 100
14x + 8y = 200.
Suy ra h (1.18) có m®t nghi m (x1, y1, z1) = (4, 18, 78) .
Tìm ”chìa khóa” (x0, y0, z0) thỏa mãn đieu ki n:
x0 + y0 + z0 = 0
5x0 + 3y0 +
3
z0 = 0
x0 + y0 + z0 = 0
14x0 + 8y0 = 0
Suy ra h có nghi m x0 = 4, y0 = −7, z0 = 3.
Suy ra ”chìa khóa” (x0, y0, z0) = (4, −7, 3).
⇔
⇔
14
Viết đề tài giá sinh viên – ZALO:0973.287.149-TEAMLUANVAN.COM
3x + 4y + 5z = 50
Khi đó
suy ra
x2 = x1 + mx0
y2 = y1 + my0
z2 = z1 + mz0
x2 = 4 + m.4
y2 = 18 + m. (−7)
z2 = 78 + m.3
• Với m = 1 h có nghi m (8, 11, 81);
• Với m = 2 h có nghi m (12, 4, 84);
• m ∈ Z−
thì x = 4 + m.4 ≤ 0;
• m ∈ Z+
, neu x = 12 + m.4 thì y = 4 + m.(−7) < 0.
V y bài toán có 3 nghi m nguyên dương.
Bài toán 1.8. Tìm nghi m nguyên dương của h phương trình
Lài giai. Ta có
x + y + z = 12
3x + 4y + 5z = 50
(1.19)
(1.19) ⇔
3x + 3y + 3z = 36
Suy ra y = 2, z = 6, x = 4
3x + 3y + 3z = 36
x + 2z = 14
V y h (1.19) có m®t nghi m (x1, y1, z1) = (4, 2, 6) .
Tìm ”chìa khóa” (x0, y0, z0) thỏa mãn đieu ki n:
x0 + y0 + z0 = 0
3x0 + 4y0 + 5z0 = 0
Suy ra
x0 + y0 + z0 = 0
y0 + 2z0 = 0
⇔
15
Viết đề tài giá sinh viên – ZALO:0973.287.149-TEAMLUANVAN.COM
⇔
−x − 15y + 20z = 0
0 0
0
nên ta có
z0 = 1, y0 = −2, x0 = 1.
V y nên, ”chìa khóa” (x0, y0, z0) = (1, −2, 1)
x2 = x1 + mx0
y2 = y1 + my0
z2 = z1 + mz0
⇔
x2 = 4 + m.1
y2 = 2 + m. (−2)
z2 = 6 + m.1
• Với m = −1 h có nghi m (3, 4, 5);
• Với m = −2 h có nghi m (2, 6, 4);
• Với m = −3 h có nghi m (1, 8, 3);
• m ∈ Z−
thì x = 1 + m.1 ≤ 0;
• m ∈ Z+
, neu x = 4 + m.1 thì y = 2 + m.(−2) ≤ 0.
V y bài toán có 4 nghi m nguyên dương.
Bài toán 1.9. Tìm nghi m nguyên dương của h phương trình
x + y + z = 6
−x − 15y + 20z = 29
(1.20)
Lài giai. Ta có
(1.20)
x + y + z = 6
−14y + 21z = 35
Suy ra y = 2, z = 3, x = 1.
V y nên h (1.20) có m®t nghi m (x1, y1, z1) = (1, 2, 3) .
Tìm ”chìa khóa” (x0, y0, z0) thỏa mãn đieu ki n:
x0 + y0 + z0 = 0
V y nên, ta có y0 = 3, z0 = 2, x0 = −5.
x0 + y0 + z0 = 0
−14y0 + 21z0 = 0
Suy ra ”chìa khóa” (x0, y0, z0) = (−5, 3, 2).
Khi đó
x2 = x1 + mx0
z2 = z1 + mz0
⇔
y2 = y1 + my0
16
Viết đề tài giá sinh viên – ZALO:0973.287.149-TEAMLUANVAN.COM
3
3 4 5
3
9
5
3
9
0
5
3
25
Suy ra
x2 = 1 + m. − 5
y2 = 2 + m.3
z2 = 3 + m.2
• m ∈ Z=
thì y = 2 + m.3 < 0
• m ∈ Z+
thì x = 1 + m.(−5) < 0
V y bài toán chỉ có 1 nghi m nguyên dương.
Bài toán 1.10. Tìm nghi m nguyên dương của h phương trình
1
x −
1
y +
1
z = 1
Lài giai. Ta có
3x + 4y − 5z = 0.
16 16
x − 4y + z = 16
Suy ra
(1.21) 3
9
3
x + 4y −
5
25z = 0.
5
25
x −
9
z = 16
3
x + 4y −
nên ta có x = 3, z = 5, y = 4
z = 0
5
Suy ra h (1.21) có m®t nghi m (x1, y1, z1) = (3, 4, 5)
Tìm ”chìa khóa” (x0, y0, z0) thỏa mãn đieu ki n:
1
x
1
—
4
y0
1
+
5
z0 = 0
16
0
⇔ 9
— 4y0
16
+
5
z0 = 0
25
suy ra
3x0 + 4y0 − 5z0 = 0
25
x =
9
z
3
x0 + 4y0 − 5
z0 = 0
3
x0 + 4y0 − 5
z0 = 0
Suy ra ”chìa khóa” (x0, y0, z0) = (27, 136, 125).
⇔
(1.21)
25
0
0 x
17
Viết đề tài giá sinh viên – ZALO:0973.287.149-TEAMLUANVAN.COM
3 4 5 15
3
9
5
25
15
120
125x − 27z = 632
9
Khi đó
x2 = x1 + mx0
y2 = y1 + my0
z2 = z1 + mz0
hay
x2 = 3 + m.27
y2 = 4 + m.136
z2 = 5 + m.125
m ∈ Z+
, suy ra x2 = 3 + 27m; y2 = 4 + 136m; z2 = 5 + 125m.
V y bài toán có vô so nghi m nguyên dương.
Ví dn 1.3. Tìm nghi m nguyên dương của h phương trình
1
x −
1
y +
1
z =
32
Lài giai. Ta có
3x + 4y − 5z = 8.
16
x − 4y +
16
z =
512
nên
(1.22) ⇔
3 5
25
3
x + 4y −
5
z =
15
120
15
25
x −
9
z =
632
hay 3
x + 4y −
5
z =
15
9 25
3
x + 4y −
5
z =
Suy ra z = 9, x = 7, y = 8.
120
15
Suy ra h (1.22) có m®t nghi m (x1, y1, z1) = (7, 8, 9) .
Tìm ”chìa khóa” (x0, y0, z0) thỏa mãn đieu ki n:
1
x
1
—
4
y0
1
+
5
z0 = 0
3x0 + 4y0 − 5z0 = 0
”Chìa khóa” van là (x0, y0, z0) = (27, 136, 125) .
Khi đó
x2 = x1 + mx0
z2 = z1 + mz0.
3
y2 = y1 + my0
(1.22)
0
18
Viết đề tài giá sinh viên – ZALO:0973.287.149-TEAMLUANVAN.COM
Suy ra
x2 = 7 + m.27
y2 = 8 + m.136
z2 = 9 + m.125
Do m ∈ Z+
nên x2 = 7 + 27m; y2 = 8 + 136m; z2 = 9 + 125m.
V y bài toán có vô so nghi m nguyên dương.
19
Viết đề tài giá sinh viên – ZALO:0973.287.149-TEAMLUANVAN.COM
2 2
Chương 2. Các phương pháp giải
phương trình Diophant
Trong chương này, ta xét m®t so phương pháp thông dụng giải phương trình
Diophant tiep c n các đe thi của các kỳ Olympiad Toán hoc nhǎng năm gan đây
(xem [3],[4]).
2.1 Phương pháp phân tích thành nhân tfi
Xét phương trình f (x, y, . . . , z) = m. Giả sả ta có sự phân tích thành tích các
nhân tả bat khả quy dạng
f(x, y, . . . , z) = f1(x, y, . . . , z) . . . fs(x, y, . . . , z).
Khi đó, ta phân tích so nguyên m và nh n được h phương trình tương áng.
Bài toán 2.1. Tìm tat cả các nghi m nguyên(x, y) của phương trình
x2
+ 4 y2
+ 4 − 4 (xy − 4) (x − y) = 9 + 16xy.
Lài giai. Phương trình đã cho tương đương với
x2
y2
+ 4x2
+ 4y2
+ 16 − 4 (xy − 4) (x − y) − 16xy = 9.
Suy ra
x2
y2
− 8xy + 16 + 4 x2
− 2xy + y2
− 4 (xy − 4) (x − y) = 9
⇔ (xy − 4) — 4 (xy − 4) (x − y) + 4 (x − y) = 9
2
⇔ [xy − 4 − 2 (x − y)] = 9
⇔ xy − 4 − 2x + 2y = ±3
⇔ (x + 2) (y − 2) = ±3.
20
Viết đề tài giá sinh viên – ZALO:0973.287.149-TEAMLUANVAN.COM
y − 2 = 3; y − 2 = 1;
x + 2 = 1 x + 2 = 3
y − pq = q;
2
y−
pq = q ;
y − pq = pq; y − pq = p
pq + 1, pq + p2
q2
, p + pq, pq + pq2
, q + pq, pq + p2
q ,
pq + q2
, pq + p2
, pq + p2
q2
, pq + 1
pq + p2
q, pq + q , .
Như v y, ta thu được 8 h sau
x + 2 = 1
x + 1 = −1
y − 2 = 3;
x + 2 = 3
x + 2 = 3
y − 2 = −1;
x + 2 = −1
y − 2 = −3;
y − 2 = −3;
x + 2 = −3
y − 2 = −1;
−
y − 2 = 1.
Giải ra, ta thu được 8 nghi m là
(−1, 5); (1, 3); (−3, −1); (−5, 1); (−1, −1); (−5, 3); (−3, 5); (1, 1).
Bài toán 2.2. Cho p; q là 2 so nguyên to phân bi t. Tìm tat cả các nghi m nguyên
dương (x, y) của phương trình
1 1 1
+ = .
x y pq
Lài giai. Bieu dien phương trình đã cho thành phương trình
(x − pq) (y − pq) = p2
q2
.
Tà h thác
1 1 1
+ =
x y pq
, suy ra
1 1
<
x pq
1 1
và < .
y pq
Như v y, x > pq; y > pq.
Ta nh n được các h sau đây
x − pq = 1
y − pq = p2
q2
;
x − pq = p
y − pq = pq2
;
x − pq = q
y − pq = p2
q
x − pq = p2
x − pq = p2
q
Giải tàng h , ta nh n 9 nghi m
x − pq = pq
x − pq = q2
y − pq = p2
;
x − pq = pq2
x − pq = p2
q2
y − pq = 1.
p2
+ pq, q2
+ pq , (2pq, 2pq) , pq + pq2
, p + pq ,
21
Viết đề tài giá sinh viên – ZALO:0973.287.149-TEAMLUANVAN.COM
u + 2 = −1 u + 2 = −5
Giải các h phương trình trên và trở lại bien x, y, ta có 4 h sau
1 1 1
Bài toán 2.3. Cháng minh rang phương trình dạng + = với n = pα1
. . . pαk
,
x y n 1 k
trong đó p1; p2; . . . ; pk là các so nguyên to, có (2α1 + 1) (2α2 + 1) . . . (αk + 1)
nghi m nguyên dương.
Lài giai. Th t v y, phương trình đã cho tương đương với
(x − n) (y − n) = n2
2α1 2αk
⇔ (x − n) (y − n) = p1 . . . pk .
M t khác, p2α1
. . . p2αk
nên ta có
1 k
(2α1 + 1) (2α2 + 1) . . . (αk + 1) ước.
V y, ta thu được đieu can cháng minh.
Bài toán 2.4 (Poland MO, xem [4]). Tìm tat cả các nghi m nguyên (x, y) của
phương trình
x2
(y − 1) + y2
(x − 1) = 1.
Lài giai. Phương trình đã cho tương đương với
x2
y − x2
+ y2
x − y2
= 1
+
Đ t
u = x + y
v = xy,
⇔ xy (x + y) − (x + y)
phương trình trở thành
2xy = 1.
Như v y, ta có 4 h sau
uv − u2
+ 2v = 1
⇔ (uv + 2v) − u2
− 4 = 5
⇔ (u + 2) (v − u + 2) = 5.
u + 2 = 1
v − u + 2 = 5;
v − u + 2 = −5;
u + 2 = 5
v − u + 2 = 1;
v − u + 2 = −1.
x + y = −1
xy = 2;
x + y = 3
xy = 2;
x + y = −3
xy = −10;
x + y = −7
xy = −10.
Giải ra được 4 nghi m (1; 2) , (2; 1) , (2; −5) , (−5; 2) .
22
Viết đề tài giá sinh viên – ZALO:0973.287.149-TEAMLUANVAN.COM
3 3 3 3 3 3 3 3 3
3 3 3 3 3 3 3 3 3
Bài toán 2.5 (xem [6]). Xác định tat cả các b® ba so nguyên dương (x, y, z) thỏa
mãn phương trình
với so nguyên to p > 3.
x3
+ y3
+ z3
− 3xyz = p,
Lài giai. Vì
và
nên
hay
p = x3
+ y3
+ z3
− 3xyz
= (x + y + z) x2
+ y2
+ z2
− xy − yz − zx
(x + y + z > 1)
x + y + z = p và x2
+ y2
+ z2
− xy − yz − zx = 1
(x − y)2
+ (y − z)2
+ (z − x)2
= 2.
Không mat tính tőng quát, ta có the giả thiet x ≥ y ≥ z ≥ 1.
Neu x > y > z thì x − y ≥ 1, y − z ≥ 1, x − z ≥ 2.
Do đó
(x − y)2
+ (y − z)2
+ (z − x)2
≥ 6 > 2.
Như v y, ta phải xét các trường hợp dưới đây
Trường hợp 1. Khi x = y thì
y = z + 1, x = z + 1 và x + y + z = p.
V y nên z =
p − 2
, x = y =
p + 1
khi p = 3k + 2.
3 3
Trường hợp 2. Khi x > y = z thì
y = z, x = y + 1 và x + y + z = p.
V y nên y = z =
p − 1
, x =
p + 2
khi p = 3k + 1.
3 3
Tóm lại, neu p = 3k + 1 thì phương trình có ba nghi m
p − 1
,
p − 1
,
p + 2
,
p − 1
,
p + 2
,
p − 1
,
p + 2
,
p − 1
,
p − 1
.
Neu p = 3k + 2 thì phương trình có ba nghi m
p − 2
,
p + 1
,
p + 1
,
p + 1
,
p − 2
,
p + 1
,
p + 1
,
p + 1
,
p − 2
.
23
Viết đề tài giá sinh viên – ZALO:0973.287.149-TEAMLUANVAN.COM
9x2
+ 9y2
+ 1 + 9xy + 3x − 3y = 823.
Bài toán 2.6 (xem [7]). Xác định tat cả các so nguyên n đe phương trình sau đây
có nghi m nguyên dương
Lài giai. Ta có
x3
+ y3
+ z3
− 3xyz = n.
Suy ra
x3
+ y3
+ z3
− 3xyz = (x + y + z) x2
+ y2
+ z2
− xy − yz − zx .
x3
+ y3
và
+ z3
— 3xyz = (x + y + z)
(x − y)2
+ (y − z)2
+ (z − x)2
2
x3
+ y3
+ z3
− 3xyz = (x + y + z)3
− 3 (x + y + z) (xy + yz + zx) .
Theo cách viet thá nhat, phương trình đã cho có nghi m nguyên dương khi
n = 3k + 1 và n = 3k + 2. Khi đó (k + 1, k, k) và (k + 1, k + 1, k), với k ≥ 1, là
nghi m dương tương áng.
Neu n chia het cho 3 thì tà cách bieu dien thá hai, x + y + z chia het cho 3 và
như v y n = x3
+ y3
+ z3
− 3xyz chia het cho 9. Ngược lại, neu n = 9k với k ≥ 2,
thì phương trình nh n nghi m (k − 1, k, k + 1) .
Với n = 0, ta có nghi m nguyên dương x = y = z ∈ N∗
.
Với n = 9 phương trình không có nghi m nguyên dương (x, y, z).
Bài toán 2.7 (Russia MO, xem [4]). Tìm tat cả các nghi m nguyên dương của
phương trình
x3
− y3
= xy + 61.
Lài giai. Nhân cả hai ve của phương trình với 27, ta thu được phương trình
tương đương với
(3x)3
+ (−3y)3
+ (−1)3
− 3(3x)(−3y)(−1) = 1642.
Ta có
a3
+ b3
+ c3
− 3abc = (a + b + c) a2
+ b2
+ c2
− ab − bc − ca .
V y nên phương trình đã cho tương đương với
(3x − 3y − 1) 9x2
+ 9y2
+ 1 + 9xy + 3x − 3y = 2.823.
Vì 9x2
+ 9y2
+ 1 + 9xy + 3x − 3y > (3x − 3y − 1) và 823 là so nguyên to
nên ta có
3x − 3y − 1 = 2
Giải h phương trình trên ta thu được nghi m là (6, 5).
24
Viết đề tài giá sinh viên – ZALO:0973.287.149-TEAMLUANVAN.COM
2.2 Phương pháp đong dư
Bang cách xét so dư hai ve của phương trình, phương pháp đong dư thường
dùng đe cháng minh phương trình không có nghi m nguyên ho c dùng đe hạn che
các khả năng của bien. Tà đó, de dàng tìm được nghi m nguyên của phương trình.
Dưới đây là m®t vài ví dụ minh hoa cho phương pháp này.
Bài toán 2.8. Cháng minh rang phương trình
(x + 1)2
+ (x + 2)2
+ · · · + (x + 2001)2
= y2
không có nghi m nguyên.
Lài giai. Đ t x = z − 1001, phương trình đã cho trở thành
(z − 1000)2
+ · · · + (z − 1)2
+ z2
+ (z + 1)2
+ · · · + (z + 1000)2
= y2
hay
2001z2
+ 2 12
+ 22
+ · · · + 10002
= y2
.
Suy ra, phương trình
tương đương với
2000z2
+ 2
1000.1001.2001
6
= y2
,
2001z2
+ 1000.1001.667 = y2
.
Ta thay ve trái ≡ 2 (mod 3) nên không the là so chính phuơng, trong khi
ve phải là m®t so chính phương. V y nên phương trình đã cho không có nghi m
nguyên.
Bài toán 2.9 (Russia MO, xem [4]). Xác định tat cả các c p so nguyên to (p, q)
thỏa mãn phương trình p3
− q5
= (p + q)2
.
Lài giai. De dàng kiem tra p > q.
Neu q = 3 thì p = 7 và ta có c p so nguyên to (7, 3).
Neu q > 3, và do p, q là so nguyên to nên p ≡ 1 hay 2 (mod 3), và q ≡ 1 hay
2 (mod 3).
Neu p ≡ q (mod 3) thì ve trái chia het cho 3, ve phải không chia het cho 3.
Neu p /≡ q (mod 3), thì ve phải chia het cho 3, ve trái không chia het cho 3.
Trong cả hai trường hợp trên, phương trình đeu vô nghi m.
V y nên phương trình đã cho có nghi m duy nhat (p, q) = (7, 3).
25
Viết đề tài giá sinh viên – ZALO:0973.287.149-TEAMLUANVAN.COM
Bài toán 2.10 (Balkan MO, xem [4]). Cháng minh rang phương trình
x5
− y2
= 4
không có nghi m nguyên.
Lài giai. Xét vành Z11. De dàng kiem tra (x5
)2
≡ x10
≡ 0 hay 1 (mod 11) với
moi x ∈ Z. Do v y x5
≡ 0 ho c ±1 (mod 11).
Tà đây suy ra x5
−4 ≡ 6 ho c 7 ho c 8 (mod 11). Vì th ng dư b c hai modulo
11 chỉ có the là 0, 1, 3, 4, 5, 9 nên phương trình đã cho không the có nghi m nguyên.
Bài toán 2.11. Tìm tat cả các nghi m nguyên của phương trình
x3
+ y3
= z6
+ 3.
Lài giai. Neu phương trình có nghi m trong Z thì nó cũng có nghi m trong Z7.
Khi đó ton tại x, y, z ∈ Z7 đe x3
+ y3
= z6
+ 3.
Trong Z7 có
0
3
= 0, 1
3
= 1, 2
3
= 1, 3
3
= −1, 4
3
= −1, 5
3
= −1, 6
3
= −1.
V y nên x3
hay y3
chỉ có the là 0 ho c 1 ho c −1.
Bang kiem tra trực tiep, ta thay x3
+ y3
chỉ có the là 0, 1, 2, −1, −2. Nhưng
z6
+ 3 chỉ có the là 0 + 3 = 3 ho c 1 + 3 = 4. Đieu này cháng tỏ phương trình vô
nghi m.
2.3 Phương pháp đánh giá
Phương pháp đánh giá là dùng các bat đȁng thác phù hợp đe hạn che lại các
khoảng đang xét. Tà đó, ta tìm được nghi m nguyên của phương trình m®t cách
đơn giản hơn.
Bài toán 2.12. Giải phương trình x3
+ y3
= (x + y)2
với x, y ∈ Z.
Lài giai. Xét x + y = 0. Khi đó x = k, y = −k với k ∈ Z đeu là nghi m. Xét
x + y 0.
De dàng ta có
hay
x2
− xy + y2
= x + y
(x − y)2
+ (x − 1)2
+ (y − 1)2
= 2.
Tà đây suy ra |x − 1|, |y − 1| ≤ 1. Như v y, x, y ∈ {0, 1, 2}.
Kiem tra (0, 1), (1, 0), (1, 2), (2, 1), (2, 2) và (k, −k) với k ∈ Z là nghi m phương
trình đã cho.
26
Viết đề tài giá sinh viên – ZALO:0973.287.149-TEAMLUANVAN.COM
−
≥
Bài toán 2.13 (Russia MO, xem [4]). Giải phương trình
x, y, z ∈ N∗
.
1 1 1 3
+ + = ,
x y z 5
3 3
Lài giai. Không hạn che có the giả thiet 2 ≤ x ≤ y ≤ z. Khi đó và suy
x 5
ra x ∈ {2, 3, 4, 5}.
1 1
Neu x = 2 thì + =
y z
1 2
10
. De thay 11 ≤ y ≤ z. Khi đó,
y
≥
100
1
suy ra
10
y ∈ {11, 12, . . . , 20} và z = 10 +
y 10
. Vì z nguyên dương nên (y − 10) | 100.
Ta thu được các nghi m
(2, 11, 110), (2, 12, 60), (2, 14, 35), (2, 15, 30), (2, 20, 20).
Neu x = 3 thì
1 1 4
+ = .
y z 15
Làm tương tự như trường hợp trên ta có y ∈ {3, 4, 5, 6, 7}.
De dàng kiem tra đe được nghi m (3, 4, 60), (3, 5, 15), (3, 6, 10).
1 1 7
Neu x = 4 thì
y
+
z
=
20
. Tương tự ta có y ∈ {4, 5}.
De dàng kiem tra đe được nghi m (4, 4, 10).
Neu x = 5 thì
1 1 2
+ =
y z 5
. Tương tự ta có nghi m (5, 5, 5).
Bài toán 2.14. Giải phương trình nghi m nguyên dương
x2
+ y2
+ z2
+ 2xy + 2x(z − 1) + 2y(z + 1) = w2
.
Lài giai. Ta có
(x + y + z + 1)2
= x2
+ y2
+ z2
+ 2xy + 2x(z + 1) + 2y(z + 1) + 2z + 1.
(x + y + z − 1)2
= x2
+ y2
+ z2
+ 2xy + 2x(z − 1) + 2y(z − 1) − 2z + 1.
Suy ra
Do đó
(x + y + z − 1)2
< w2
< (x + y + z + 1)2
.
x2
+ y2
+ z2
+ 2xy + 2x(z − 1) + 2y(z + 1) = (x + y + z)2
.
Suy ra x = y. Đ t x = y = m; z = n thì w = 2m + n, trong đó m; n là các so
nguyên dương.
V y nghi m phương trình đã cho là các b® so (m; m; n; 2m +n), với m; n là các
so nguyên dương.
27
Viết đề tài giá sinh viên – ZALO:0973.287.149-TEAMLUANVAN.COM
z
3
1
x y z
x y
x y 3
y 3 x
x y 2
đ t y = tx, với t ∈ Z. V y x3
= x2
+ 2t2
x2
hay x = 2t2
+ 1, u = t
Lài giai. Với z = −y ta có x3
= x2
+ 2y2
. Khi x = 0 thì y = 0. Khi x /= 0 ta
2t2
+ 1 .
Bài toán 2.15. Giải phương trình nghi m nguyên dương
1 +
1
1 +
1
1 +
1
= 2.
Lài giai. Không hạn che có the giả thiet x ≥ y ≥ z ≥ 1.
Khi đó, ta có 1 + ≥ 2 và suy ra z ≤ 3. Xét các trường hợp sau đây:
Trường hợp z = 1. Khi đó 1 +
1
1 +
1
= 1 vô nghi m.
Trường hợp z = 2. De dàng chỉ ra phương trình
1 +
1
1 +
1
=
4
và suy ra 1 +
1 2
≥
4
. De dàng suy ra y < 7. Vì 1 +
1
> 1 nên y > 3. Ta de
dàng nh n được nghi m (7, 6, 2), (9, 5, 2), (15, 4, 2).
Trường hợp z = 3. De dàng có 1 +
1
1 +
1
=
3
và suy ra 3 = z ≤ y < 5.
De dàng kiem tra đe được nghi m (8, 3, 3), (5, 4, 3).
Tóm lại, ta nh n được các nghi m (7, 6, 2), (9, 5, 2), (15, 4, 2), (8, 3, 3) và (5, 4, 3).
2.4 Phương pháp tham so hóa
M®t so phương trình nghi m nguyên f(x1; x2; . . . ; xn) = 0 có vô so nghi m và
ta không the li t kê được het tat cả các nghi m đó. Khi đó, ta sě tìm nghi m này
dưới dạng tham so như sau:
x1 = g1(k1, k2, . . . , kl), x2 = g2(k1, k2, . . . , kl), . . . , xn = gn(k1, k2, . . . , kl),
trong đó g1, g2, . . . , gn là hàm l bien, k1, . . . , kl ∈ Z.
Điem mạnh của phương pháp này là có the dùng đe cháng minh phương trình
nghi m nguyên có vô so nghi m.
Sau đây ta sě xét m®t so bài toán minh hoa cho phương pháp này.
Bài toán 2.16. Cháng minh rang, có vô hạn b® ba (x, y, z) ∈ Z3
thỏa mãn phương
trình
x3
+ y3
+ z3
= x2
+ y2
+ z2
.
Với t ∈ Z, ta nh n được vô so nghi m 2t2
+ 1, t 2t2
+ 1 , −t 2t2
+ 1 .
28
Viết đề tài giá sinh viên – ZALO:0973.287.149-TEAMLUANVAN.COM
∈
≥
Bài toán 2.17. Cháng minh rang, phương trình x2
= y3
+ z5
có vô so nghi m
nguyên dương (x, y, z).
Lài giai. Xét
Khi đó
y = t7
(t + 1)5
, z = t4
(t + 1)3
.
x2
= t21
(t + 1)15
+ t20
(t + 1)15
= t20
(t + 1)16
.
Giải ra, ta thu được
x = t10
(t + 1)8
.
Do v y, phương trình đã cho có nhieu vô so nghi m nguyên dương
x = t10
(t + 1)8
, y = t7
(t + 1)5
, z = t4
(t + 1)3
.
Bài toán 2.18. Cháng minh rang, phương trình x2
+ y2
= 13z
có vô so nghi m
nguyên dương (x, y, z).
Lài giai. Vì 22
+ 32
= 13 nên khi chon x = 2.13n
, y = 3.13n
, ta có
13z
= 132n+1
.
Suy ra
z = 2n + 1.
Do v y, phương trình có nhieu vô so nghi m nguyên dương
x = 2.13n
, y = 3.13n
, z = 2n + 1, n ∈ N.
Bài toán 2.19. Cháng minh rang, phương trình xn
+ yn
= zn−1
có vô so nghi m
nguyên dương (x, y, z) khi n ≥ 3.
Lài giai. Đ t x = ty, t N. Khi đó yn
(tn
+ 1) = zn−1
.
Chon y = (tn
+ 1)n−2
ta có z = (tn
+ 1)n−1
và x = t (tn
+ 1)n−2
.
Do v y, phương trình có nhieu vô so nghi m nguyên dương
x = t (tn
+ 1)n−2
, 13n
, y = (tn
+ 1)n−2
, z = (tn
+ 1)n−1
, t ∈ N∗.
Bài toán 2.20. Cháng minh rang, phương trình 2x
+ 1 = xy có vô so nghi m
nguyên dương (x, y).
Lài giai. Trước tiên ta cháng minh 23n
+ 1 chia het cho 3n
với moi n 0. Với
n = 0, ket lu n là hien nhiên. Giả sả 23n
+ 1 chia het cho 3n
. Với n + 1 ta bieu
dien
23n+1
+ 1 = 23n
+ 1 22.3n
23n
+ 1 .
−
29
Viết đề tài giá sinh viên – ZALO:0973.287.149-TEAMLUANVAN.COM
Xét thàa so 22.3n
−23n
+1 = 23n
+1
2
−3.23n
chia het cho 3. Do v y 23n+1
+1 chia
Thàa so 23n
+ 1 chia het cho 3n
theo giả thiet quy nạp.
het cho 3n+1
. Phương trình đã cho có vô so nghi m nguyên dương
với moi so nguyên n ≥ 0.
3n
,
23n
+ 1
3n
Bài toán 2.21. Cháng minh rang, với so nguyên n ≥ 2 phương trình xn
+ yn
=
zn+1
có vô so nghi m nguyên dương (x, y, z).
Lài giai. Phương trình đã cho có vô so nghi m nguyên dương dạng (tn
, t(tn
+
1),tn
+ 1) với moi so nguyên t ≥ 1.
Bài toán 2.22. Tìm các b® ba so nguyên dương (x, y, z) thỏa mãn
1 1 1
+ = .
x y z
xy
Lài giai. Ta có z =
x + y
. Giả sả d = (x, y). Đ t x = dm, y = dn. Khi đó
dmn
(m, n) = 1 và (mn, m + n) = 1. Do v y, z =
xy m + n
và suy ra m + n là ước của
d. Đ t t = . Ta có x = tm(m + n), y = tn(m + n), z = tmn với m, n, t ∈ N∗
z
và (m, n) = 1.
Nh n xét 2.3. 1. Neu a, b, c là các so đôi m®t nguyên to cùng nhau và thỏa mãn
đieu ki n
1 1 1
+ =
a b c
thì a + b là m®t so chính phương.
1 1 1
2. Neu a, b, c là các so nguyên dương và thỏa mãn đieu ki n +
a b
a2
+ b2
+ c2
là m®t so chính phương.
Th t v y,
= thì
c
a2
+ b2
+ c2
= k2
h
m2
(m + n)2
+ n2
(m + n)2
+ m2
n2
i
= k2
h
(m + n)4
− 2mn (m + n)2
+ m2
n2
i
= k2
h
(m + n)2
− mn
i2
Bài toán 2.23. Xác định tat cả các c p so nguyên (x, y) thỏa mãn đieu ki n
x3
+ y3
+ 21xy = 343.
Lài giai. Ta có
x3
+ y3
+ (−7)3
− 3(−7)xy = 0.
30
Viết đề tài giá sinh viên – ZALO:0973.287.149-TEAMLUANVAN.COM
Suy ra
(x + y − 7)
h
(x − y)2
+ (y + 7)2
+ (x + 7)2
i
= 0.
Do v y, ta nh n được nghi m (−7, −7) và (a, −7 − a) với a ∈ Z.
Bài toán 2.24. Xác định tat cả các c p so nguyên (x, y) thỏa mãn đieu ki n
x3
+ y3
+ 6xy = 8.
Lài giai. Ta có
Suy ra
x3
+ y3
+ (−2)3
− 3(−2)xy = 0.
(x + y − 2) x2
+ y2
+ 22
− xy + 2x + 2y = 0.
Do v y, ta nh n được nghi m (−2, −2) và (a, 2 − a) với a ∈ Z.
2.5 Phương pháp quy nạp toán hoc
Ta xét m®t so quy tac sả dụng quy nạp toán hoc.
- Nguyên lý quy nạp thá nhat.
M nh đe P(n) là đúng với moi so tự nhiên n ≥ α neu
(1) Bư c cơ s : M nh đe P(α) đúng.
(2) Bư c quy nạp: Neu P(n) đúng thì P(n + 1) cũng đúng, trong đó n ≥ α.
Như v y, sau khi kiem tra bước cơ sở và cháng minh tính đúng của m nh đe
P (n + 1) dưới giả thiet m nh đe P (n) đúng, ta ket lu n P (n) đúng cho moi so tự
nhiên n ≥ α.
- Nguyên lý quy nạp thá hai.
M nh đe P(n) là đúng với moi so tự nhiên n ≥ α neu
(1) Bư c cơ s : M nh đe P(α) đúng.
(2) Bư c quy nạp: M nh đe P (n + 1) là đúng khi các m nh đe P (α), P (α + 1),
. . . , P(n) đeu đúng, trong đó n ≥ α, n ∈ N.
Bài toán 2.25 (Belarussia MO, xem [4]). Cháng minh rang với moi so nguyên
n ≥ 3, phương trình sau luôn có nghi m (x; y) trong đó x, y là các so nguyên
dương lẻ
7x2
+ y2
= 2n
.
31
Viết đề tài giá sinh viên – ZALO:0973.287.149-TEAMLUANVAN.COM
+
Lài giai. Ta sě cháng minh, với moi n ≥ 3, ton tại so nguyên dương lẻ xn; yn
thỏa mãn đieu ki n 7x2
+ y2
= 2n
.
n n
Với n = 3, de dàng tìm được nghi m x3 = y3 = 1.
Giả sả ton tại so nguyên dương lẻ xn; yn thỏa mãn đieu ki n
7x2
+ y2
= 2n
.
n n
Ta phải cháng minh ton tại so nguyên dương lẻ xn+1; yn+1 thỏa mãn đieu ki n
Th t v y,
2
n+1
2
n+1 = 2n+1
.
xn ± yn
2
7xn ± yn
2
= 2(7x2
+ y2
) = 2n+1
.
2 2 n n
Rõ ràng m®t trong hai so
xn + yn
2
và
|xn − yn|
2
là so lẻ vì
xn + yn
+
|xn − yn|
= x
ho c
2 2 n
xn + yn
+
|xn − yn|
= y .
2 2 n
M t khác, xn; yn là các so lẻ.
Neu
xn + yn
2
là lẻ thì
xn − yn
2
chȁn. Khi đó
là so lẻ. Do đó ta chon
7xn − yn
2
= 3xn +
xn − yn
2
xn+1 =
xn + yn
; y
2
n+1 =
7xn − yn
.
2
Neu
xn − yn
2
là lẻ thì
xn + yn
2
chȁn. Khi đó
là so lẻ. Do đó ta chon
7xn + yn
2
= 3xn +
xn + yn
2
|xn − yn|
; y
2
n+1 =
7xn + yn
.
2
7x + y
7
32
Viết đề tài giá sinh viên – ZALO:0973.287.149-TEAMLUANVAN.COM
k+2 k+2 k+2 k k k
k
Bài toán 2.26 (xem [4]). Cháng minh rang với moi so nguyên dương n, phương
trình sau luôn có nghi m (x; y) trong đó x, y là các so nguyên dương
x2
+ y2
+ z2
= 59n
.
Lài giai. Ta sě sả dụng phương pháp quy nạp với b c s = 2; n0 = 1.
De thay 12
+ 32
+ 72
= 59 và 142
+ 392
+ 422
= 592
nên (x1; y1; z1) = (1; 3; 7)
và (x2; y2; z2) = (14; 39; 42) là 2 nghi m của phương trình.
Giả sả với moi n = k, (k ≥ 1) phương trình có nghi m (xk; yk; zk). Tác là
x2
+ y2
+ z2
= 59k
.
k k k
Đ t xk+2 = 59xn, yk+2 = 59yn, zn+2 = 59zn, ∀k ≥ 1. Ta có
x2
+ y2
+ z2
= 592
x2
+ y2
+ z2
= 59k+2
.
Do đó (xk+2; yk+2; zk+2) xác định như trên là nghi m của phương trình.
Bài toán 2.27. Cháng minh rang với moi so nguyên n ≥ 3, phương trình sau
luôn có nghi m (x1; x2; . . . .; xn) là các so nguyên dương đôi m®t khác nhau
1 1
+
x1 x2
1 1
1
+ · · · +
x
1
= 1.
Lài giai. Với n = 3, ta có + +
2 3 6
= 1.
Giả sả phương trình có nghi m đen n = k, (k ≥ 3), tác là
1 1
+
x1 x2
1
+ · · · +
x
= 1,
với x1; x2; . . . .; xk là các so nguyên dương đôi m®t khác nhau.
Suy ra
Do đó
1
2x1
1
1
+
2x2
1
1 1
+ · · · +
2x
=
2
.
1 1
+
2 2x1
+
2x2
+ · · · +
2x
= 1,
với 2; 2x1; 2x2; . . . .; 2xk là các so nguyên dương đôi m®t khác nhau.
V y nên (2; 2x1; 2x2; . . . .; 2xk) là nghi m can tìm.
n
k
k
33
Viết đề tài giá sinh viên – ZALO:0973.287.149-TEAMLUANVAN.COM
2.6 Phương pháp xuong thang
Fermat đã dùng phương pháp xuong thang đe cháng minh phương trình x4
+
y4
= z4
không có nghi m nguyên dương.
Cơ sở của phương pháp xuong thang là tính sap thá tự tot của N (và Nk
, tích
Đe các của k phiên bản N): M®t t p con khác rong bat kỳ của N đeu có phan tả
nhỏ nhat.
Đe cháng minh m®t phương trình là vô nghi m, ta giả sả ngược lại rang t p R
các nghi m nguyên (tự nhiên, nguyên dương) của phương trình khác rong. Ta đưa
ra m®t thá tự tot trên R và giả sả α0 là nghi m nhỏ nhat (theo thá tự nêu trên).
Neu bang cách nào đó ta xây dựng được nghi m a1 nhỏ hơn α0 thì chúng ta sě
đi đen mâu thuan. Mâu thuan này cháng tỏ đieu giả sả là sai và như v y phương
trình đã cho vô nghi m.
Bài toán 2.28. Cháng minh rang phương trình x4
+ y4
= z2
không có nghi m
nguyên dương.
Lài giai. Giả sả rang phương trình đã cho có nghi m nguyên dương. Giả sả
(x, y) = d, tác là x = da, y = db, trong đó (a, b) = 1. Khi đó a4
+ b4
= (z/d2
)2
.
Giả sả z = d2
c, trong đó c ∈ Q, khi đó
a4
+ b4
= c2
(2.1)
Vì c ∈ Q, c2
∈ N nên c ∈ N+
. Trong tat cả các nghi m của phương trình (2.1),
chon nghi m có c nhỏ nhat. Ta có (a2
)2
+ (b2
)2
= c2
trong đó (a, b) = 1, suy
ra (a2
, b2
) = 1, tác là (a2
, b2
, c) là b® ba Pythagore nguyên thủy. Ton tại các so
nguyên dương m, n sao cho a2
= m2
− n2
, b2
= 2mn, c = m2
+ n2
, trong đó m, n
khác tính chȁn lẻ, m > n và (m, n) = 1, nghĩa là a2
= m2
− n2
lẻ. Giả sả m chȁn,
n lẻ. Khi đó n2
, a2
chia 4 dư 1, nghĩa là m2
= n2
+ a2
chia 4 dư 2 mâu thuan. V y
m lẻ, n chȁn, ngòai ra (a, n, m) l p thành b® Pythagore nguyên thủy, do đó ton
tại p, q ∈ N+
sao cho a = p2
− q2
, n = 2pq, m = p2
+ q2
, trong đó p, q khác tính
chȁn lẻ, p > q và (p, q) = 1, ngòai ra b2
= 2mn, nghĩa là b2
= 4pq(p2
+ q2
), suy ra
b = 2h, h ∈ N+
, khi đó
h2
= pq(p2
+ q2
). (2.2)
Giả sả rang ton tại so nguyên to r chia het pq, p2
+ q2
. Vì r chia het pq nên không
mat tőng quát, có the giả sả r chia het p, khi đó r chia het (p2
+ q2
) − p2
= q2
, suy
ra r chia het q, mâu thuan vì (p, q) = 1. V y (pq, p2
+ q2
) = 1, như the, tà (2.2),
theo định lý ??, ta có pq = s2
, p2
+ q2
= t2
với s, t ∈ N+
. Vì pq = s2
, (p, q) = 1
nên p = u2
, q = v2
với u, v ∈ N+
, nghĩa là (u2
)2
+ (v2
)2
= t2
hay u4
+ v4
= t2
,
trong đó c = m2
+ n2
> m = p2
+ q2
= t2
> t, mâu thuan với cách chon c. Như
v y đieu giả sả ban đau là sai và ta có đieu phải cháng minh.
34
Viết đề tài giá sinh viên – ZALO:0973.287.149-TEAMLUANVAN.COM
Bài toán 2.29. Tìm các nghi m nguyên của phương trình
x3
− 3y3
= 9z3
.
Lài giai. Hien nhiên 3|x. Đ t x = 3x1, với x1 nguyên. Thay vào phương trình
ban đau roi chia cả hai ve cho 3, ta thu được
9x1
3
− y3
= 3z3
.
Suy ra 3|y. Đ t y = 3y1, với y1 nguyên. Thay vào phương trình trên roi chia cả
hai ve cho 3, ta được
3x1
3
+ 9y1
3
= z3
.
Suy ra 3|z. Đ t z = 3z1, với z1 nguyên. Thay vào phương trình trên roi chia cả
hai ve cho 3 ta được
x1
3
− 3y1
3
= 9z1
3
.
Như v y neu (x, y, z) là nghi m của phương trình đã cho thì (x1, y1, z1) cũng là
nghi m, trong đó,
x = 3x1, y = 3y1, z = 3z1.
L p lu n tương tự như trên thì (x2, y2, z2) cũng là nghi m, trong đó
x1 = 3x2, y1 = 3y2, z1 = 3z2.
Cá tiep tục như v y dan đen x, y, z đeu chia het cho 3k
, với k là so tự nhiên
tùy ý. Đieu này chỉ xảy ra khi x = y = z = 0.
Đó là nghi m nguyên duy nhat của phương trình đã cho.
Nh n xét 2.4. Trong ví dụ trên, neu yêu cau tìm các nghi m nguyên dương của
phương trình x3
− 3y3
= 9z3
, ta có the dùng nguyên tac cực hạn đe cháng minh
phương trình không có nghi m nguyên dương như sau.
Giả sả (x0, y0, z0) là nghi m nguyên dương của phương trình, trong đó x0 là giá
trị nguyên dương nhỏ nhat trong các giá trị. M t khác, x có the nh n được.
L p lu n như trong các giải trên, ta thu được (x1, y1, z1) cũng là nghi m nguyên
dương của phương trình.
M t khác, x0 = 2x1, tác là x0 > x1. Đieu này trái với giả thiet x0 là so nguyên
dương nhỏ nhat trong các giá trị nh n được của x.
V y nên phương trình đã cho không có nghi m nguyên dương.
Bài toán 2.30 (Korea MO, xem [4]). Tìm các nghi m nguyên của phương trình
x2
+ y2
+ z2
= 2xyz.
Lài giai. Do ve phải chia het cho 2 nên x2
+ y2
+ z2
chia het cho 2. Có 2 khả
năng xảy ra.
35
Viết đề tài giá sinh viên – ZALO:0973.287.149-TEAMLUANVAN.COM
Neu trong ba so x, y, z có m®t so chȁn, hai so lẻ. Giả sả x chȁn, y, z lẻ thì
x2
+ y2
+ z2
chia cho 4 dư 2 còn 2xyz chia het cho 4. nên khă năng này loại.
V y ba so x, y, z phải cùng chȁn. Đ t x = 2x1, y = 2y1, z = 2z1, với x1, y1, z1
là các so nguyên. Thay vào phương trình ban đau roi chia cả hai ve cho 4 ta có
x1
2
+ y1
2
+ z1
2
= 4x1y1z1.
L p lu n tuơng tự, ta thu được x1, y1, z1 cũng là các so chȁn. Cá tiep tục như
v y dan đen x, y, z đeu chia het cho 2k
, với k là so tự nhiên tùy ý. Đieu này chỉ
xảy ra khi x = y = z = 0.
Đó là nghi m nguyên duy nhat của phương trình đã cho.
Bài toán 2.31. Tìm các nghi m nguyên của phương trình
x4
+ y4
+ z4
= 9u4
.
Lài giai. Neu u = 0 thì x = y = z = 0. Ta sě cháng minh không còn nghi m
nào khác nghi m (0, 0, 0, 0).
Giả sả x, y, z là các so nguyên thỏa mãn phương trình và u 0.
Neu u không chia het cho 5 thì theo Định lý Fermat nhỏ ta có u4
≡ 1 (mod 5)
nên 9u4
≡ 4 (mod 5) nhưng x4
, y4
, z4
≡ 0 ho c 1 (mod 5) nên khả năng này
không xảy ra. V y u phải chia het cho 5. Đ t u = 5u1, với u1 nguyên, ta có
x4
+ y4
+ z4
≡ 0 (mod 5).
Theo Định lý Fermat nhỏ thì x, y, z chia het cho 5. Đ t x = 5x1, y = 5y1, z =
5z1, với x1, y1, z1 là các so nguyên. Thay vào phương trình ban đau roi chia cả hai
ve cho 54
, ta có
x1
4
+ y1
4
+ z1
4
= 9u1
4
.
Như v y neu (x, y, z) là nghi m của phương trình đã cho thì (x1, y1, z1) cũng là
nghi m. Trong đó,
x = 5x1, y = 5y1, z = 5z1, u = 5u1.
L p lu n tương tự như trên thì (x2, y2, z2, u2) cũng là nghi m, trong đó
x1 = 5x2, y1 = 5y2, z1 = 5z2, u1 = 5u2.
Cá tiep tục như v y dan đen x, y, z, u đeu chia het cho 5k
, với k là so tự nhiên
tùy ý. Đieu này chỉ xảy ra khi x = y = z = u = 0.
Đó là nghi m nguyên duy nhat của phương trình đã cho.
Bài toán 2.32. Tìm các nghi m nguyên dương của phương trình
x2
− y2
= 2xyz.
36
Viết đề tài giá sinh viên – ZALO:0973.287.149-TEAMLUANVAN.COM
xy zx
Lài giai. Giả sả x0, y0, z0 là các so nguyên dương thỏa mãn phương trình, trong
đó x0 là giá trị nguyên dương nhỏ nhat trong các giá trị M t khác, x có the nh n
được.
Đ t d = x0y0. Neu d = 1 thì x0 = y0 = 1, z0 = 0, (loại).
V y d > 1. Goi p là ước nguyên to của d.
Ta có
(x0 + y0)(x0 − y0) = x0
2
− y0
2
= 2x0y0z0 ≡ 0 (mod p)).
Suy ra, x0 ≡ y0 (mod p) ho c x0 ≡ −y0 (mod p). Vì x0y0 chia het cho p nên
x0 ≡ y0 ≡ 0 (mod p).
Đ t x0 = px1, y0 = py1, với x1, y1 là các so nguyên dương. Thay vào phương
trình ban đau roi chia cả hai ve cho p2
, ta có
x1
2
− y1
2
= 2x1y1z0.
Suy ra, (x1, y1, z0) cũng là nghi m nguyên dương của phương trình. M t khác,
x0 = px1, tác là x0 > x1. Đieu này trái với giả thiet x0 là so nguyên dương nhỏ
nhat trong các giá trị nh n được của x.
V y nên phương trình đã cho không có nghi m nguyên dương.
Nh n xét 2.5. Đe sả dụng phương pháp xuong thang, người ta can đen nhǎng
điem cực biên của t p hợp so đã cho.
Bài toán 2.33. Tìm tat cả các so nguyên to p, q, r thỏa mãn phương trình
pq
+ qp
= r.
Lài giai. Nh n xét rang trong 3 so nguyên to p, q, r toi thieu phải có m®t so
nguyên to chȁn. So đó phải bang 2.
Vì p, q là hai so nguyên to nên r /= 2. Không mat tính tőng quát, có the giả
thiet q = 2. V y p2
+2p
= r. Vì r là so nguyên to nên p phải là só lẻ. Neu p =
/ 3 thì
p2
+ 2p
≡ (±1)2
+ (−1)p
(mod 3) ≡ 1 − 1 (mod 3). V y r chia het cho 3, nhưng
p2
+ 2p
= 3 chỉ thỏa mãn cho p = 1 : mâu thuan. Tà đây suy ra p = 3, r = 17.
Bài toán 2.34. Tìm nghi m nguyên dương của phương trình
x + y + z = xyz (2.3)
Lài giai. Vì vai trò của x, y, z như nhau nên ta có the giả sả 1 ≤ x ≤ y ≤ z. Tà
1 1
(2.3) suy ra 1 = +
1 3
+ ≤ , suy ra x2
≤ 3 suy ra x = 1.
Thay x = 1 vào (2.3), ta được 1 + y + z = yz hay
(y − 1)(z − 1) = 2 = 1.2.
yz x2
37
Viết đề tài giá sinh viên – ZALO:0973.287.149-TEAMLUANVAN.COM
.
z x y x x y z x z
Do đó y = 2, z = 3 (vì y − 1 ≤ z − 1).
V y nghi m của phương trình (2.3) là (1; 2;3) và các hoán vị của nó.
Nh n xét 2.6. đây vì chỉ có hǎu hạn so nguyên dương và chúng có vai trò như
nhau nên ta có the giả sả x là so nguyên dương bé nhat.
Bài toán 2.35. Tìm tat cả các so tự nhiên n sao cho ton tại các so tự nhiên
(x, y, z) có tính chat (x + y + z)2
chia het cho nxyz.
(x + y + z)2
Lài giai. Đ t F (x; y; z) = .
xyz
Giả sả n là so nguyên dương nào đó sao cho ton tại b® so nguyên dương x, y, z
đe n = F (x; y; z).
Ta giả sả (x, y, z) là m®t b® so như v y, đong thời và z đạt cực tieu trong tat
cả các b® thỏa mãn đieu ki n bài ra.
Ta có
n(xyz) = (x + y + z)2
= (x + y)2
+ 2z(x + y) + z2
(2.4)
Tà đó, suy ra z (x + y)2
.
M t khác neu (x, y, z) thỏa mãn đieu ki n đòi hỏi thì
F
(x + y)2
!
x, y;
(x + y)2
z
= n,
do b® so x, y;
z
thỏa mãn (2.4).
(x + y)2
Như v y neu z > x + y thì
z
z. Tà đó, suy ra x + y ≥ z.
Ta có
> z, trái với giả thiet ve tính cực tieu của
x y z
n = + +
yz xz xy
2 2 2
+ + +
x y z
≤
1
+
1
+
1
+
1
+
2
+
2
+
2 7 3
≤ + .
V y n lớn nhat khi z = 1, suy ra x = y = z = 1, tác là n 7.
7 3
Giả sả ngược lại, 7 ≤
x
+
z
.
7 3 7 3
Khi đó x ≥ 2, suy ra
x
+
z
≤ + = 5, vô lý.
2 3
Tính toán trực tiep, ta được
F (9; 9; 9) = 1, F (4, 4, 8) = 2, F (3; 3; 3) = 3, F(2; 2; 2) = 4,
!
38
Viết đề tài giá sinh viên – ZALO:0973.287.149-TEAMLUANVAN.COM
F(1; 4; 5) = 5, F(1; 2; 6) = 6, F(1; 1; 2) = 8, F(1; 1; 1) = 9.
V y các giá trị n can tìm là 1; 2; 3; 4; 5; 6; 8; 9.
Nh n xét 2.7. - Ta chỉ can tìm các so nguyên dương n đe dau đȁng thác xảy ra
m c dù yêu cau bài toán là (x + y + z)2
chia het cho nxyz, vì neu (x + y + z)2
=
k(nxyz) thì ta thay kn bởi n′.
- Vì trong t p hợp các so nguyên dương bao giờ cũng ton tại so nguyên dương
bé nhat nên ta có the giả sả z nhỏ nhat.
Bài toán 2.36. Cho n là so tự nhiên, n ≥ 3. Tìm so tự nhiên k nhỏ nhat có tính
chat sau:
Với moi b® n so tự nhiên d1, d2, . . . , dn nguyên to cùng nhau sao cho tőng
d1 + d2 + · · · + dn chia het cho moi di với 1 ≤ i ≤ n, ta có (d1 + d2 + · · · + dn)k
chia het cho tích d1d2 . . . dn.
Lài giai. Giả sả p là so nguyên to, p là ước của tích d1d2 . . . dn.
Goi s là so lớn nhat sao cho ton tại j, trong đó 1 ≤ j ≤ n đe dj chia het cho ps
.
Vì p1 + p2 + · · · + pn chia het cho dj nên d1 + d2 + · · · + dn chia het cho ps
, tà
đó suy ra d1 + d2 + · · · + dn)n−1
chia het cho ps(n−2)
.
Vì d1, d2, . . . , dn không có ước chung khác 1 nên phải ton tại dl nào đó không
chia het cho p.
Như v y lũy thàa cao nhat của p chia het tích d1d2 . . . dn không vượt quá
s(n − 2).
Vì p được chon là ước nguyên to tùy ý của tích nên ta có
(d1 + d2 + · · · + dn)n−2
.d1d2 . . . dn.
Như v y so k nhỏ nhat can tìm thỏa mãn đieu ki n k ≤ n − 2.
Ta cháng tỏ k = n − 2, tác là neu ton tại d1, d2, . . . , dn thỏa mãn giả thiet của
bài toán sao cho (d1 + d2 + · · · + dn)n−3
không chia het cho tích d1d2 . . . dn.
Ta lay d1 = 1, d2 = n − 1, di = n, 3 ≤
.
i ≤ n.
Khi đó d1 + d2 + · · · + dn = n(n − 1).di ∀1 ≤ i ≤ n.
M t khác, d1d2 . . . dn = (n − 1) nn−2
.
Do (n, n−1) = 1 nên lũy thàa bé hơn (n−2) của tőng d1 +d2 +· · ·+dn không
the chia het cho tích d1, d2, . . . , dn.
Bài toán 2.37. (IMO 2002) Cho n là so nguyên dương lớn hơn 1. Tat cả các ước
nguyên dương của n là d1; d2; . . . ; dk với 1 = d1 < d2 < · · · < dk = n.
Đ t d = d1d2 + d2d3 + · · · + dk−1dk.
a) Cháng minh rang D < n2
.
39
Viết đề tài giá sinh viên – ZALO:0973.287.149-TEAMLUANVAN.COM
n2 n2 n2
b) Xác định tat cả n sao cho D là m®t ước so của n2
.
Lài giai.
a) Nh n xét rang
d1dk = d2dk−1 = · · · = dl.dk−l+1 = · · · = dkd1 = n.
Do đó
D < n2
⇔ d1d2 + d2d3 + · · · + dk−1dk < n
⇔
d1d2
+
d2d3
+ · · · +
dk−1dk
< 1
⇔
d1d2
+
d2d3
+ · · · +
dk−1dk
< 1
d1dk.d2dk−1 d2dk−1.d3dk−2 d2dk−1.dk.d1
⇔
M t khác, thì
1
d1d2
1
+
d2d3
+ · · · +
d
1
k−1 .dk
< 1. (2.5)
Ve trái (2.5) <
1
− d +
1
− d + · · · +
1
−
1
= 1 −
1
= 1.
d1
2
d2
3
dk−1 dk dk
Vì v y bat đȁng thác (2.5) đúng và D < n2
.
b) Neu n = p là so nguyên to thì D = n là ước của n2
.
Neu n là hợp so và goi p là ước so nguyên to nhỏ nhat của n thì dk = n, d2 =
p, dk−1 =
n
và do đó D >
p
n2
p
= dk−1dk.
2 n2
2
Theo ket quả câu a, ta có n > D >
p
nên D ko the là ước so của n được.
Bài toán 2.38 (Hungari MO 2000). Tìm tat cả các so nguyên to p sao cho với so
p đó, ton tại các so nguyên dương n, x, y thỏa mãn đieu ki n pn
= x3
+ y3
.
Lài giai. Các so nguyên to tìm được là 2 và 3.
Th t v y, ta có 21
= 13
+ 13
và 32
= 12
+ 23
.
Xét p > 3. Giả sả ton tại các so nguyên dương n, x, y thỏa mãn đieu ki n
pn
= x3
+ y3
.
Khi đó, ta chon n, x, y sao cho so n nhỏ nhat trong tat cả các b® ba so như the
ton tại. Vì p /= 2 nên (x; y) (1; 1).
Vì v y x2
− xy + y2
= (x − y)2
+ xy > 1 và x + y > 1.
Do x2
− xy + y2
và x + y cùng chia het x3
+ y3
nên suy ra chúng là b®i so của
p, do đó p chia het (x + y)2
− (x2
− xy + y2
) = 3xy.
2
40
Viết đề tài giá sinh viên – ZALO:0973.287.149-TEAMLUANVAN.COM
3 2
2x + 2y+
3= 5,
3 2
2x 2y
+3 = 5,
Suy ra p chia het m®t trong ít nhat hai so x, y vì p không chia het 3.
Hơn nǎa, p không the chia het chỉ m®t trong hai so x và y vì ta có p| (x + y).
V y nên p chia het x và p chia het y.
Tà đó, suy ra n > 3, chia hai ve của pn
= x3
+ y3
cho p3
, ta có
pn−1
=
x
3
y
3
Đieu này mâu thuan với cách chon n bé nhat như đã nói ở trên.
2.7 M t so phương pháp khác
Tiep theo, xét m®t so bài toán ve phương trình nghi m nguyên có trong các kỳ
Olympic, thi hoc sinh giỏi.
Bài toán 2.39 (Rumania MO, xem [4]). Tìm tat cả các nghi m nguyên của phương
trình sau
x6
+ 3x3
+ 1 = y4
.
Lài giai. Phương trình đã cho tương đương với
(x3
+ 1)2
+ (x3
+ 1) = y4
+ 1,
hay
Do đó
(2x3
+ 3)2
− 4y4
= 5.
(2x3
+ 3 + 2y2
).(2x3
+ 3 − 2y2
) = 5.
Ta thu được các h phương trình sau
2x3
− 2y2
+ 3 = 1,
2x3
− 2y2
+ 3 = 5,
2x3
+ 2y2
+ 3 = 1,
2x3
− 2y2
+ 3 = −1,
2x3
+ 2y2
+ 3 = −5,
— −
2x3
+ 2y2
+ 3 = −1.
Giải các h trên ta thu được nghi m của phương trình ban đau là (0, 1), (0, −1).
Bài toán 2.40 (16th USA MO, xem [5]). Tìm tat cả các so nguyên x, y khác 0
của phương trình sau
(x2
+ y)(x + y2
) = (x − y)3
.
p
p
+ .
41
Viết đề tài giá sinh viên – ZALO:0973.287.149-TEAMLUANVAN.COM
Lài giai. Ta viet phương trình đã cho dưới dạng phương trình b c hai ȁn y
2y2
+ (x2
− 3x)y + 3x2
+ x = 0.
Ta có
∆ = (x2
− 3x)2
− 8(3x+
x) = x(x + 1)2
(x − 8).
Phương trình trên có nghi m nguyên khi và chỉ khi x(x + 1)2
(x − 8) là so chính
phương.
Đ t
hay
tương đương với
x(x − 8) = z2
,
(x − 4)2
− z2
= 16,
(x − z − 4)(x + z − 4) = 16.
De dàng tìm được x ∈ {−1, 8, 9}. Do đó, nghi m của phương trình ban đau là
(−1, −1), (8, −10), (9, −6), (9, −21).
Bài toán 2.41 (Armenia MO, xem [4]). Tìm tat cả các nghi m nguyên (x, y) của
phương trình sau
Lài giai. Ta có
(x + 1)4
− (x − 1)4
= y3
.
(x + 1)4
− (x − 1)4
= 8x3
+ 8x.
Giả sả (x, y) là m®t nghi m của phương trình và x ≥ 1. Khi đó
(2x)3
< (x + 1)4
− (x − 1)4
< (2x + 1)3
.
Do đó
2x < y < 2x + 1, (vô lí) .
Như v y, neu (x, y) là nghi m của phương trình thì x phải là so nguyên không
dương.
Ta thay rang, neu (x, y) là nghi m của phương trình thì (−x, −y) cũng là
nghi m. Do đó, −x cũng phải là so nguyên không dương. V y chỉ có the x = 0.
V y nên phương trình đã cho có nghi m duy nhat là (0, 0).
Bài toán 2.42 (29th IMO). Cho a, b là các so nguyên dương thỏa mãn đieu ki n
ab + 1|a2 + b2
. Cháng tỏ rang
a2
+ b2
là m®t so chính phương.
ab + 1
42
Viết đề tài giá sinh viên – ZALO:0973.287.149-TEAMLUANVAN.COM
Lài giai. Goi (a, b) là c p so nguyên dương thỏa mãn đieu ki n bài toán thì (a, b)
là nghi m của phương trình
a2
− kab + b2
= k.
De thay a = 0 ho c b = 0 thì k là so chính phương.
Neu a 0 và b 0 thì a và b cùng dau.
Th t v y, neu ab < 0 thì a2
− kab + b2
> k.
Giả sả a > 0, b > 0 và k > 0. Neu a = b thì (2 − k)a2
= k > 0. Suy ra k = 1.
Neu a > b > 0. Goi b là so nguyên dương nhỏ nhat sao cho (a, b) là nghi m của
phương trình. Rõ ràng (b, kb − a) cũng là nghi m. Theo trên, neu kb = a thì k là
so chính phương.
M t khác kb − a > 0 (vì (kb − a) và b cùng dau) nên kb − a < b.
Th t v y,
kb − b < b ⇔ k <
a + b
b
⇔
a2
+ b2
<
1 + ab
a
+ 1.
b
Ta cháng minh bat đȁng thác cuoi cùng luôn đúng.
Th t v y,
a2
+ b2
1 + ab
a2
+ ab
< <
1 + ab
a2
+ ab
=
ab
a
+ 1.
b
Như v y, ta đã cháng minh được (b, kb − a) là nghi m của phương trình ban
đau nhưng kb − a < b. Đieu này mâu thuan với giả sả b là so nguyên dương nhỏ
nhat sao cho (a, b) là nghi m. V y k là so chính phương.
Bài toán 2.43 (Olympic Toán Hà N®i mở r®ng (HOMC), 2009). Cháng minh
phương trình
(x + 1)2
+ (x + 2)2
+ · · · + (x + 99)2
= yz
không có nghi m nguyên (x, y, z) với z > 1.
Lài giai. Ta có
tương đương với
yz
= (x + 1)2
+ (x + 2)2
+ · · · + (x + 99)2
,
Do đó
yz
= 99x2
+ 2(1 + 2 + · · · + 99)x + (12
+ 22
+ · · · + 992
).
yz
= 99x2
+
2.99.100
.x +
99.100.199 = 33(3x2
+ 300x + 50.199).
2 6
Vì ve phải chia het cho 3 nên 3|y. Suy ra, 32
|yz
. Nhưng ve phải không chia het
cho 9. V y nên phương trình đã cho không có nghi m nguyên với z > 1.
43
Viết đề tài giá sinh viên – ZALO:0973.287.149-TEAMLUANVAN.COM
2
√ √
Bài toán 2.44 (Bucharest MO, xem [4]). Tìm tat cả các c p so nguyên (x, y) của
phương trình
x3
− 4xy + y3
= −1.
Lài giai. Nhân cả hai ve của phương trình với 27 và công cả hai ve với 64, ta
được
Sả dụng ket quả
27x3
+ 27y3
+ 43
− 4.27xy = 37.
a3
+ b3
+ c3
− 3abc = (a + b + c)(a2
+ b2
+ c2
− ab − bc − ca),
phương trình tương đương với
(3x + 3y + 4)(9x2
+ 9y2
+ 16 − 9xy − 12x − 12y) = 37.
Vì 37 là so nguyên to và
(9x2
+ 9y2
+ 16 − 9xy − 12x − 12y) =
1
[(3x − 3y)2
+ (3x − 4)2
+ (3y − 4)2
] ≥ 0.
nên 3x + 3y + 4 = 1 ho c 3x + 3y + 4 = 37.
Neu 3x + 3y + 4 = 1 thì 9x2
+ 9y2
+ 16 − 9xy − 12x − 12y = 37.
Suy ra nghi m là (−1, 0) và (0, −1).
Neu 3x + 3y + 4 = 37 thì 9x2
+ 9y2
+ 16 − 9xy − 12x − 12y = 1. Suy ra
(3x − 3y)2
+ (3x − 4)2
+ (3y − 4)2
= 2.
Đieu này không the xảy ra vì x, y là các so nguyên khác nhau nên |3x − 3y| ≥ 3.
V y nghi m của phương trình ban đau là (−1, 0) và (0, −1).
Bài toán 2.45 (Hungary MO, xem [4]). Tìm tat cả các nghi m nguyên dương
(x, y, z) của phương trình
x3
+ 3y3
+ 9z3
− 3xyz = 0.
Lài giai. De thay (0, 0, 0) là m®t nghi m của phương trình. Giả sả (x1, y1, z1)
là m®t nghi m khác của phương trình trong đó x1 là so nguyên dương nhỏ nhat
trong các giá trị nh n được của x.
Neu m®t trong các so x1, y1, z1 bang 0. Do 3
3 và 3
9 là các so vô tỉ nên hai so
còn lại cũng bang 0. Ta có the giả sả x1, y1, z1 > 0.
Rõ ràng 3|x1. Đ t x1 = 3x2, với x1 nguyên. Thay vào phương trình roi chia cả
hai ve cho 3 ta có
9x2
3
+ y1
3
+ 3z1
3
− 3x2y1z1 = 0.
44
Viết đề tài giá sinh viên – ZALO:0973.287.149-TEAMLUANVAN.COM
Suy ra 3|y‘1. Đ t y1 = 3y2, với y2 nguyên. Thay vào phương trình trên roi chia cả
hai ve cho 3 ta có
3x2
3
+ 9y2
3
+ z1
3
− 3x2y2z1 = 0.
Như v y 3|z1. Đ t z1 = 3z2, với z2 nguyên. Thay vào phương trình trên roi chia
cả hai ve cho 3 ta có
x2
3
+ 3y2
3
+ 9z2
3
− 3x2y2z2 = 0.
Do đó (x2, y2, z2) cũng là nghi m của phương trình ban đau.
M t khác, x1 = 3x2, tác là x1 > x2. Đieu này trái với giả thiet x1 là so nguyên
dương nhỏ nhat trong các giá trị nh n được của x.
V y nên phương trình đã cho không có nghi m nguyên dương.
Bài toán 2.46 (5th USA MO, xem [6]). Tìm tat cả các nghi m nguyên (a, b, c)
của phương trình
a2
+ b2
+ c2
= a2
b2
.
Lài giai. Ta sě cháng minh a, b, c đeu là các so chȁn.
Ta có the giả sả a, b, c là các so nguyên không âm. Ta có
(2n)2
≡ 0 (mod 4)và (2n + 1)2
≡ 1 (mod 4).
Trường hợp 1. Khi a, b, c đeu là so lẻ.
a2
+ b2
+ c2
≡ 3 (mod 4)nhưnga2
b2
≡ 1 (mod 4).
Trường hợp 2. Hai trong ba so a, b, c là so lẻ.
a2
+ b2
+ c2
≡ 2 (mod 4)nhưnga2
b2
≡ 0ho c1 (mod 4).
Trường hợp 3. Hai trong ba so a, b, c là so chȁn.
a2
+ b2
+ c2
≡ 1 (mod 4)nhưnga2
b2
≡ 0 (mod 4).
V y cả ba so a, b, c đeu chȁn. Đ t a = 2a1, b = 2b1, c = 2c1, với a1, b1, c1 là các so
nguyên. Thay vào phương trình roi chia cả hai ve cho 4 ta có
a1
2
+ b1
2
+ c1
2
= 4a1
2
b1
2
.
L p lu n tương tự, ta có 4a1
2
b1
2
≡ 0 (mod 4) và moi so a1
2
, b1
2
, c1
2
≡ 0 ho c 1
(mod 4). Suy ra
a1
2
≡ b1
2
≡ c1
2
≡ 0 (mod 4),
áng với a1, b1, c1 là các so chȁn.
45
Viết đề tài giá sinh viên – ZALO:0973.287.149-TEAMLUANVAN.COM
Đ t a1 = 2a2, b1 = 2b2, c1 = 2c2, với a2, b2, c2 là các so nguyên. Thay vào
phương trình roi chia cả hai ve cho 4 ta có
a2
2
+ b2
2
+ c2
2
= 16a2
2
b2
2
.
Cá tiep tục như v y, dan đen a, b, c đeu chia het cho 2k
, với k là so tự nhiên tùy
ý. Đieu này chỉ xảy ra khi a = b = c = 0.
Đó chính là nghi m nguyên duy nhat của phương trình đã cho.
Bài toán 2.47 (8th USA MO). Tìm tat cả các nghi m nguyên không âm (x1, x2, . . . , x14)
(không ke các hoán vị của nó) của phương trình
x1
4
+ x2
4
+ · · · + x14
4
= 15999.
Lài giai.
- Với n là so chȁn, đ t n = 2k, với k nguyên thì n4
= 16k4
≡ 0 (mod 16).
- Với n là so lẻ thì
n4
− 1 = (n − 1) (n + 1) n2
+ 1 .
Vì (n − 1), (n + 1) là hai so chȁn liên tiep và n2
+ 1 cũng là so chȁn nên
n4
− 1 = (n − 1) . (n + 1) . n2
+ 1 ≡ 0 (mod 16).
Do đó
ho c
Suy ra
nên
M t khác, ta có
n4
≡ 0 (mod 16)
n4
≡ 1 (mod 16).
x1
4
+ x2
4
+ · · · + x144
≡ r (mod 16)
0 ≤ r ≤ 14.
1599 = 16000 − 1 ≡ 15 (mod 16).
V y nên phương trình đã cho không có nghi m.
Bài toán 2.48 (37th IMO). Tìm tat cả các nghi m nguyên dương (a, b) của
phương trình
ab2
= ba
.
Lài giai. Ta sě cháng minh phương trình chỉ có các nghi m là (1, 1), (16, 2),
(27, 3).
46
Viết đề tài giá sinh viên – ZALO:0973.287.149-TEAMLUANVAN.COM
Goi (a, b) là m®t nghi m của phương trình. Đ t (a, b) = d thì a = du, b = dv,
trong đó (u, v) = 1.
Phương trình đã cho tương đương với
(du)dv
2
= (dv)u
.
Ta xét các trường hợp sau.
Trường hợp 1. Neu dv2
= u thì u = v. M t khác (u, v) = 1 nên u = v = 1, d = 1.
Do đó nghi m của phương trình là
(a, b) = (1, 1).
Trường hợp 2. Neu dv2
> u thì ta viet được phương trình dưới dạng
ddv2
−u
udv2
= vu
.
Rõ ràng
udv2
|vu
.
Vì (u, v) = 1 nên u = 1. Khi đó, phương trình trở thành
ddv2
−1
= v.
Neu d = 1 thì v = 1. Đieu này trái với giả thiet dv2
> u.
Neu d ≥ 2 thì
ddv2−1
geq22v2−1
≥ 22v−1
> v, v = 1, 2, 3, . . .
Do đó, trong trường hợp này phương trình cũng không có nghi m.
Trường hợp 3. Neu dv2
< u thì d < u. Ta viet được phương trình dưới dạng
udv2
= du−dv2
vu
.
Rõ ràng
vu
|udv2
.
Vì (u, v) = 1 nên v = 1. Khi đó, phương trình trở thành
ud
= du−d
.
47
Viết đề tài giá sinh viên – ZALO:0973.287.149-TEAMLUANVAN.COM
2018
Chương 3. Các dạng toán liên quan đen
phương trình và h phương trình
Diophant
Trong nhieu bài toán thực te, người ta can tìm các nghi m nguyên dương (chá
không đơn thuan là nghi m nguyên) của h phương trình đại so dạng Ax = b,
trong đó A ∈ Rm×n
là ma tr n nguyên và rank(A) = m, b ∈ Rm
là véctơ nguyên.
Khi đó, người ta thường sả dụng các phương pháp như đã trình bày trong
Chương 2 đe tìm nghi m nguyên dương của h phương trình tương áng.
3.1 M t so dạng toán ve đa thfíc nguyên
Bài toán 3.1 (Thi chon ĐTQG, Thanh Hóa 2017). Tìm tat cả các đa thác P (x)
với h so là các so nguyên thỏa mãn đieu ki n n(n−1)
− 1 chia het cho P (n), với
moi so nguyên dương n.
Lài giai. Giả sả P (x) là m®t đa thác thỏa mãn bài toán. Xét các trường hợp:
- Neu deg P (x) = 0 thì P (x) ≡ a0, a0 ∈ Z.
M t khác
V y P (x) ≡ ±1
2(2−1)2018
− 1 .P (2) = a0 ⇔ a0 = ±1.
- Neu deg P (x) ≥ 1. Đ t
P (x) = amxm
+ am−1xm−1
+ · · · + a1x + a0
với m ∈ N∗
, ai ∈ Z, ∀i = 0; m.
- Neu am > 0 thì ton tại so nguyên dương N sao cho P (x) > 0, ∀x > N. Với
n ∈ N∗
, n > N, xét so nguyên to p là ước của P (n).
(1) .
.p
n(n−1)2018
− 1
Tà giả thiet suy ra
48
Viết đề tài giá sinh viên – ZALO:0973.287.149-TEAMLUANVAN.COM
p
M t khác
P (n + p) = am(n + p)m
+ am−1(n + p)m−1
+ · · · + a1 (n + p) + a0
= P (n) + pQ (n) ,
suy ra P (n + p) .p (với Q (n ) ∈ Z).
Vì (n + p)(n+p−1)
2018
− 1 .P (n + p) nên
(n + p)(n+p−1)
2018
≡ 1 (mod p)
nên n(n+p−1)
2018
≡ 1 (mod p), do đó (n, p) = 1. Theo Định lý Euler, ta có np−1
≡ 1
(mod p).
Khi đó, tà (n + p − 1)2018
= n2018
+ (p − 1) A (với A ∈ N∗
), suy ra
n(n+p−1)2018
≡ nn2018
+(p−1)A
≡ nn2018
np−1 A
≡ nn2018
(mod p),
do n(n+p−1)
2018
≡ 1 (mod p) suy ra nn2018
≡ 1 (mod p) hay
nn2018
− 1 .p. (2)
Tà (1) , (2) và sả dụng tính chat (am
− 1; an
− 1) = a(m;n)
− 1, suy ra
nn2018
− 1; n(n−1)
2018
− 1 .p
nên
Do đó
n(n2018
;(n−1)2018
) − 1 .p.
(do n2018
; (n − 1)2018
= 1)
(n − 1) .p, (3)
Xét so nguyên to q, q > N thỏa mãn đieu ki n P (q + 1) .p. Theo (3) ta có
(q + 1 − 1) .p nên q.p. Vì p, q là các so nguyên to nên q = p. Tà đó, suy ra
P (q + 1) = pkp
với moi so nguyên to p > N, kp là so nguyên dương phụ thu®c p.
Goi vp (n) là so mũ đúng của ước nguyên to p trong n, nghĩa là n.pk
nhưng
n / .
p
k+1.
Sả dụng tính chat, áng với x, y là các so nguyên, n nguyên dương và p là so
nguyên to lẻ sao cho p |(x − y) và x, y không chia het cho p thì
vp (xn
− yn
) = vp (x − y) + vp (n) .
Suy ra
v (p + 1)p
2018
− 1 = v ((p + 1) − 1) + vp p2018
= 2019
p
49
Viết đề tài giá sinh viên – ZALO:0973.287.149-TEAMLUANVAN.COM
Σ Y
Σ Y Σ
= (−1)
C
2016
k
2016
k
k+1
2016 2017 2016
nên kp ≤ 2019 với moi so nguyên to p > N.
Do dãy (kp) có vô hạn phan tả (vì có vô hạn so nguyên to p > N) nên ton tại
m®t dãy con thỏa mãn đieu ki n P (q + 1) = pk
với vô so so nguyên to p. Suy ra
P (x) = (x − 1)k
với k ∈ N∗
, 1 ≤ k ≤ 2019.
- Neu am < 0, bang cách đ t Q (x) = −P (x) và làm tương tự ta có Q (x) =
(x − 1)k
, suy ra
P (x) = −(x − 1)k
với k ∈ N∗, 1 ≤ k ≤ 2019.
Thả lại, ta thay các đa thác
P (x) = (x − 1)k
và
P (x) = −(x − 1)k
với k ∈ N∗
, 1 ≤ k ≤ 2019 không thỏa mãn yêu cau bài toán tại n = 1.
V y tat cả đa thác can tìm là P (x) ≡ ±1.
Bài toán 3.2 (Thi chon ĐTQG, Đà Nȁng 2017). Tìm so nguyên dương n nhỏ
nhat sao cho ton tại đa thác f (x) b c n có h so nguyên thỏa mãn đieu ki nf
(0) = 0, f (1) = 1 và với moi m ∈ N∗
, f (m) (f (m) − 1) là b®i của 2017.
Lài giai. Ta sě cháng minh n ≥ 2016 thỏa mãn. Giả sả 1 ≤ n ≤ 2015 và ton tại
đa thác f (x) thỏa mãn bài toán. Áp dụng công thác n®i suy Lagrange
Suy ra
2015
f (x) = f (k)
k=0
2015
i=0,i=
/
x − i
.
k
k − i
2015
f (2016) = f (k)
k=0
2015
i=0,i=
/ k
2016 − i
k − i
2015
k+1 k
2016
k=0
f (k).
Ta cháng minh Ck
≡ (−1) (mod 2017) , k = 0, 1, 2, . . . , 2015 bang phương
pháp quy nạp.
Với k = 0 thì C0
≡ 1 (mod 2017). Giả sả đúng đen k, tác là
thì
k
2016 ≡ (−1) (mod 2017)
Ck+1
= Ck+1
− Ck
≡ (−1) (mod 2017) .
C
50
Viết đề tài giá sinh viên – ZALO:0973.287.149-TEAMLUANVAN.COM
Σ
Σ Σ
Σ
Σ
V y f (k) = −
2015
k=0
f (k) (mod 2017) nên
Nhưng
2016
k=0
f (k) = 1 +
2016
k=2
2016
f (k) ≡ 0 (mod 2017) .
k=0
f (k) và
Do đó
2016
k=0
f (k) ≡ 0; 1 (mod 2017) ; k = 2, . . . , 2016.
f (k) / ≡0 (mod 2017) (mâu thuan với nh n xét trên).
V y n ≥ 2016.
Với n = 2016, xét đa thác f (x) = x2016
thỏa mãn yêu cau bài toán. Do đó n
nhỏ mhat bang 2016.
Bài toán 3.3 (Thailand MO 2014). Tìm tat cả các đa thác P (x) với h so là các
so nguyên thỏa mãn đieu ki n
P (n) |2557n
+ 213.2014, ∀n ∈ N∗.
Lài giai. De thay P (n) = 1, ∀n ∈ N∗
và P (n) = −1, ∀n ∈ N∗
là nhǎng đa
thác thỏa mãn yêu cau bài toán.
Giả sả đa thác P (x) thỏa mãn bài toán và ton tại n0 ∈ N∗
: |P (n0)| ≥ 2.
Goi p là m®t ước nguyên to của P (n0).
Ta có P (n0) |2557n0
+ 213.2014 và P (n0 + p) |2557n0+p
+ 213.2014.
Do đó p |(P (n0 + p) − P (n0)) |2557n0
(2557p
− 1).
M t khác, vì p |P (n0) |2557n0
+ 213.2014 nên
p / ∈ {2, 3, 19, 53, 71, 2557} .
Do đó p |(2557p
− 1).
M t khác theo Định lý Fermat p |(2557p
− 2557) thì p |2556 suy ra p ∈ {2, 3, 71},
vô lý.
V y chỉ có P (n) = 1, ∀n ∈ N∗
và P (n) = −1, ∀n ∈ N∗
là hai đa thác thỏa
mãn yêu cau bài toán.
3.2 M t so dạng toán lư ng giác liên quan
Trong phan này ta xét m®t so dạng toán lượng giác liên quan can đen ky thu t
giải phương trình và h Diophant.
51
Viết đề tài giá sinh viên – ZALO:0973.287.149-TEAMLUANVAN.COM
2
2x + y = 12k + 1
x + y = 6l + 1
Bài toán 3.4. Xác định so nghi m nguyên của phương trình lượng giác
cos
π
x = 0
trong khoảng (−2015; 2015).
Lài giai. Phương trình đã cho tương đương với
hay
π π
2
x =
2
+ kπ (k ∈ Z)
x = 2k + 1.
Do x ∈ (−2015; 2015) , nên ta có
−2015 < 2k + 1 < 2015,
tác là
−1008 < k < 1007.
V y trong khoảng (−1008; 1007) có 2016 so nguyên.
V y nên phương trình đã cho có 2016 nghi m nguyên x trong khoảng (−2015; 2015).
Bài toán 3.5. Tìm các nghi m nguyên (x, y) của h phương trình lượng giác
sin
π(2x + y)
=
1
6 2
π(x + y) 1
cos
3
=
2
Biet rang x, y thu®c khoảng (−6; 10).
Lài giai. H phương trình đã cho tương đương với
2x + y = 12m + 5
(k, m, l, h ∈ Z).
x + y = 6h − 1
Trường hợp 1. Xét h phương trình
2x + y = 12k + 1
x + y = 6l + 1.
52
Viết đề tài giá sinh viên – ZALO:0973.287.149-TEAMLUANVAN.COM
H này tương đương với
Ket hợp với đieu ki n
x = 12k − 6l
y = 12l − 12k + 1.
x, y ∈ (−6; 10)
ta thu được h bat phương trình
−6 < 12k − 6l < 10
−7 < 12l − 12k < 9.
Tà đó
suy ra
−19 < 12k < 29,
−1 ≤ k ≤ 2.
- Với k = −1, thay vào h bat phương trình trên ta không thu được giá trị
nguyên nào của l.
- Với k = 0, thay vào h bat phương trình trên ta thu được l = 0. Do đó
(x, y) = (0, 1).
- Với k = 1, thay vào h bat phương trình trên ta thu được l = 1. Do đó
(x, y) = (6, 1).’
- Với k = 2, thay vào h bat phương trình trên ta không thu được giá trị nguyên
nào của l.
Trường hợp 2. Xét h phương trình
2x + y = 12k + 1
H này tương đương với
x + y = 6h − 1.
Ket hợp với đieu ki n
x = 12k − 6h + 2
y = 12h − 12k − 3.
x, y ∈ (−6; 10)
ta thu được h bat phương trình
−8 < 12k − 6h < 8
−3 < 12h − 12k < 13.
53
Viết đề tài giá sinh viên – ZALO:0973.287.149-TEAMLUANVAN.COM
Tà đó
suy ra
−19 < 12k < 29,
−1 ≤ k ≤ 2.
- Với k = −1, thay vào h bat phương trình trên ta thu được h = −1. Do đó
(x, y) = (−4, −3).
- Với k = 0, thay vào h bat phương trình trên ta thu được h = 0, h = 1. Do đó
(x, y) = (2, −3), (x, y) = (−4, 9).
- Với k = 1, thay vào h bat phương trình trên ta thu được h = 1, h = 2. Do
đó (x, y) = (8, −3), (x, y) = (2, 9).
- Với k = 2, thay vào h bat phương trình trên, ta thu được h = 3. Do đó
(x, y) = (8, 9).
Trường hợp 3. Xét h phương trình
2x + y = 12m + 5
H này tương đương với
x + y = 6l + 1.
Ket hợp với đieu ki n
x = 12m − 6l + 4
y = 12l − 12m − 3.
x, y ∈ (−6; 10)
ta thu được h bat phương trình
−10 < 12m − 6l < 6
−3 < 12l − 12m < 13.
Tà đó
suy ra
−23 < 12m < 25,
−1 ≤ m ≤ 2.
- Với m = −1, thay vào h bat phương trình trên ta thu được l = −1. Do đó
(x, y) = (−2, −3).
- Với m = 0, thay vào h bat phương trình trên ta thu được l = 0, l = 1. Do
đó (x, y) = (4, −3), (x, y) = (−2, 9).
54
Viết đề tài giá sinh viên – ZALO:0973.287.149-TEAMLUANVAN.COM
- Với m = 1, thay vào h bat phương trình trên, ta thu được l = 2. Do đó
(x, y) = (4, 9).
- Với m = 2, thay vào h bat phương trình trên, ta không thu được giá trị
nguyên nào của l.
Trường hợp 4. Xét h phương trình
2x + y = 12m + 5
H này tương đương với
x + y = 6l − 1.
Ket hợp với đieu ki n
x = 12m − 6h + 6
y = 12h − 12m − 7.
x, y ∈ (−6; 10)
ta thu được h bat phương trình
−12 < 12m − 6h < 4
1 < 12h − 12m < 17.
Tà đó
suy ra
−23 < 12m < 25,
−1 ≤ m ≤ 2.
- Với m = −1, thay vào h bat phương trình trên, ta không thu được giá trị
nguyên nào của h.
- Với m = 0, thay vào h bat phương trình trên, ta thu được h = 1. Do đó
(x, y) = (0, 5).
- Với m = 1, thay vào h bat phương trình trên, ta thu được h = 2. Do đó
(x, y) = (6, 5).
- Với m = 2, thay vào h bat phương trình trên ta không thu được giá trị
nguyên nào của h.
V y h phương trình đã cho ban đau có 14 nghi m nguyên (x, y) bao gom
(0, 5) , (6, 5) , (−2, −3) , (4, −3) , (−2, 9) , (4, 9) , (−4, −3) ,
(2, −3) , (−4, 9) , (8, −3) , (2, 9) , (8, 9) , (0, 1) , (6, 1)
55
Viết đề tài giá sinh viên – ZALO:0973.287.149-TEAMLUANVAN.COM
sin 2015x >
1
6 6
3
3
445 3 445 3
4
hay
với x, y thu®c khoảng (−6; 10).
Tóm lại, đe khảo sát phương trình, h phương trình, bat phương trình, h bat
phương trình lượng giác có đieu ki n, ta thực hi n các bước sau:
Bước 1. Tìm nghi m tőng quát (x1, x2, . . . , xn) của phương trình (h phương
trình, bat phương trình, h bat phương trình) lượng giác.
Bước 2. Tà đieu ki n của phương trình (h phương trình, bat phương trình, h
bat phương trình) lượng giác, ta hạn che đieu ki n của các tham so trong nghi m
tőng quát (x1, x2, . . . , xn).
Bài toán 3.6. Giải h bat phương trình
2
1
cos 445x ≤
2
.
Lài giai. H bat phương trình đã cho tương đương với
π 5π
+ k2π < 2015x < + k2π
π 4π
+ m2π ≤ 445x ≤ + m2π
x =
π 1
+ 2k < x <
π 5
+ 2k = x
2015 6 2015 6 2
x =
π 1
+ 2m ≤ x ≤
π 4
+ 2m = x .
- Trường hợp 1: x ∈ [x3; x2) neu x1 < x3 < x2 ≤ x4.
+ x1 < x3 tương đương với
do đó
3
a = 403m − 89k > −59
4
a ≥ −59.
+ x3 < x2 tương đương với
1
a = 403m − 89k < −30
12
1
(k, m ∈ Z)
3
56
Viết đề tài giá sinh viên – ZALO:0973.287.149-TEAMLUANVAN.COM
1
445 3 2015 6
do đó
+ x2 ≤ x4 tương đương với
a ≤ −31.
7
do đó
a = 403m − 89k ≥ −231
12
a ≥ −231.
V y nên x ∈ [x3; x2) neu
403m − 89k = a
trong đó a ∈ Z, −59 ≤ a ≤ −31.
Ta có
nên
403
89
C =
p4
4
q4
= [4; 1, 1, 8, 2, 2]
163
= [4; 1, 1, 8, 2] =
36
Khi đó, phương trình
có m®t nghi m riêng là
Suy ra phương trình
p4 = 163, q4 = 36.
403m − 89k = 1
(m1; k1) = (36; 163) .
403m − 89k = a
trong đó a ∈ Z, −59 ≤ a ≤ −31 có nghi m tőng quát là
m = 36a + 89t
k = 163a + 403t
(t ∈ Z) .
S =
π 1
+ 72a + 178t ;
π 5
+ 326a + 806t ,
trong đó a, t ∈ Z, −59 ≤ a ≤ −31.
- Trường hợp 2: x ∈ (x3; x4] neu x1 ≤ x3 và x4 < x2.
+ x1 ≤ x3 tương đương với
3
a = 403m − 89k ≥ −59
4
V y trong trường hợp này, ta có t p nghi m của h bat phương trình đã cho là
57
Viết đề tài giá sinh viên – ZALO:0973.287.149-TEAMLUANVAN.COM
2
2015 6 2015 6
sin(2x + y) =
1
1
do đó
+ x4 < x2 tương đương với
a ≥ −59.
7
Do đó
a = 403m − 89k < −231
12
.
a ≤ −232.
Như v y trường hợp 2 không xảy ra.
- Trường hợp 3: x ∈ (x1; x2) neu x3 ≤ x1 và x2 ≤ x4.
+ x3 ≤ x1 tương đương với
Do đó
3
a = 403m − 89k ≤ −59
4
.
a ≤ −60.
+ x2 ≤ x4, theo ket quả của trường hợp 1 thì tương đương với
a ≥ −231.
V y trong trường hợp này, ta có t p nghi m của h bat phương trình đã cho là
S =
π 1
+ 326b + 806l ;
π 5
+ 326b + 806l ,
trong đó b, l ∈ Z, −231 ≤ b ≤ −60.
Ket lu n: T p nghi m của h bat phương trình ban đau là
S = S1 ∪ S2.
Bài toán 3.7. Giải h phương trình
2
với đieu ki n x − y ≥ 10π.
cos(x + y) =
2
Lài giai. H phương trình đã cho tương đương với
58
Viết đề tài giá sinh viên – ZALO:0973.287.149-TEAMLUANVAN.COM
−
2
2x + y =
π
+ k2π
2x + y =
5π
+ m2π
x + y =
π
+ l2π
2x + y =
π
+ k2π
x = −
π
+ (a + t)2π
π
6
6
(k, m, l, h ∈ Z).
3
x + y =
π
+ h2π
3
- Trường hợp 1: Xét h phương trình
6
có nghi m là x + y =
3
+ l2π
x = −
π
+ (k − l)2π
6
π
y =
2
+ (2l − k)2π.
Ket hợp đieu ki n x − y ≥ 10π ta thu được bat phương trình
16
2k − 3l ≥
3
.
Bat phương trình này tương đương với
trong đó a ∈ Z, a ≥ 6.
Tà đó, ta có
2k − 3l = a
V y nên
k = 2a + 3t
l = a + 2t
(t ∈ Z).
6
y =
π
+ t2π
59
Viết đề tài giá sinh viên – ZALO:0973.287.149-TEAMLUANVAN.COM
3
2x + y =
π
+ k2π
x =
π
+ (b + t)2π
2x + y =
5π
+ m2π
π
trong đó a, t ∈ Z, a ≥ 6.
- Trường hợp 2: Xét h phương trình
6
có nghi m là
x + y = −
3
+ h2π
x =
π
+ (k − h)2π
2
5π
y = −
6
+ (2h − k)2π.
Ket hợp đieu ki n x − y ≥ 10π, ta thu được bat phương trình
13
2k − 3h ≥
3
.
Bat phương trình này tương đương với
trong đó b ∈ Z, b ≥ 5.
Tà đó, ta có
2k − 3h = b,
V y nên
k = 2b + 3t
h = b + 2t
(t ∈ Z).
2
trong đó b, t ∈ Z, b ≥ 5.
5π
y = −
6
+ t2π
- Trường hợp 3: Xét h phương trình
6
x + y =
π
+ l2π
60
Viết đề tài giá sinh viên – ZALO:0973.287.149-TEAMLUANVAN.COM
x =
π
+ (c + t)2π
2x + y =
5π
+ m2π
π
π
có nghi m là
x =
π
+ (m − l)2π
2
π
y = −
6
+ (2l − m)2π.
Ket hợp đieu ki n x − y ≥ 10π, ta thu được bat phương trình
14
2m − 3l ≥
3
.
Bat phương trình này tương đương với
trong đó c ∈ Z, c ≥ 5.
Tà đó, ta có
2m − 3l = c,
V y nên
m = 2c + 3t
l = c + 2t
(t ∈ Z).
2
trong đó c, t ∈ Z, c ≥ 5. y = −
6
+ t2π
- Trường hợp 4: Xét h phương trình
6
có nghi m là
x + y = −
3
+ h2π
x =
7π
+ (m − h)2π
6
3π
y = −
2
+ (2h − m)2π.
Ket hợp đieu ki n x − y ≥ 10π, ta thu được bat phương trình
11
2m − 3h ≥
3
.
61
Viết đề tài giá sinh viên – ZALO:0973.287.149-TEAMLUANVAN.COM
x =
7π
+ (d + t)2π
sin(2x + y) =
1
π
Bat phương trình này tương đương với
2m − 3h = d,
trong đó d ∈ Z, d ≥ 4.
Tà đó, ta có
V y nên
m = 2d + 3t
h = d + 2t
(t ∈ Z).
6
trong đó d, t ∈ Z, d ≥ 4.
3π
y = −
2
+ t2π
Tóm lại, ta có 4 ho nghi m (x; y) thỏa mãn giả thiet bài ra, bao gom
π
x = − + (a + t)2π x =
π
+ (b + t)2π
6
π
y = + t2π,
2
2
5π
y = −
6
+ t2π,
x =
π
+ (c + t)2π x =
7π
+ (d + t)2π
2
y = −
6
+ t2π,
6
3π
y = −
2
+ t2π,
trong đó a, b, c, d, t ∈ Z, a ≥ 6, b ≥ 5, c ≥ 5, d ≥ 4.
Bài toán 3.8. Giải h phương trình
2
cos(x + y) =
1
2
trên đoạn [−6π; 6π] và thỏa mãn đieu ki n x − y ≥ 10π.
Lài giai. H đã cho có 4 ho nghi m (x; y) thỏa mãn đieu ki n x − y ≥ 10π, bao
gom
62
Viết đề tài giá sinh viên – ZALO:0973.287.149-TEAMLUANVAN.COM
π
x = −
6
+ (a + t)2π
6 ≤ a ≤ 6 +
1
π
x = − + (a + t)2π x =
π
+ (b + t)2π
6
π
y = + t2π,
2
2
5π
y = −
6
+ t2π,
x = + (c + t)2π x = + (d + t)2π
π 7π
2 6
π
y = −
6
+ t2π,
3π
y = −
2
+ t2π
trong đó a, b, c, d, t ∈ Z, a ≥
π
6, b ≥ 5, c ≥ 5, d ≥ 4.
- Trường hợp 1:
y =
2
+ t2π
(a, t ∈ Z, a ≥ 6).
Với đieu ki n x, y ∈ [−6π; 6π] ta có
π
−6π ≤ −
6
+ (a + t)2π ≤ 6π
−6π ≤
π
+ t2π ≤ 6π
2
a, t ∈ Z, a ≥ 6.
H này tương đương với
1 1
12
− 3 ≤ a + t ≤
12
+ 3
1 1
suy ra
−
4
− 3 ≤ t ≤ −
4
+ 3
a, t ∈ Z, a ≥ 6
3
a ∈ Z
63
Viết đề tài giá sinh viên – ZALO:0973.287.149-TEAMLUANVAN.COM
2
−
4
− 3 ≤ b + t ≤ −
4
+ 3
- Trường hợp 2: (b, t ∈ Z, b ≥ 5).
nên a = 6.
Với a = 6, t = −3, ta thu được
(x; y) =
35π
; −
11π
.
6 2
x =
π
+ (b + t)2π
5π
y = −
6
+ t2π
Với đieu ki n x, y ∈ [−6π; 6π] , ta có
π
−6π ≤
2
+ (b + t)2π ≤ 6π
−6π ≤ −
5π
+ t2π ≤ 6π
6
H này tương đương với
b, t ∈ Z, b ≥ 5.
1 1
5 5
12
− 3 ≤ t ≤
12
+ 3
suy ra
b, t ∈ Z, b ≥ 5
5 ≤ b ≤ 5 +
1
suy ra b = 5.
Với b = 5, ta thu được
5
12
3
b ∈ Z,
1
— 3 ≤ t ≤ −
4
− 2
nên t ∈ ∅.
t ∈ Z
64
Viết đề tài giá sinh viên – ZALO:0973.287.149-TEAMLUANVAN.COM
2
−
4
− 3 ≤ c + t ≤ −
4
+ 3
V y trong trường hợp này ta không thu được nghi m (x; y) thỏa mãn yêu cau của
bài toán.
x =
π
+ (c + t)2π
π
y = −
6
+ t2π
Với đieu ki n x, y ∈ [−6π; 6π] ta có
π
−6π ≤
2
+ (c + t)2π ≤ 6π
−6π ≤ −
π
+ t2π ≤ 6π
6
c, t ∈ Z, c ≥ 5.
H này tương đương với
1 1
1 1
12
− 3 ≤ t ≤
12
+ 3
suy ra
c, t ∈ Z, c ≥ 5
5 ≤ c ≤ 5 +
2
Với c = 5 ta thu được
c ∈ Z
3
nên c = 5.
1
12
1
— 3 ≤ t ≤ −
4
− 2
nên t ∈ ∅.
t ∈ Z
V y trường hợp này ta không thu được nghi m (x; y) thỏa mãn yêu cau của bài
toán.
- Trường hợp 3: (c, t ∈ Z, c ≥ 5).
65
Viết đề tài giá sinh viên – ZALO:0973.287.149-TEAMLUANVAN.COM
d ∈ Z
6
6 2
6 2 6 2
- Trường hợp 4: (d, t ∈ Z, d ≥ 4).
x =
7π
+ (d + t)2π
3π
y = −
2
+ t2π
Với đieu ki n x, y ∈ [−6π; 6π] ta có
7π
−6π ≤
6
+ (d + t)2π ≤ 6π
−6π ≤ −
3π
+ t2π ≤ 6π
2
d, t ∈ Z, d ≥ 4.
H này tương đương với
7 7
−
12
− 3 ≤ d + t ≤ −
12
+ 3
3
− 3 ≤ t ≤
3
+ 3
4 4
suy ra
d, t ∈ Z, d ≥ 4
4 ≤ d ≤ 4 +
2
Với d = 4, t = −2, ta t
hu được
3
nên d = 4.
(x; y) =
31π
; −
11π
.
Tóm lại, bài toán đã cho có hai nghi m (x; y) bao gom
35π
; −
11π
,
31π
; −
11π
.
66
Viết đề tài giá sinh viên – ZALO:0973.287.149-TEAMLUANVAN.COM
3.3 M t so dạng toán thi Olympic liên quan
Bài toán 3.9 (Đe thi chon đ®i tuyen quoc gia dự thi IMO 2006). Hãy tìm tat cả
các c p so tự nhiên (n; k) với n là so nguyên không âm và k là so nguyên lớn hơn
1 sao cho so A = 172006n
+ 4.172n
+ 7.195n
có the phân tích được thành tích của k
so nguyên dương liên tiep.
Lài giai. Trước het, ta thay rang tích của 4 so tự nhiên liên tiep phải chia het
cho 8 vì trong 4 so đó có 1 so chia het cho 4 và m®t so chia 4 dư 2.
Tà A = 172006n
+ 4.172n
+ 7.195n
, suy ra
- Neu n là so chȁn, ta có
172006n
≡ 1 (mod 8);
4.172n
≡ 4.1 (mod 8);
195n
≡ 7.35n
≡ 7.310
≡ 7 (mod 8).
V y nên A ≡ 12 ≡ 4 (mod 8), tác là A không chia het cho 8.
- Neu n là so lẻ, cũng tương tự
172006n
≡ 1 (mod 8);
4.172n
≡ 4.1 (mod 8);
195n
≡ 7.35n
≡ 7.35
≡ 7.3 (mod 8).
Suy ra A ≡ 10 ≡ 2 (mod 8), tác là A cũng không chia het cho 8.
Tác là, trong moi trường hợp luôn có A không chia het cho 8. Suy ra neu k
thỏa mãn đe bài thì k < 4, suy ra k ∈ {2, 3}.
Xét tàng trường hợp cụ the.
- Neu k = 2 thì ton tại x tự nhiên sao cho A = x(x + 1).
+ Neu n = 0 thì A = 12, x = 3, thỏa mãn đe bài.
+ Neu n > 0 thì rõ ràng 171003n
> 4.172n
+ 7.195n
. Ta thay
A = x(x + 1) = 172006n
+ 4.172n
+ 7.195n
> 172006n
.
Suy ra x > 171003n
. Nhưng x(x + 1) > 172006n
+ 171003n
> A, mâu thuan.
Do đó, trong trường hợp này không có n thỏa mãn đe bài.
- Neu k = 3 thì ton tại x tự nhiên sao cho
A = x(x − 1)(x + 1), x ≥ 1.
De thay x phải là so chȁn (vì neu ngược lại thì A chia het cho 8, mâu thuan).
67
Viết đề tài giá sinh viên – ZALO:0973.287.149-TEAMLUANVAN.COM
n 2
n
n−1
n−1
n−1
2
n n n−1
Ta thay
A ≡ 12.(−1)n
≡ 2.(−1)n
(mod 5),
trong khi x(x − 1)(x + 1) = x(x2
− 1) ≡ {0; ±1} (mod 5), mâu thuan.
Do đó, trong trường hợp này không có n thỏa mãn đe bài.
V y tat cả các c p so thỏa mãn đe bài là (n; k) = 0; 2).
Bài toán 3.10 (Thi HSG Hà N®i, vòng 2, 2012). Cho dãy so (un) xác định bởi
a1 = 20; a2 = 30; an+2 = 3an+1 − an (n ∈ N∗). (3.1)
Tìm tat cả các so nguyên dương n sao cho 5an+1an + 1 là so chính phương.
Lài giai. Áp dụng ket quả (3.1) với dãy (an), ta được
Suy ra
an+1an−1 − a2
= a1a3 − a2
= 500.
Xét với n ≥ 4, ta có
a2
+ 500 = an+1an−1.
(an + an+1)2
= a2
+ 2anan+1 + a2
.
M t khác
n n+1
a2
= 9a2
− 6anan−1 + a2
.
Suy ra
n+1 n n−1
(an + an+1)2
= a2
+ 2anan+1 + 9a2
− 6anan−1 + a2
= 2anan+1 + 3an(3an − an−1) + a2
+ an(an − 3an−1)
= 5anan+1 + a2
— anan−2
Do đó
= 5anan+1 + a2
— (an−1 + 500) = 5anan+1 − 500.
(an + an+1)2
= 5anan+1 − 500 < 5anan+1 + 1.
Tà dãy (an) tăng và n ≥ 4, ta có
an + an+1 ≥ 180 + 470 = 650.
Suy ra
(an + an+1 + 1)2
= (an + an+1)2
+ 2(an + an+1) + 1
> (an + an+1)2
+ 501 = 5anan+1 + 1.
68
Viết đề tài giá sinh viên – ZALO:0973.287.149-TEAMLUANVAN.COM
··
·
V y (an + an+1)2
< 5anan+1 + 1 < (an + an+1 + 1)2
nên 5anan+1 + 1 không
chính phương.
Bang phép thả trực tiep với n = 1, 2, 3, ta thu được n = 3 là giá trị duy nhat
can tìm.
Bài toán 3.11 (VMO 1991 - Bảng B). Cho dãy so (an) xác định bởi
a1 = 1.2.3, u2 = 2.3.4, . . . , un = n(n + 1)(n + 2).
Đ t Sn = a1 + a2 + · · · + an. Cháng minh rang 4Sn + 1 là so chính phương.
Lài giai. Ta có
an = (n + 1) n2
+ 2n = (n + 1)
h
(n + 1)2
− 1
i
= (n + 1)3
− (n + 1).
Suy ra
Sn = a1 + a2 + · · · + an
= 23
+ 33
+ · · · + (n + 1)3
− [2 + 3 + · · · + (n + 1)]
= 13
+ 23
+ 33
+ · · · + (n + 1)3
− [1 + 2 + 3 + · · · + (n + 1)] .
Bang phương pháp quy nạp, ta cháng minh được
2 2
13
+ 23
+ 33
+ + (n + 1)3
=
(n + 1) (n + 2)
4
(n + 1)(n + 2)
Suy ra
1 + 2 + 3 + · · · + (n + 1) =
2
.
Tà đó, ta có
Sn =
n(n + 1)(n + 2)(n + 3)
.
4
4Sn + 1 = n(n + 1)(n + 2)(n + 3) + 1
= n2
+ 3n n2
+ 3n + 2 + 1
= n2
+ 3n
2
+ 2 n2
+ 3n + 1 = n2
+ 3n + 1
2
.
M t khác n ∈ N∗
nên n2
+ 3n + 1 là so nguyên dương.
V y 4Sn + 1 là so chính phương.
Nh n xét 3.8. Ta có the bien đői như sau
Sn = 1.2.3 + 2.3.4 + · · · + n.(n + 1).(n + 2),
69
Viết đề tài giá sinh viên – ZALO:0973.287.149-TEAMLUANVAN.COM
4
n
3
n
n
suy ra
4Sn + 1 = 1.2.3.4 + 2.3.4.(5 − 1) + · · · +
+ n.(n + 1).(n + 2)[(n + 3) − (n − 1)] + 1
= n.(n + 1).(n + 2).(n + 3) + 1.
Bài toán 3.12 (Romania TST, 2002). Cho dãy so (un) xác định bởi
u0 = 1, u1 = 1
un+2 = 14un+1 − un, n ≥ 1.
Cháng minh rang với moi so tự nhiên n thì 2un − 1 là m®t so chính phương.
Lài giai.
t1,2 = 7 ± 4
Xét phương trình đ c trưng tương áng t2
= 14t − 1 có hai nghi m
3.
Do đó, so hạng tőng quát của dãy là
u = a 2 +
√
3
2n
+ b 2 −
√
3
2n
.
Theo giả thiet u0 = u1 = 1, ta tìm được
1 1
Suy ra
a =
4 2 +
√ , b =
2 −
√
3
Do đó
u =
1
2 +
√
3
2n−1
+ 2 −
√
3
2n−1
.
√
2un − 1 =
3 + 1
2n−1
—
√
3 − 1
2n
2n−1 2
Xét hai dãy nguyên dương (An), (Bn) mà
√
3 ± 1
2n−1
= A
√
3 ± Bn.
Do đó
√
3 + 1
2n−1
−
√
3 − 1
2n−1
= 2B .
Tà đó ta có 2un − 1 =
Bn
2
2n−1
là m®t so chính phương.
V y ta có đieu phải cháng minh.
(
√
n
4
.
.
70
Viết đề tài giá sinh viên – ZALO:0973.287.149-TEAMLUANVAN.COM
u − 1
2
n
Bài toán 3.13 (Thi HSG thành pho Can Thơ, 2012 - 2013). Cho dãy so nguyên
(un) xác định bởi
u1 = 1; u2 = 2
un = 4un−1 − un−2, ∀n ≥ 2, n ∈ N.
a) Cháng minh rang un
2
+ u2
− 4unun−1 = −3 với n ≥ 2, n ∈ N.
2
b) Cháng minh n
3
n−1
là so chính phương với moi n nguyên dương.
Lài giai.
a) Xét phương trình đ c trưng tương áng λ2
− 4λ + 1 = 0 có hai nghi m
λ1 = 2 −
√
3; λ2 = 2 +
√
3.
Khi đó un = A.λn
+ B.λn
.
1 2
Với u1 = 1; u2 = 2 ta tìm được
A =
√
3 + 2
; B =
2 −
√
3
.
2 2
Suy ra
Khi đó
u =
1
2 −
√
3
n−1
+ 2 +
√
3
n−1
; n ≥ 1.
u2
+ u2
=
1
2 −
√
3
2(n−1)
+ 2 +
√
3
2(n−1)
+
V y
n n−1
4unun−1
4
+ 2 −
√
3
2(n−2)
+ 2 +
√
3
2(n−2)
+ 4
= 2 −
√
3
2n−3
+ 2 +
√
3
2n−3
+ 1.
= 2 −
√
3
n−1
+ 2 +
√
3
n−1
×
× 2 −
√
3
n−2
+ 2 +
√
3
n−2
= 2 −
√
3
2n−3
+ 2 +
√
3
2n−3
+ 4.
u2
+ u2
— 4unun−1 = −3.
n
u2
− 1
n−1
b) Cháng minh n
3
là so chính phương.
71
Viết đề tài giá sinh viên – ZALO:0973.287.149-TEAMLUANVAN.COM
n
n
.
.
n n
— − ≡ − ≡
(
n n−1 n
2u2 − u1 = 4 − 1 = 3.3 , ∀n ≥ 2
k
Tà câu a) ta có 4u2
+ u2
— 4unun−1 = 3u2
− 3. Suy ra
hay
(2un − un−1)2
= 3u2
− 3
u2
− 1
3
(2un − un−1)2
9
Ta sě cháng minh rang 2un − un−1.3, ∀n ≥ 2
2un−1 − un.3, ∀n ≥ 2.
Th t v y, với n = 2 thì
.
2u1 − u2 = 0.3, ∀n ≥ 2.
Giả sả 2uk − uk−1.3, ∀k ≥ 2
2uk−1 − uk.3, ∀k ≥ 2
. Suy ra
2uk+1 − uk = 2(4uk − uk−1) − uk = 6uk + uk − 2uk−1 . 3;
2uk − uk+1 = 2uk − (4uk − uk−1) = −2uk + uk−1 . 3.
Nói riêng, ta có 2un − un−1.3, ∀n ≥ 1.
Suy ra 2un − un−1 = 3k, k ∈ Z.
V y
u2
− 1
3
= k2
. Suy ra
u 2
− 1
3
là so chính phương.
Bài toán 3.14 (IMO 1964). a) Tìm tat cả các so nguyên dương n sao cho 2n
− 1
chia het cho 7.
b) Cháng minh rang không có so tự nhiên n nào đe 2n
+ 1 chia het cho 7.
Lài giai.
a) Vì n là so nguyên dương nên ta xét các trường hợp của n như sau:
• Với n = 3k, k ∈ Z ta có
2n
− 1 = 23
− 1 ≡ 1k
− 1 ≡ 0 (mod 7).
Do đó, với n là b®i của 3 thỏa yêu cau bài toán.
• Với n = 3k + r, k ∈ Z, r = 1, 2 ta có
2n
1 = 23k
.2r
1 2r
1
1 (mod 7), r = 1
3 (mod 7), r = 2.
Tà đó, suy ra n = 3k, k ∈ Z ta luôn có 2n
− 1 chia het cho 7.
b) Theo trên ta có 2n
≡ 1, 2, 4 (mod 7) với moi so tự nhiên n.
Do đó 2n
+ 1 /≡ (mod 7) với moi so nguyên dương n.
.
= .
Một số phương pháp giải các đề thi olympic Về phương trình diophant.docx
Một số phương pháp giải các đề thi olympic Về phương trình diophant.docx
Một số phương pháp giải các đề thi olympic Về phương trình diophant.docx
Một số phương pháp giải các đề thi olympic Về phương trình diophant.docx
Một số phương pháp giải các đề thi olympic Về phương trình diophant.docx
Một số phương pháp giải các đề thi olympic Về phương trình diophant.docx
Một số phương pháp giải các đề thi olympic Về phương trình diophant.docx

More Related Content

Similar to Một số phương pháp giải các đề thi olympic Về phương trình diophant.docx

Tich phan %28 nguyen duy khoi%29
Tich phan %28 nguyen duy khoi%29Tich phan %28 nguyen duy khoi%29
Tich phan %28 nguyen duy khoi%29
trongphuckhtn
 
Tich phan (nguyen duy khoi)
Tich phan (nguyen duy khoi)Tich phan (nguyen duy khoi)
Tich phan (nguyen duy khoi)
roggerbob
 

Similar to Một số phương pháp giải các đề thi olympic Về phương trình diophant.docx (20)

Bat Phương Trình Hàm Sinh B I Các Đại Lư Ng Trung Bình B C Tùy Ý Và Các Dạng ...
Bat Phương Trình Hàm Sinh B I Các Đại Lư Ng Trung Bình B C Tùy Ý Và Các Dạng ...Bat Phương Trình Hàm Sinh B I Các Đại Lư Ng Trung Bình B C Tùy Ý Và Các Dạng ...
Bat Phương Trình Hàm Sinh B I Các Đại Lư Ng Trung Bình B C Tùy Ý Và Các Dạng ...
 
Một Số Dạng Toán Cực Trị Trong Lîp Hàm Mũ Và Hàm Hyperbolic.docx
Một Số Dạng Toán Cực Trị Trong Lîp Hàm Mũ Và Hàm Hyperbolic.docxMột Số Dạng Toán Cực Trị Trong Lîp Hàm Mũ Và Hàm Hyperbolic.docx
Một Số Dạng Toán Cực Trị Trong Lîp Hàm Mũ Và Hàm Hyperbolic.docx
 
Ve Bat Đang Thức Ho¨ Lder Và Áp Dụng.docx
Ve  Bat  Đang  Thức  Ho¨ Lder  Và  Áp  Dụng.docxVe  Bat  Đang  Thức  Ho¨ Lder  Và  Áp  Dụng.docx
Ve Bat Đang Thức Ho¨ Lder Và Áp Dụng.docx
 
Tích Ngoài Của Véc Tơ Và Ứng Dụng.docx
Tích Ngoài Của Véc Tơ Và Ứng Dụng.docxTích Ngoài Của Véc Tơ Và Ứng Dụng.docx
Tích Ngoài Của Véc Tơ Và Ứng Dụng.docx
 
Tich phan %28 nguyen duy khoi%29
Tich phan %28 nguyen duy khoi%29Tich phan %28 nguyen duy khoi%29
Tich phan %28 nguyen duy khoi%29
 
Tich phan (nguyen duy khoi)
Tich phan (nguyen duy khoi)Tich phan (nguyen duy khoi)
Tich phan (nguyen duy khoi)
 
M T So Ứng Dụng Của Công Thức N I Suy Lagrange Và Hermite.docx
M T So Ứng Dụng Của Công Thức N I Suy Lagrange Và Hermite.docxM T So Ứng Dụng Của Công Thức N I Suy Lagrange Và Hermite.docx
M T So Ứng Dụng Của Công Thức N I Suy Lagrange Và Hermite.docx
 
Về Phương Trình Diophantine Dạng Ax2 - By2 = C.docx
Về Phương Trình Diophantine Dạng Ax2 - By2 = C.docxVề Phương Trình Diophantine Dạng Ax2 - By2 = C.docx
Về Phương Trình Diophantine Dạng Ax2 - By2 = C.docx
 
M T So Dạng Toán Cực Tr± Trong L P Hàm Mũ Và Hàm Hyperbolic.docx
M T So Dạng Toán Cực Tr± Trong L P Hàm Mũ Và Hàm Hyperbolic.docxM T So Dạng Toán Cực Tr± Trong L P Hàm Mũ Và Hàm Hyperbolic.docx
M T So Dạng Toán Cực Tr± Trong L P Hàm Mũ Và Hàm Hyperbolic.docx
 
Tổng quát về tích phân
Tổng quát về tích phân Tổng quát về tích phân
Tổng quát về tích phân
 
M T So Ứng Dụng Của Công Thức N I Suy Lagrange Và Hermite (2).docx
M T So Ứng Dụng Của Công Thức N I Suy Lagrange Và Hermite (2).docxM T So Ứng Dụng Của Công Thức N I Suy Lagrange Và Hermite (2).docx
M T So Ứng Dụng Của Công Thức N I Suy Lagrange Và Hermite (2).docx
 
Phương Pháp Diện Tích Và Thể Tích Trong Hình Học Sơ Cấp.doc
Phương Pháp Diện Tích Và Thể Tích Trong Hình Học Sơ Cấp.docPhương Pháp Diện Tích Và Thể Tích Trong Hình Học Sơ Cấp.doc
Phương Pháp Diện Tích Và Thể Tích Trong Hình Học Sơ Cấp.doc
 
Bài Toán Cực Trị Với Điều Kiện Ràng Buộc Bất Đẳng Thức, Hệ Bất Đẳng Thức.docx
Bài Toán Cực Trị Với Điều Kiện Ràng Buộc Bất Đẳng Thức, Hệ Bất Đẳng Thức.docxBài Toán Cực Trị Với Điều Kiện Ràng Buộc Bất Đẳng Thức, Hệ Bất Đẳng Thức.docx
Bài Toán Cực Trị Với Điều Kiện Ràng Buộc Bất Đẳng Thức, Hệ Bất Đẳng Thức.docx
 
Một số phương pháp tìm cực trị của các hàm phân thức Sinh bởi số tự nhiên.docx
Một số phương pháp tìm cực trị của các hàm phân thức Sinh bởi số tự nhiên.docxMột số phương pháp tìm cực trị của các hàm phân thức Sinh bởi số tự nhiên.docx
Một số phương pháp tìm cực trị của các hàm phân thức Sinh bởi số tự nhiên.docx
 
M T So L P Phương Trình Diophantine.docx
M T So L P Phương Trình Diophantine.docxM T So L P Phương Trình Diophantine.docx
M T So L P Phương Trình Diophantine.docx
 
Về Tổng Gauss Và Một Số Ứng Dụng.docx
Về Tổng Gauss Và Một Số Ứng Dụng.docxVề Tổng Gauss Và Một Số Ứng Dụng.docx
Về Tổng Gauss Và Một Số Ứng Dụng.docx
 
Bat đang thức trong so hoc và m t so Dạng toán liên quan.docx
Bat đang thức trong so hoc và m t so Dạng toán liên quan.docxBat đang thức trong so hoc và m t so Dạng toán liên quan.docx
Bat đang thức trong so hoc và m t so Dạng toán liên quan.docx
 
Bat Đang Thức V I Hàm Loi B Ph N Và Ứng Dụng.docx
Bat Đang Thức V I Hàm Loi B Ph N Và Ứng Dụng.docxBat Đang Thức V I Hàm Loi B Ph N Và Ứng Dụng.docx
Bat Đang Thức V I Hàm Loi B Ph N Và Ứng Dụng.docx
 
Kỹ thuật biến đổi tâm tỷ cự Và ứng dụng vào giải toán.doc
Kỹ thuật biến đổi tâm tỷ cự Và ứng dụng vào giải toán.docKỹ thuật biến đổi tâm tỷ cự Và ứng dụng vào giải toán.doc
Kỹ thuật biến đổi tâm tỷ cự Và ứng dụng vào giải toán.doc
 
Phương Trình Bậc Bốn Và Các Hệ Thức Hình Học Trong Tứ Giác Hai Tâm.docx
Phương Trình Bậc Bốn Và Các Hệ Thức Hình Học Trong Tứ Giác Hai Tâm.docxPhương Trình Bậc Bốn Và Các Hệ Thức Hình Học Trong Tứ Giác Hai Tâm.docx
Phương Trình Bậc Bốn Và Các Hệ Thức Hình Học Trong Tứ Giác Hai Tâm.docx
 

More from DV Viết Luận văn luanvanmaster.com ZALO 0973287149

More from DV Viết Luận văn luanvanmaster.com ZALO 0973287149 (20)

Ảnh Hưởng Của Marketing Quan Hệ Đến Lòng Trung Thành Của Khách Hàng.Tình Huốn...
Ảnh Hưởng Của Marketing Quan Hệ Đến Lòng Trung Thành Của Khách Hàng.Tình Huốn...Ảnh Hưởng Của Marketing Quan Hệ Đến Lòng Trung Thành Của Khách Hàng.Tình Huốn...
Ảnh Hưởng Của Marketing Quan Hệ Đến Lòng Trung Thành Của Khách Hàng.Tình Huốn...
 
Phát triển nguồn nhân lực tại Uỷ ban nhân dân huyện Trà Bồng, tỉnh Quảng Ngãi...
Phát triển nguồn nhân lực tại Uỷ ban nhân dân huyện Trà Bồng, tỉnh Quảng Ngãi...Phát triển nguồn nhân lực tại Uỷ ban nhân dân huyện Trà Bồng, tỉnh Quảng Ngãi...
Phát triển nguồn nhân lực tại Uỷ ban nhân dân huyện Trà Bồng, tỉnh Quảng Ngãi...
 
Báo cáo tốt Nghiệp tài chính hợp nhất tại tổng công ty Indochina gol...
Báo cáo tốt Nghiệp  tài chính hợp nhất tại tổng công ty Indochina gol...Báo cáo tốt Nghiệp  tài chính hợp nhất tại tổng công ty Indochina gol...
Báo cáo tốt Nghiệp tài chính hợp nhất tại tổng công ty Indochina gol...
 
Tạo động lực thúc đẩy nhân viên làm việc tại ngân hàng TMCP Ngoại Thương Việt...
Tạo động lực thúc đẩy nhân viên làm việc tại ngân hàng TMCP Ngoại Thương Việt...Tạo động lực thúc đẩy nhân viên làm việc tại ngân hàng TMCP Ngoại Thương Việt...
Tạo động lực thúc đẩy nhân viên làm việc tại ngân hàng TMCP Ngoại Thương Việt...
 
Phát triển công nghiệp trên địa bàn Thành phố Tam Kỳ, Tỉnh Quảng Na...
Phát triển công nghiệp trên địa bàn Thành phố Tam Kỳ, Tỉnh Quảng Na...Phát triển công nghiệp trên địa bàn Thành phố Tam Kỳ, Tỉnh Quảng Na...
Phát triển công nghiệp trên địa bàn Thành phố Tam Kỳ, Tỉnh Quảng Na...
 
Giải pháp phát triển cho vay xuất nhập khẩu tại ngân hàng NN&PTNN ch...
Giải pháp phát triển cho vay xuất nhập khẩu tại ngân hàng NN&PTNN ch...Giải pháp phát triển cho vay xuất nhập khẩu tại ngân hàng NN&PTNN ch...
Giải pháp phát triển cho vay xuất nhập khẩu tại ngân hàng NN&PTNN ch...
 
Hoàn thiện công tác lập báo cáo tài chính hợp nhất tại tổng công ...
Hoàn thiện công tác lập báo cáo tài chính hợp nhất tại tổng công ...Hoàn thiện công tác lập báo cáo tài chính hợp nhất tại tổng công ...
Hoàn thiện công tác lập báo cáo tài chính hợp nhất tại tổng công ...
 
Luận Văn Thạc Sĩ Quản trị thành tích nhân viên tại Cục Hải quan TP Đà Nẵng.doc
Luận Văn Thạc Sĩ  Quản trị thành tích nhân viên tại Cục Hải quan TP Đà Nẵng.docLuận Văn Thạc Sĩ  Quản trị thành tích nhân viên tại Cục Hải quan TP Đà Nẵng.doc
Luận Văn Thạc Sĩ Quản trị thành tích nhân viên tại Cục Hải quan TP Đà Nẵng.doc
 
Hoàn thiện công tác quản lý thuế thu nhập cá nhân tại cục thuế Tỉ...
Hoàn thiện công tác quản lý thuế thu nhập cá nhân tại cục thuế Tỉ...Hoàn thiện công tác quản lý thuế thu nhập cá nhân tại cục thuế Tỉ...
Hoàn thiện công tác quản lý thuế thu nhập cá nhân tại cục thuế Tỉ...
 
Đề Tài Phát triển bền vững nông nghiệp Huyện Ba Tơ, Tỉnh Quảng Ngãi....
Đề Tài Phát triển bền vững nông nghiệp Huyện Ba Tơ, Tỉnh Quảng Ngãi....Đề Tài Phát triển bền vững nông nghiệp Huyện Ba Tơ, Tỉnh Quảng Ngãi....
Đề Tài Phát triển bền vững nông nghiệp Huyện Ba Tơ, Tỉnh Quảng Ngãi....
 
Hoàn thiện công tác bảo trợ xã hội trên địa bàn huyện Phong Điền, tỉnh Thừa T...
Hoàn thiện công tác bảo trợ xã hội trên địa bàn huyện Phong Điền, tỉnh Thừa T...Hoàn thiện công tác bảo trợ xã hội trên địa bàn huyện Phong Điền, tỉnh Thừa T...
Hoàn thiện công tác bảo trợ xã hội trên địa bàn huyện Phong Điền, tỉnh Thừa T...
 
Đề Tài Luận VănPhát triển sản phẩm du lịch tại thành phố Đà Nẵng.doc
Đề Tài Luận VănPhát triển sản phẩm du lịch tại thành phố Đà Nẵng.docĐề Tài Luận VănPhát triển sản phẩm du lịch tại thành phố Đà Nẵng.doc
Đề Tài Luận VănPhát triển sản phẩm du lịch tại thành phố Đà Nẵng.doc
 
Đào tạo nghề cho lao động thuộc diện thu hồi đất trên địa bàn Thàn...
Đào tạo nghề cho lao động thuộc diện thu hồi đất trên địa bàn Thàn...Đào tạo nghề cho lao động thuộc diện thu hồi đất trên địa bàn Thàn...
Đào tạo nghề cho lao động thuộc diện thu hồi đất trên địa bàn Thàn...
 
Tóm Tắt Luận Văn Thạc Sĩ Quản Trị Kinh Doanh Xây dựng chính sách Marketing tạ...
Tóm Tắt Luận Văn Thạc Sĩ Quản Trị Kinh Doanh Xây dựng chính sách Marketing tạ...Tóm Tắt Luận Văn Thạc Sĩ Quản Trị Kinh Doanh Xây dựng chính sách Marketing tạ...
Tóm Tắt Luận Văn Thạc Sĩ Quản Trị Kinh Doanh Xây dựng chính sách Marketing tạ...
 
Đề Tài Nghiên cứu rủi ro cảm nhận đối với mua hàng thời trang trực tuyến.docx
Đề Tài Nghiên cứu rủi ro cảm nhận đối với mua hàng thời trang trực tuyến.docxĐề Tài Nghiên cứu rủi ro cảm nhận đối với mua hàng thời trang trực tuyến.docx
Đề Tài Nghiên cứu rủi ro cảm nhận đối với mua hàng thời trang trực tuyến.docx
 
Giải pháp nâng cao động lực thúc đẩy người lao động tại công ty khai...
Giải pháp nâng cao động lực thúc đẩy người lao động tại công ty khai...Giải pháp nâng cao động lực thúc đẩy người lao động tại công ty khai...
Giải pháp nâng cao động lực thúc đẩy người lao động tại công ty khai...
 
Giải pháp phát triển dịch vụ ngân hàng điện tử tại ngân hàng đầu ...
Giải pháp phát triển dịch vụ ngân hàng điện tử tại ngân hàng đầu ...Giải pháp phát triển dịch vụ ngân hàng điện tử tại ngân hàng đầu ...
Giải pháp phát triển dịch vụ ngân hàng điện tử tại ngân hàng đầu ...
 
Giải pháp phát triển dịch vụ ngân hàng điện tử tại ngân hàng đầu ...
Giải pháp phát triển dịch vụ ngân hàng điện tử tại ngân hàng đầu ...Giải pháp phát triển dịch vụ ngân hàng điện tử tại ngân hàng đầu ...
Giải pháp phát triển dịch vụ ngân hàng điện tử tại ngân hàng đầu ...
 
Quản trị quan hệ khách hàng tại Chi nhánh Viettel Đà Nẵng – Tập đoàn Viễn thô...
Quản trị quan hệ khách hàng tại Chi nhánh Viettel Đà Nẵng – Tập đoàn Viễn thô...Quản trị quan hệ khách hàng tại Chi nhánh Viettel Đà Nẵng – Tập đoàn Viễn thô...
Quản trị quan hệ khách hàng tại Chi nhánh Viettel Đà Nẵng – Tập đoàn Viễn thô...
 
Đề Tài Đánh giá thành tích đội ngũ giảng viên trường Đại Học Phạm ...
Đề Tài Đánh giá thành tích đội ngũ giảng viên trường Đại Học Phạm ...Đề Tài Đánh giá thành tích đội ngũ giảng viên trường Đại Học Phạm ...
Đề Tài Đánh giá thành tích đội ngũ giảng viên trường Đại Học Phạm ...
 

Recently uploaded

Recently uploaded (20)

40 ĐỀ LUYỆN THI ĐÁNH GIÁ NĂNG LỰC ĐẠI HỌC QUỐC GIA HÀ NỘI NĂM 2024 (ĐỀ 1-20) ...
40 ĐỀ LUYỆN THI ĐÁNH GIÁ NĂNG LỰC ĐẠI HỌC QUỐC GIA HÀ NỘI NĂM 2024 (ĐỀ 1-20) ...40 ĐỀ LUYỆN THI ĐÁNH GIÁ NĂNG LỰC ĐẠI HỌC QUỐC GIA HÀ NỘI NĂM 2024 (ĐỀ 1-20) ...
40 ĐỀ LUYỆN THI ĐÁNH GIÁ NĂNG LỰC ĐẠI HỌC QUỐC GIA HÀ NỘI NĂM 2024 (ĐỀ 1-20) ...
 
Báo cáo thực tập tốt nghiệp Kế toán tiền mặt tại Công ty trách nhiệm hữu hạn ...
Báo cáo thực tập tốt nghiệp Kế toán tiền mặt tại Công ty trách nhiệm hữu hạn ...Báo cáo thực tập tốt nghiệp Kế toán tiền mặt tại Công ty trách nhiệm hữu hạn ...
Báo cáo thực tập tốt nghiệp Kế toán tiền mặt tại Công ty trách nhiệm hữu hạn ...
 
Giải pháp nâng cao chất lượng sản phẩm ở Công ty TNHH Sơn Alex Việt Nam
Giải pháp nâng cao chất lượng sản phẩm ở Công ty TNHH Sơn Alex Việt NamGiải pháp nâng cao chất lượng sản phẩm ở Công ty TNHH Sơn Alex Việt Nam
Giải pháp nâng cao chất lượng sản phẩm ở Công ty TNHH Sơn Alex Việt Nam
 
Phân tích báo cáo tài chính tại công ty TNHH xây dựng và thương mại Thịnh An
Phân tích báo cáo tài chính tại công ty TNHH xây dựng và thương mại Thịnh AnPhân tích báo cáo tài chính tại công ty TNHH xây dựng và thương mại Thịnh An
Phân tích báo cáo tài chính tại công ty TNHH xây dựng và thương mại Thịnh An
 
Mở rộng hoạt động cho vay tiêu dùng tại Ngân hàng TMCP Hàng Hải Việt Nam (Mar...
Mở rộng hoạt động cho vay tiêu dùng tại Ngân hàng TMCP Hàng Hải Việt Nam (Mar...Mở rộng hoạt động cho vay tiêu dùng tại Ngân hàng TMCP Hàng Hải Việt Nam (Mar...
Mở rộng hoạt động cho vay tiêu dùng tại Ngân hàng TMCP Hàng Hải Việt Nam (Mar...
 
Nhân tố ảnh hưởng tới động lực làm việc của kiểm toán viên tại Chi nhánh Công...
Nhân tố ảnh hưởng tới động lực làm việc của kiểm toán viên tại Chi nhánh Công...Nhân tố ảnh hưởng tới động lực làm việc của kiểm toán viên tại Chi nhánh Công...
Nhân tố ảnh hưởng tới động lực làm việc của kiểm toán viên tại Chi nhánh Công...
 
Hệ thống ca dao than thân người Việt từ góc nhìn thi pháp
Hệ thống ca dao than thân người Việt từ góc nhìn thi phápHệ thống ca dao than thân người Việt từ góc nhìn thi pháp
Hệ thống ca dao than thân người Việt từ góc nhìn thi pháp
 
4.NGÂN HÀNG KĨ THUẬT SỐ-slide CHƯƠNG 3.pptx
4.NGÂN HÀNG KĨ THUẬT SỐ-slide CHƯƠNG 3.pptx4.NGÂN HÀNG KĨ THUẬT SỐ-slide CHƯƠNG 3.pptx
4.NGÂN HÀNG KĨ THUẬT SỐ-slide CHƯƠNG 3.pptx
 
PHONG TRÀO “XUNG KÍCH, TÌNH NGUYỆN VÌ CUỘC SỐNG CỘNG ĐỒNG” CỦA ĐOÀN TNCS HỒ...
PHONG TRÀO “XUNG KÍCH, TÌNH NGUYỆN VÌ CUỘC SỐNG   CỘNG ĐỒNG” CỦA ĐOÀN TNCS HỒ...PHONG TRÀO “XUNG KÍCH, TÌNH NGUYỆN VÌ CUỘC SỐNG   CỘNG ĐỒNG” CỦA ĐOÀN TNCS HỒ...
PHONG TRÀO “XUNG KÍCH, TÌNH NGUYỆN VÌ CUỘC SỐNG CỘNG ĐỒNG” CỦA ĐOÀN TNCS HỒ...
 
Thực trạng ứng dụng công nghệ trong lĩnh vực giống cây trồng: Nghiên cứu điển...
Thực trạng ứng dụng công nghệ trong lĩnh vực giống cây trồng: Nghiên cứu điển...Thực trạng ứng dụng công nghệ trong lĩnh vực giống cây trồng: Nghiên cứu điển...
Thực trạng ứng dụng công nghệ trong lĩnh vực giống cây trồng: Nghiên cứu điển...
 
TỔNG HỢP HƠN 100 ĐỀ THI THỬ TỐT NGHIỆP THPT TIẾNG ANH 2024 - TỪ CÁC TRƯỜNG, ...
TỔNG HỢP HƠN 100 ĐỀ THI THỬ TỐT NGHIỆP THPT TIẾNG ANH 2024 - TỪ CÁC TRƯỜNG, ...TỔNG HỢP HƠN 100 ĐỀ THI THỬ TỐT NGHIỆP THPT TIẾNG ANH 2024 - TỪ CÁC TRƯỜNG, ...
TỔNG HỢP HƠN 100 ĐỀ THI THỬ TỐT NGHIỆP THPT TIẾNG ANH 2024 - TỪ CÁC TRƯỜNG, ...
 
Hoàn thiện công tác kiểm soát chi NSNN qua Kho bạc Nhà nước huyện Tri Tôn – t...
Hoàn thiện công tác kiểm soát chi NSNN qua Kho bạc Nhà nước huyện Tri Tôn – t...Hoàn thiện công tác kiểm soát chi NSNN qua Kho bạc Nhà nước huyện Tri Tôn – t...
Hoàn thiện công tác kiểm soát chi NSNN qua Kho bạc Nhà nước huyện Tri Tôn – t...
 
Báo cáo tốt nghiệp Đánh giá rủi ro môi trường ô nhiễm hữu cơ trong nước thải ...
Báo cáo tốt nghiệp Đánh giá rủi ro môi trường ô nhiễm hữu cơ trong nước thải ...Báo cáo tốt nghiệp Đánh giá rủi ro môi trường ô nhiễm hữu cơ trong nước thải ...
Báo cáo tốt nghiệp Đánh giá rủi ro môi trường ô nhiễm hữu cơ trong nước thải ...
 
Vận dụng thi pháp học vào phân tích truyện ngắn Chiếc thuyền ...
Vận dụng thi pháp học vào phân tích truyện ngắn Chiếc thuyền ...Vận dụng thi pháp học vào phân tích truyện ngắn Chiếc thuyền ...
Vận dụng thi pháp học vào phân tích truyện ngắn Chiếc thuyền ...
 
Báo cáo thực tập tốt nghiệp Phân tích hiệu quả hoạt động huy động và cho vay ...
Báo cáo thực tập tốt nghiệp Phân tích hiệu quả hoạt động huy động và cho vay ...Báo cáo thực tập tốt nghiệp Phân tích hiệu quả hoạt động huy động và cho vay ...
Báo cáo thực tập tốt nghiệp Phân tích hiệu quả hoạt động huy động và cho vay ...
 
Tiểu luận tổng quan về Mối quan hệ giữa chu kỳ kinh tế và đầu tư trong nền ki...
Tiểu luận tổng quan về Mối quan hệ giữa chu kỳ kinh tế và đầu tư trong nền ki...Tiểu luận tổng quan về Mối quan hệ giữa chu kỳ kinh tế và đầu tư trong nền ki...
Tiểu luận tổng quan về Mối quan hệ giữa chu kỳ kinh tế và đầu tư trong nền ki...
 
30 ĐỀ PHÁT TRIỂN THEO CẤU TRÚC ĐỀ MINH HỌA BGD NGÀY 22-3-2024 KỲ THI TỐT NGHI...
30 ĐỀ PHÁT TRIỂN THEO CẤU TRÚC ĐỀ MINH HỌA BGD NGÀY 22-3-2024 KỲ THI TỐT NGHI...30 ĐỀ PHÁT TRIỂN THEO CẤU TRÚC ĐỀ MINH HỌA BGD NGÀY 22-3-2024 KỲ THI TỐT NGHI...
30 ĐỀ PHÁT TRIỂN THEO CẤU TRÚC ĐỀ MINH HỌA BGD NGÀY 22-3-2024 KỲ THI TỐT NGHI...
 
40 ĐỀ LUYỆN THI ĐÁNH GIÁ NĂNG LỰC ĐẠI HỌC QUỐC GIA HÀ NỘI NĂM 2024 (ĐỀ 21-30)...
40 ĐỀ LUYỆN THI ĐÁNH GIÁ NĂNG LỰC ĐẠI HỌC QUỐC GIA HÀ NỘI NĂM 2024 (ĐỀ 21-30)...40 ĐỀ LUYỆN THI ĐÁNH GIÁ NĂNG LỰC ĐẠI HỌC QUỐC GIA HÀ NỘI NĂM 2024 (ĐỀ 21-30)...
40 ĐỀ LUYỆN THI ĐÁNH GIÁ NĂNG LỰC ĐẠI HỌC QUỐC GIA HÀ NỘI NĂM 2024 (ĐỀ 21-30)...
 
Báo cáo tốt nghiệp Hoàn thiện công tác đào tạo và phát triển nguồn nhân lực c...
Báo cáo tốt nghiệp Hoàn thiện công tác đào tạo và phát triển nguồn nhân lực c...Báo cáo tốt nghiệp Hoàn thiện công tác đào tạo và phát triển nguồn nhân lực c...
Báo cáo tốt nghiệp Hoàn thiện công tác đào tạo và phát triển nguồn nhân lực c...
 
Báo cáo tốt nghiệp Đánh giá thực trạng an toàn vệ sinh lao động và rủi ro lao...
Báo cáo tốt nghiệp Đánh giá thực trạng an toàn vệ sinh lao động và rủi ro lao...Báo cáo tốt nghiệp Đánh giá thực trạng an toàn vệ sinh lao động và rủi ro lao...
Báo cáo tốt nghiệp Đánh giá thực trạng an toàn vệ sinh lao động và rủi ro lao...
 

Một số phương pháp giải các đề thi olympic Về phương trình diophant.docx

  • 1. ĐẠI HỌC THÁI NGUYÊN TRƢỜNG ĐẠI HỌC KHOA HỌC --------------------------- Tải tài liệu tại sividoc.com Viết đề tài giá sinh viên – ZALO:0973.287.149-TEAMLUANVAN.COM ĐẶNG THỊ THU HÀ MỘT SỐ PHƢƠNG PHÁP GIẢI CÁC ĐỀ THI OLYMPIC VỀ PHƢƠNG TRÌNH DIOPHANT LUẬN VĂN THẠC SĨ TOÁN HỌC THÁI NGUYÊN - 2019
  • 2. ĐẠI HỌC THÁI NGUYÊN TRƢỜNG ĐẠI HỌC KHOA HỌC --------------------------- Tải tài liệu tại sividoc.com Viết đề tài giá sinh viên – ZALO:0973.287.149-TEAMLUANVAN.COM ĐẶNG THỊ THU HÀ MỘT SỐ PHƢƠNG PHÁP GIẢI CÁC ĐỀ THI OLYMPIC VỀ PHƢƠNG TRÌNH DIOPHANT Chuyên ngành: Phƣơng pháp Toán sơ cấp Mã số: 8 46 01 13 LUẬN VĂN THẠC SĨ TOÁN HỌC NGƯỜI HƯỚNG DẪN KHOA HỌC GS.TSKH. Nguyễn Văn Mậu THÁI NGUYÊN - 2019
  • 3. i Viết đề tài giá sinh viên – ZALO:0973.287.149-TEAMLUANVAN.COM L i cảm ơn Lu n văn này được hoàn thành tại trường Đại hoc Khoa hoc - Đại hoc Thái Nguyên. Tác giả xin bày tỏ lòng biet ơn sâu sac đoi với GS.TSKH Nguyen Văn M u (Trường ĐH Khoa hoc Tự nhiên, ĐHQGHN), thay đã trực tiep hướng dan t n tình và đ®ng viên tác giả trong suot thời gian nghiên cáu vàa qua. Xin chân thành cảm ơn tới các quý thay, cô giáo đã trực tiep giảng dạy lớp cao hoc Toán K11, các bạn hoc viên, và các bạn đong nghi p đã tạo đieu ki n thu n lợi, đ®ng viên giúp đơ tác giả trong quá trình hoc t p và nghiên cáu tại trường. Tác giả cũng xin bày tỏ lòng biet ơn sâu sac tới gia đình và người thân luôn khuyen khích đ®ng viên tác giả trong suot quá trình hoc cao hoc và viet lu n văn này. M c dù có nhieu co gang nhưng lu n văn khó tránh khỏi nhǎng thieu sót và hạn che. Tác giả mong nh n được nhǎng ý kien đóng góp của các thay cô và các bạn đoc đe lu n văn được hoàn thi n hơn. Xin chân thành cảm ơn! Thái Nguyên, tháng 11 năm 2019 Tác giả Đ ng Thị Thu Hà
  • 4. ii Viết đề tài giá sinh viên – ZALO:0973.287.149-TEAMLUANVAN.COM Mnc lnc M ĐAU 1 Chương 1. Phương trình Diophant và h Diophant cơ bản 2 1.1 Phương trình Diophant tuyen tính . . . . . . . . . . . . . . . . . . 2 1.1.1 Nghi m riêng . . . . . . . . . . . . . . . . . . . . . . . . . 3 1.1.2 Nghi m nguyên dương . . . . . . . . . . . . . . . . . . . . . 9 1.2 Nghi m nguyên dương của h phương trình Diophant tuyen tính cơ bản.................................................................................................................. 10 Chương 2. Các phương pháp giải phương trình Diophant 19 2.1 Phương pháp phân tích thành nhân tả...................................................... 19 2.2 Phương pháp đong dư.................................................................................. 24 2.3 Phương pháp đánh giá ................................................................................. 25 2.4 Phương pháp tham so hóa........................................................................... 27 2.5 Phương pháp quy nạp toán hoc.................................................................. 30 2.6 Phương pháp xuong thang........................................................................... 33 2.7 M®t so phương pháp khác ........................................................................... 40 Chương 3. Các dạng toán liên quan đen phương trình và h phương trình Diophant 47 3.1 M®t so dạng toán ve đa thác nguyên......................................................... 47 3.2 M®t so dạng toán lượng giác liên quan ...................................................... 50 3.3 M®t so dạng toán thi Olympic liên quan ............................................... 66 KET LU N 77 TÀI LI U THAM KHẢO 78
  • 5. 1 Viết đề tài giá sinh viên – ZALO:0973.287.149-TEAMLUANVAN.COM M đau Trong các kì thi hoc sinh giỏi toán các cap, Olympic Toán sinh viên, các bài toán liên quan tới phương trình Diophant (dạng tuyen tính và phi tuyen) thường xuyên được đe c p. Nhǎng dạng toán này thường được xem là thu®c loại khó vì phan kien thác ve phương trình Diophant tőng quát không nam trong chương trình chính thác của giáo trình So hoc và Đại so b c trung hoc phő thông. Đe đáp áng nhu cau boi dương giáo viên và boi dương hoc sinh giỏi ve chuyên đe phương trình Diophant, tôi chon đe tài lu n văn "M®t so phương pháp giải các đe thi Olympic ve phương trình Diophant". Tiep theo, khảo sát m®t so lớp h phương trình Diophant liên quan. Cau trúc lu n văn gom 3 chương: Chương 1. Các kien thác bő túc ve so hoc và phương trình Diophant cơ bản. Chương 2. Các phương pháp giải phương trình Diophant. Chương 3. Các dạng toán liên quan đen h phương trình Diophant. Tiep theo, cuoi các chương đeu trình bày các bài t p áp dụng và giải các đe thi HSG quoc gia và Olympic liên quan.
  • 6. 2 Viết đề tài giá sinh viên – ZALO:0973.287.149-TEAMLUANVAN.COM Chương 1. Phương trình Diophant và h Diophant cơ bản 1.1 Phương trình Diophant tuyen tính Ta nhac lại thu t toán Euclid và liên phân so đã được trình bày tương đoi chi tiet trong chương trình toán b c THCS. Định nghĩa 1.1. So nguyên c được goi là m®t ước so chung của hai so nguyên a và b (không đong thời bang không) neu c chia het a và c chia het b (hay a và b đeu chia het cho c). Định nghĩa 1.2 (xem [3,5, 7]). M®t ước so chung d của hai so nguyên a và b (không đong thời bang không) được goi là ước so chung lớn nhat của a và b neu moi ước so chung c của a và b đeu là ước của d. Nh n xét 1.1. Neu d là ước so chung lớn nhat của a và b thì −d cũng là ước so chung lớn nhat của a và b. Vì v y, ta quy ước rang ước so chung lớn nhat của a và b là so nguyên dương. Ước so chung lớn nhat của hai so a và b được ký hi u là (a, b) hay gcd(a,b) (greatest common divisor). Như v y d = (a, b) hay d = gcd(a, b). Ví dn 1.1. (25,30) = 5, (25,-72) = 1. Định nghĩa 1.3 (xem [3,5,7]). M®t so nguyên c được goi là m®t ước so chung của b® n so nguyên a1, a2, a3, . . . , an (không đong thời bang không) neu c là ước của moi so đó. Định nghĩa 1.4 (xem [5,7]). M®t ước so chung d của b® n so nguyên a1, a2, a3, . . . , an (không đong thời bang không) được goi là ước so chung lớn nhat của a1, a2, a3, . . . , an neu moi ước so chung c của a1, a2, a3, . . . , an đeu là ước của d. Tương tự, ta cũng quy ước rang ước so chung lớn nhat của n so nguyên a1, a2, a3, . . . , an là so nguyên dương.
  • 7. 3 Viết đề tài giá sinh viên – ZALO:0973.287.149-TEAMLUANVAN.COM n−1 x = ( − 1 ) .c.q 0 n−1 x0 = (−1)n−1 .c.qn−1 Bài toán 1.1. Giải phương trình Diophant tuyen tính Ước so chung lớn nhat của a1, a2, a3, . . . , an ký hi u là (a1, a2, a3, . . . , an) hay gcd(a1, a2, a3, . . . , an). Như v y d = (a1, a2, a3, . . . , an) hay d = gcd(a1, a2, a3, . . . , an). Định lý 1.1 (Ước so chung lớn nhat của nhieu so). Cho các so nguyên a1, a2, a3, . . . , an không đong thời bang không. Khi đó ton tại ước so chung lớn nhat của a1, a2, a3, . . . , an. Tính chat 1.1. Cho a, b, q, r là các so nguyên (a2 + b2 /= 0). Neu a = bq + r và 0 ≤ r < |b| thì (a, b) = (b, r). 1.1.1 Nghi m riêng Trong mục này, ta trình bày hai thu t toán tìm nghi m riêng của phương trình Diophant, đó là thu t toán giản phân và thu t toán Euclid. Xét phương trình Diophant tuyen tính Ax + By = C. (1.1) Đe tìm nghi m riêng dựa vào giản phân, ta tien hành thực hi n theo các bước như sau: - Bước 1. Tìm d = (A, B) đe đưa phương trình (1.1) ve phương trình (1.2) với (a, b) = 1. phương trình Diophant tuyen tính - Bước 2. Viet a |b| ax + by = c. (1.2) = [a0; a1, a2, . . . , an]. p - Bước 3. Tính giản phân Cn−1 = [a0; a1, . . . , an−1] = qn−1. n−1 . Suy ra p và qn−1 n−1 - Bước 4. Suy ra m®t nghi m riêng (x0, y0) của phương trình (1.2). Neu b > 0 thì Neu b < 0 thì y0 = (−1)n .c.pn−1. y0 = (−1)n−1 .c.pn−1. 342x − 123y = 15. (1.3)
  • 8. 4 Viết đề tài giá sinh viên – ZALO:0973.287.149-TEAMLUANVAN.COM 0 d Cm Lài giai. Phương trình đã cho tương đương với phương trình 114x − 41y = 5. (1.4) Ta có 114 41 = [2; 1, 3, 1, 1, 4], với n = 5. C = p4 4 q4 25 = [2; 1, 3, 1, 1] = 9 nên p4 = 25 (p4, q4) = 1 Do b = −41 < 0 nên m®t nghi m riêng của (1.4) là x0 = (−1)5−1 .5.9 = 45 y0 = (−1)5−1 .5.25 = 125. q4 = 9. x = 45 + 41t y = 125 + 114t , t ∈ Z. Đe tìm nghi m riêng dựa vào thu t toán Euclid, ta tien hành thực hi n theo các bước như sau: - Bước 1. Xác định d = (|A| , |B|) theo thu t toán Euclid mở r®ng. - Bước 2. Bieu thị d như m®t tő hợp tuyen tính của A và B, chȁng hạn d = nA + mB (n, m ∈ Z) . C - Bước 3. Nhân hai ve đȁng thác trên với d ta thu được Cn Cm A + B d d = C. - Bước 4. Suy ra m®t nghi m riêng (x0, y0) của phương trình (1.1) là Cn x0 = d y = . Bài toán 1.2. Giải phương trình Diophant tuyen tính 342x − 123y = 15. (1.5) V y nghi m của phương trình (1.4), tác phương trình (1.14) là
  • 9. 5 Viết đề tài giá sinh viên – ZALO:0973.287.149-TEAMLUANVAN.COM 3 , t ∈ Z Lài giai. Vì (342, −123) = 3 |15 nên phương trình đã cho có nghi m. Ta có 342 = 123.2 + 96, 123 = 96.1 + 27, 96 = 27.3 + 15, 27 = 15.1 + 12, 15 = 12.1 + 3, 12 = 3.4 + 0. Suy ra 3 = 15 − 12.1 = 15 − (27 − 15.1).1 = 15.2 − 27.1 = (96 − 27.3).2 − 27.1 = 96.2 − 27.7 = 96.2 − (123 − 96.1).7 = 96.9 − 123.7 = (342 − 123.2).9 − 123.7 = = 342.9 − 123.25. Suy ra 342.45 − 123.125 = 15. Tà đó, phương trình (1.14) có m®t nghi m riêng là (x0; y0) = (45; 125). Suy ra nghi m tőng quát của phương trình (1.14) là x = 45 + 123 t hay 342 y = 125 + 3 t x = 45 + 41t y = 125 + 114t , t ∈ Z. Định lj 1.2. Xét phương trình Diophant tuyen tính a1x1 + a2x2 + . . . + anxn = c. (1.6) 1. Phương trình (1.6) có nghi m khi và chỉ khi d = (a1, a2, . . . , an) |c. 2. Neu phương trình (1.6) có nghi m thì nó sě có vô so nghi m.
  • 10. 6 Viết đề tài giá sinh viên – ZALO:0973.287.149-TEAMLUANVAN.COM Σ .Σ . Σ Σ Σ Σ k ChGng minh. 1) ⇒). Giả sả (x1, x2, . . . , xn) là m®t nghi m của phương trình (1.6), tác là n aixi = c. i=1 n Ta có d = (a1, a2, . . . , an), suy ra d i=1 aixi , suy ra d |c. ⇐) Ta cháng minh khȁng định bang phương pháp quy nạp theo n. Với n = 2, khȁng định là đúng. Giả sả khȁng định đúng với n = k (k ≥ 2). Với n = k + 1, xét d = (a1, a2, . . . , ak+1) |c, đ t h = (a1, a2, . . . , ak). Khi đó, ta có Suy ra, ton tại t, xk+1 ∈ Z đe d = (h, ak+1) |c ht + ak+1xk+1 = c. Vì h |ht nên theo giả thiet quy nạp sě ton tại x1, x2, . . . , xk ∈ Z đe k aixi = ht. i=1 Do đó k+1 aixi = c. i=1 V y nên phương trình a1x1+a2x2+. . .+ak+1xk+1 = c có nghi m (x1, x2, . . . , xk+1). 2). Ta cháng minh khȁng định bang phương pháp quy nạp theo n. Với n = 2: khȁng định là đúng. Giả sả khȁng định đúng với n = k (k ≥ 2), tác là phương trình aixi = c neu có nghi m thì sě có vô so nghi m. Với n = k + 1, ta i=1 k Σ +1 sě cháng tỏ phương trình aixi = c neu có nghi m thì sě có vô so nghi m. i=1 k Σ +1 Th t v y, goi (t1, t2, . . . , tk+1) là m®t nghi m của phương trình là k+1 aiti = c. i=1 i=1 aixi = c, tác
  • 11. 7 Viết đề tài giá sinh viên – ZALO:0973.287.149-TEAMLUANVAN.COM Σ Σ Σ Σ Σ Khi đó Xét phương trình k aiti = c − ak+1tk+1. i=1 k aixi = c − ak+1tk+1 (1.7) i=1 có ve phải là hang so và có m®t nghi m là (t1, t2, . . . , tk) nên theo giả thiet quy nạp thì phương trình này có vô so nghi m. Úng với moi nghi m (x1, x2, . . . , xk) của (6) thì phương trình k Σ +1 k+1 i=1 aixi = c có Ta de dàng cháng minh được nghi m tőng quát của phương trình (1.6) có dạng x1 = x1 n−1 + i=1 n Σ −1 Li1ti , x2 = x2 + . . . xn = xn + i=1 n−1 i=1 Li2ti , Linti. trong đó (x1, x2, . . . , xn) là m®t nghi m riêng của (1.6), ti ∈ Z, ∀i = 1, 2, . . . , n. Tuy nhiên, các h so Lij (i, j = 1, 2, . . . , n) không có công thác tính tường minh. Bài toán 1.3. Giải phương trình Diophant tuyen tính 6x + 15y + 10z = 3. (1.8) Lài giai. Phương trình đã cho tương đương với 6(x + z) + 15y + 4z = 3. Đ t u = x + z, ta thu được phương trình 15y + 4z = 3 − 6u. (1.9) Ta nh n thay phương trình 15y + 4z = 1 i=1 nghi m là (x1, x2, . . . , xk, tk+1). Cháng tỏ phương trình nghi m. aixi = c có vô so
  • 12. 8 Viết đề tài giá sinh viên – ZALO:0973.287.149-TEAMLUANVAN.COM . x = −12 + 25u + 15t có m®t nghi m riêng là (y1; z1) = (−1; 4) nên phương trình (7a) có m®t nghi m riêng là (y0; z0) = (−3 + 6u; 12 − 24u). Do đó nghi m tőng quát của phương trình (1.9) là y = −3 + 6u + 4t z = 12 − 24u − 15t , u, t ∈ Z. M t khác, u = x + z suy ra x = u − z = −12 + 25u + 15t. V y nên phương trình (1.8) có nghi m tőng quát là y = −3 + 6u + 4t z = 12 − 24u − 15t , u, t ∈ Z. Nh n xét 1.2. Có the tóm lược cách giải phương trình Diophant tuyen tính nhieu ȁn trên như sau: Tà phương trình Diophant tuyen tính n ȁn, ta đưa ve phương trình Diophant tuyen tính n − 1 ȁn (giảm ȁn), tiep tục như v y, sau hǎu hạn bước, ta nh n được phương trình Diophant tuyen tính 2 ȁn. Moi lan giảm so ȁn như v y ta lại giải phương trình Diophant tuyen tính 2 ȁn (cháa tham so). Vì lě đó, ta thu được h nghi m phụ thu®c vào n − 1 tham so. Ví dn 1.2. Xét phương trình (1.6) với n ≥ 3, ai 0, ∀i = 1, 2, . . . , n. Goi d = (an−1, an). Khi đó an−1 = d.bn−1, an = d.bn, (bn−1, bn) = 1. Do đó (1.6) trở thành a1x1 + a2x2 + . . . + an−2xn−2 + d (bn−1xn−1 + bnxn) = c. (1.10) Đưa vào ȁn mới t bang h thác bn−1xn−1 + bnxn = t. (1.11) Khi đó (1.6) trở thành a1x1 + a2x2 + . . . + an−2xn−2 + dt = c. (1.12) Giả sả x1, x2, . . . , xn−2, t là m®t nghi m nguyên của (1.12). Úng với so t xác định, xét phương trình bn−1xn−1 + bnxn = t. (1.13) Do (bn−1, bn) = 1 t nên (1.13) nhat định có nghi m nguyên, chȁng hạn (xn−1, xn). Khi đó, rõ ràng (x1, x2, . . . , xn−2, xn−1, xn) là nghi m nguyên của (1.6). De thay moi nghi m nguyên của (1.13) chính là nghi m của (1.12) với đieu ki n (1.11).
  • 13. 9 Viết đề tài giá sinh viên – ZALO:0973.287.149-TEAMLUANVAN.COM ai x = 25 thì z = 2. Hơn nǎa, tà giả thiet bài ra, suy ra x . 5 và y . 2. 1.1.2 Nghi m nguyên dương Xét phương trình Dipophant tuyen tính a1x1 + a2x2 + . . . + anxn = c (1.14) với các h so ai, c ∈ Z+ , các bien so xi ∈ Z+ , ∀i = 1, 2, . . . , n. Khi đó phương trình (1.14) luôn có hǎu hạn nghi m nguyên dương x = (x1, x2, . . . , xn). Tà giả thiet bài ra, ta có the hạn che đieu ki n của các bien so bởi 1 ≤ xi ≤ (c + ai) − (a1 + a2 + . . . + an) , ∀i = 1, 2, . . . , n. Khi đó, cách đơn giản nhat đe tìm nghi m nguyên dương x = (x1, x2, . . . , xn) của phương trình (1.14) là ta cho m®t bien so xi nào đó lan lượt chạy qua các giá trị có the có của nó và tìm các bien so còn lại tà phương trình đã cho. Bài toán 1.4. Tìm các nghi m nguyên dương của phương trình Diophant tuyen tính 6x + 15y + 10z = 200. (1.15) Lài giai. Theo giả thiet, ta có 1 ≤ x ≤ 29 1 ≤ y ≤ 12 1 ≤ z ≤ 17. - Với y = 2, ta có 3x + 5z = 85 nên x = 5 thì z = 14; x = 10 thì z = 11; x = 15 thì z = 8; x = 20 thì z = 5; - Với y = 4, ta có 3x + 5z = 70 nên x = 5 thì z = 11; x = 10 thì z = 8; x = 15 thì z = 5; x = 20 thì z = 2.
  • 14. 10 Viết đề tài giá sinh viên – ZALO:0973.287.149-TEAMLUANVAN.COM x = 10 thì z = 2. - Với y = 6, ta có 3x + 5z = 55 nên x = 5 thì z = 8; x = 10 thì z = 5; x = 15 thì z = 2. - Với y = 8, ta có 3x + 5z = 40 nên x = 5 thì z = 5; - Với y = 10, ta có 3x + 5z = 25 nên x = 5 thì z = 2. - Với y = 12, ta có 3x + 5z = 10. Phương trình này không có nghi m nguyên dương. Như v y, phương trình (1.15) có cả thảy 15 nghi m nguyên dương (x, y, z) bao gom (5, 2, 14) , (5, 4, 11) , (5, 6, 8) , (5, 8, 5) , (5, 10, 2) , (10, 2, 11) , (10, 4, 8) , (10, 6, 5) , (10, 8, 2) , (15, 2, 8) , (15, 4, 5) , (15, 6, 2) , (20, 2, 5) , (20, 4, 2) , (25, 2, 2) . 1.2 Nghi m nguyên dương của h phương trình Diophant tuyen tính cơ bản Phan này đe c p tới h phương trình Diophant tuyen tính, trình bày ve h hai phương trình ba ȁn với nguy n nguyên dương giải bang phương pháp ”chìa khóa” (xem [1],[3] và [5]). Bài toán tong quát 1.1. Tìm nghi m nguyên dương của h phương trình Phương pháp giải: a1x + b1y + c1z = s1 a2x + b2y + c2z = s2. (1.16) Ta goi ”chìa khóa” của h (1) là b® so (x0, y0, z0) thỏa mãn đieu ki n: x0, y0, z0 ∈ Z và
  • 15. 11 Viết đề tài giá sinh viên – ZALO:0973.287.149-TEAMLUANVAN.COM a x + b y + c z = 0.20 2 0 2 0 4x + y + z = 322 3 2 1 1 1 1 a1x0 + b1y0 + c1z0 = 0 Neu (x1, y1, z1) là m®t nghi m của h (1.16) thì với x2 = x1 + mx0 ; y2 = y1 + my0 ; z2 = z1 + mz0, ta cũng có a1x2 + b1y2 + c1z2 = s1 ; a2x2 + b2y2 + c2z2 = s2, tác là (x2, y2, z2) cũng là m®t nghi m của h (1). Neu các nghi m này nguyên dương thì ta có nghi m nguyên dương. Ta đe ý rang trong h (1) neu x xác định thì y và z cũng xác định. Vì v y khi ta cho x chạy qua tat cả các giá trị nguyên dương có the có của nó, ta sě tìm được các giá trị tương áng của y và z. Trong các trường hợp này, có bao nhiêu trường hợp các giá trị tương áng của y và z đeu nguyên dương thì h có bay nhiêu nghi m nguyên dương. Bài toán 1.5. Tìm nghi m nguyên dương của h phương trình x + y + z = 134 (1.17) Lài giai. Ta có 2x + 2 y + 3 z = 161. (1.17) ⇔ x + y + z = 134 Suy ra z = 3, x = 63, y = 68. x + y + z = 134 3x − 3 z = 188. Suy ra h (1.17) có m®t nghi m (x1, y1, z1) = (63, 68, 3) . Tìm chìa khóa (x0, y0, z0) thỏa mãn đieu ki n x0 + y0 + z0 = 0 Suy ra 2x0 + 2 y0 + 3 z0 = 0. x0 + y0 + z0 = 0 3x0 − 3 z0 = 0. Suy ra x0 = 1, z0 = 9, y0 = −10 hay (x0, y0, z0) = (1, −10, 9). Khi đó 1 1 ⇔
  • 16. 12 Viết đề tài giá sinh viên – ZALO:0973.287.149-TEAMLUANVAN.COM 2 ⇔ 1 suy ra x2 = x1 + mx0 y2 = y1 + my0 z2 = z1 + mz0, x2 = 63 + m.1 y2 = 68 + m. (−10) z2 = 3 + m.9. • Với m = 1, h có nghi m (64, 58, 12); • Với m = 2, h có nghi m (65, 48, 21); • Với m = 3, h có nghi m (66, 38, 30); • Với m = 4, h có nghi m (67, 28, 39); • Với m = 5, h có nghi m (68, 18, 48); • Với m = 6, h có nghi m (69, 8, 57); • m ∈ Z− , neu x = 63 + m.1 thì y = 68 + m.(−10) ; z = 3 + m.9 < 0. • m ∈ Z+ , neu x = 69 + m.1 thì y = 8 + m.(−10) < 0. V y bài toán có 7 nghi m nguyên dương. Bài toán 1.6. Tìm nghi m nguyên dương của h phương trình x + y + z = 134 (2) 2x + 2 y + 3 z = 40. Lài giai. (2) ⇔ −x − y − z = −134 4x + y + 3 z = 80 3x − 3 z = −54 −x − y − z = −134 nên x = 1, z = 171, y = −38. Suy ra h (2) có m®t nghi m (x1, y1, z1) = (1, −38, 171) . Tìm chìa khóa (x0, y0, z0) thỏa mãn đieu ki n: 1 1
  • 17. 13 Viết đề tài giá sinh viên – ZALO:0973.287.149-TEAMLUANVAN.COM 1 15x + 9y + z = 300 1 1 1 x0 + y0 + z0 = 0 2x0 + 2 y0 + 3 z0 = 0 ”Chìa khóa” van là (x0, y0, z0) = (1, −10, 9) . Khi đó suy ra x2 = x1 + mx0 y2 = y1 + my0 z2 = z1 + mz0 x2 = 1 + m.1 y2 = −38 + m. (−10) z2 = 171 + m.9. • Với m ∈ Z+ , neu x = 1 + m.1 suy ra y = −38 + m.(−10) < 0. V y bài toán không có nghi m nguyên dương. Bài toán 1.7. Tìm nghi m nguyên dương của h phương trình x + y + z = 100 5x + 3y + 3 z = 100 (1.18) Lài giai. Ta có (1.18) ⇔ x + y + z = 100 Suy ra x . 4 nên x = 4, y = 18, z = 78. x + y + z = 100 14x + 8y = 200. Suy ra h (1.18) có m®t nghi m (x1, y1, z1) = (4, 18, 78) . Tìm ”chìa khóa” (x0, y0, z0) thỏa mãn đieu ki n: x0 + y0 + z0 = 0 5x0 + 3y0 + 3 z0 = 0 x0 + y0 + z0 = 0 14x0 + 8y0 = 0 Suy ra h có nghi m x0 = 4, y0 = −7, z0 = 3. Suy ra ”chìa khóa” (x0, y0, z0) = (4, −7, 3). ⇔ ⇔
  • 18. 14 Viết đề tài giá sinh viên – ZALO:0973.287.149-TEAMLUANVAN.COM 3x + 4y + 5z = 50 Khi đó suy ra x2 = x1 + mx0 y2 = y1 + my0 z2 = z1 + mz0 x2 = 4 + m.4 y2 = 18 + m. (−7) z2 = 78 + m.3 • Với m = 1 h có nghi m (8, 11, 81); • Với m = 2 h có nghi m (12, 4, 84); • m ∈ Z− thì x = 4 + m.4 ≤ 0; • m ∈ Z+ , neu x = 12 + m.4 thì y = 4 + m.(−7) < 0. V y bài toán có 3 nghi m nguyên dương. Bài toán 1.8. Tìm nghi m nguyên dương của h phương trình Lài giai. Ta có x + y + z = 12 3x + 4y + 5z = 50 (1.19) (1.19) ⇔ 3x + 3y + 3z = 36 Suy ra y = 2, z = 6, x = 4 3x + 3y + 3z = 36 x + 2z = 14 V y h (1.19) có m®t nghi m (x1, y1, z1) = (4, 2, 6) . Tìm ”chìa khóa” (x0, y0, z0) thỏa mãn đieu ki n: x0 + y0 + z0 = 0 3x0 + 4y0 + 5z0 = 0 Suy ra x0 + y0 + z0 = 0 y0 + 2z0 = 0 ⇔
  • 19. 15 Viết đề tài giá sinh viên – ZALO:0973.287.149-TEAMLUANVAN.COM ⇔ −x − 15y + 20z = 0 0 0 0 nên ta có z0 = 1, y0 = −2, x0 = 1. V y nên, ”chìa khóa” (x0, y0, z0) = (1, −2, 1) x2 = x1 + mx0 y2 = y1 + my0 z2 = z1 + mz0 ⇔ x2 = 4 + m.1 y2 = 2 + m. (−2) z2 = 6 + m.1 • Với m = −1 h có nghi m (3, 4, 5); • Với m = −2 h có nghi m (2, 6, 4); • Với m = −3 h có nghi m (1, 8, 3); • m ∈ Z− thì x = 1 + m.1 ≤ 0; • m ∈ Z+ , neu x = 4 + m.1 thì y = 2 + m.(−2) ≤ 0. V y bài toán có 4 nghi m nguyên dương. Bài toán 1.9. Tìm nghi m nguyên dương của h phương trình x + y + z = 6 −x − 15y + 20z = 29 (1.20) Lài giai. Ta có (1.20) x + y + z = 6 −14y + 21z = 35 Suy ra y = 2, z = 3, x = 1. V y nên h (1.20) có m®t nghi m (x1, y1, z1) = (1, 2, 3) . Tìm ”chìa khóa” (x0, y0, z0) thỏa mãn đieu ki n: x0 + y0 + z0 = 0 V y nên, ta có y0 = 3, z0 = 2, x0 = −5. x0 + y0 + z0 = 0 −14y0 + 21z0 = 0 Suy ra ”chìa khóa” (x0, y0, z0) = (−5, 3, 2). Khi đó x2 = x1 + mx0 z2 = z1 + mz0 ⇔ y2 = y1 + my0
  • 20. 16 Viết đề tài giá sinh viên – ZALO:0973.287.149-TEAMLUANVAN.COM 3 3 4 5 3 9 5 3 9 0 5 3 25 Suy ra x2 = 1 + m. − 5 y2 = 2 + m.3 z2 = 3 + m.2 • m ∈ Z= thì y = 2 + m.3 < 0 • m ∈ Z+ thì x = 1 + m.(−5) < 0 V y bài toán chỉ có 1 nghi m nguyên dương. Bài toán 1.10. Tìm nghi m nguyên dương của h phương trình 1 x − 1 y + 1 z = 1 Lài giai. Ta có 3x + 4y − 5z = 0. 16 16 x − 4y + z = 16 Suy ra (1.21) 3 9 3 x + 4y − 5 25z = 0. 5 25 x − 9 z = 16 3 x + 4y − nên ta có x = 3, z = 5, y = 4 z = 0 5 Suy ra h (1.21) có m®t nghi m (x1, y1, z1) = (3, 4, 5) Tìm ”chìa khóa” (x0, y0, z0) thỏa mãn đieu ki n: 1 x 1 — 4 y0 1 + 5 z0 = 0 16 0 ⇔ 9 — 4y0 16 + 5 z0 = 0 25 suy ra 3x0 + 4y0 − 5z0 = 0 25 x = 9 z 3 x0 + 4y0 − 5 z0 = 0 3 x0 + 4y0 − 5 z0 = 0 Suy ra ”chìa khóa” (x0, y0, z0) = (27, 136, 125). ⇔ (1.21) 25 0 0 x
  • 21. 17 Viết đề tài giá sinh viên – ZALO:0973.287.149-TEAMLUANVAN.COM 3 4 5 15 3 9 5 25 15 120 125x − 27z = 632 9 Khi đó x2 = x1 + mx0 y2 = y1 + my0 z2 = z1 + mz0 hay x2 = 3 + m.27 y2 = 4 + m.136 z2 = 5 + m.125 m ∈ Z+ , suy ra x2 = 3 + 27m; y2 = 4 + 136m; z2 = 5 + 125m. V y bài toán có vô so nghi m nguyên dương. Ví dn 1.3. Tìm nghi m nguyên dương của h phương trình 1 x − 1 y + 1 z = 32 Lài giai. Ta có 3x + 4y − 5z = 8. 16 x − 4y + 16 z = 512 nên (1.22) ⇔ 3 5 25 3 x + 4y − 5 z = 15 120 15 25 x − 9 z = 632 hay 3 x + 4y − 5 z = 15 9 25 3 x + 4y − 5 z = Suy ra z = 9, x = 7, y = 8. 120 15 Suy ra h (1.22) có m®t nghi m (x1, y1, z1) = (7, 8, 9) . Tìm ”chìa khóa” (x0, y0, z0) thỏa mãn đieu ki n: 1 x 1 — 4 y0 1 + 5 z0 = 0 3x0 + 4y0 − 5z0 = 0 ”Chìa khóa” van là (x0, y0, z0) = (27, 136, 125) . Khi đó x2 = x1 + mx0 z2 = z1 + mz0. 3 y2 = y1 + my0 (1.22) 0
  • 22. 18 Viết đề tài giá sinh viên – ZALO:0973.287.149-TEAMLUANVAN.COM Suy ra x2 = 7 + m.27 y2 = 8 + m.136 z2 = 9 + m.125 Do m ∈ Z+ nên x2 = 7 + 27m; y2 = 8 + 136m; z2 = 9 + 125m. V y bài toán có vô so nghi m nguyên dương.
  • 23. 19 Viết đề tài giá sinh viên – ZALO:0973.287.149-TEAMLUANVAN.COM 2 2 Chương 2. Các phương pháp giải phương trình Diophant Trong chương này, ta xét m®t so phương pháp thông dụng giải phương trình Diophant tiep c n các đe thi của các kỳ Olympiad Toán hoc nhǎng năm gan đây (xem [3],[4]). 2.1 Phương pháp phân tích thành nhân tfi Xét phương trình f (x, y, . . . , z) = m. Giả sả ta có sự phân tích thành tích các nhân tả bat khả quy dạng f(x, y, . . . , z) = f1(x, y, . . . , z) . . . fs(x, y, . . . , z). Khi đó, ta phân tích so nguyên m và nh n được h phương trình tương áng. Bài toán 2.1. Tìm tat cả các nghi m nguyên(x, y) của phương trình x2 + 4 y2 + 4 − 4 (xy − 4) (x − y) = 9 + 16xy. Lài giai. Phương trình đã cho tương đương với x2 y2 + 4x2 + 4y2 + 16 − 4 (xy − 4) (x − y) − 16xy = 9. Suy ra x2 y2 − 8xy + 16 + 4 x2 − 2xy + y2 − 4 (xy − 4) (x − y) = 9 ⇔ (xy − 4) — 4 (xy − 4) (x − y) + 4 (x − y) = 9 2 ⇔ [xy − 4 − 2 (x − y)] = 9 ⇔ xy − 4 − 2x + 2y = ±3 ⇔ (x + 2) (y − 2) = ±3.
  • 24. 20 Viết đề tài giá sinh viên – ZALO:0973.287.149-TEAMLUANVAN.COM y − 2 = 3; y − 2 = 1; x + 2 = 1 x + 2 = 3 y − pq = q; 2 y− pq = q ; y − pq = pq; y − pq = p pq + 1, pq + p2 q2 , p + pq, pq + pq2 , q + pq, pq + p2 q , pq + q2 , pq + p2 , pq + p2 q2 , pq + 1 pq + p2 q, pq + q , . Như v y, ta thu được 8 h sau x + 2 = 1 x + 1 = −1 y − 2 = 3; x + 2 = 3 x + 2 = 3 y − 2 = −1; x + 2 = −1 y − 2 = −3; y − 2 = −3; x + 2 = −3 y − 2 = −1; − y − 2 = 1. Giải ra, ta thu được 8 nghi m là (−1, 5); (1, 3); (−3, −1); (−5, 1); (−1, −1); (−5, 3); (−3, 5); (1, 1). Bài toán 2.2. Cho p; q là 2 so nguyên to phân bi t. Tìm tat cả các nghi m nguyên dương (x, y) của phương trình 1 1 1 + = . x y pq Lài giai. Bieu dien phương trình đã cho thành phương trình (x − pq) (y − pq) = p2 q2 . Tà h thác 1 1 1 + = x y pq , suy ra 1 1 < x pq 1 1 và < . y pq Như v y, x > pq; y > pq. Ta nh n được các h sau đây x − pq = 1 y − pq = p2 q2 ; x − pq = p y − pq = pq2 ; x − pq = q y − pq = p2 q x − pq = p2 x − pq = p2 q Giải tàng h , ta nh n 9 nghi m x − pq = pq x − pq = q2 y − pq = p2 ; x − pq = pq2 x − pq = p2 q2 y − pq = 1. p2 + pq, q2 + pq , (2pq, 2pq) , pq + pq2 , p + pq ,
  • 25. 21 Viết đề tài giá sinh viên – ZALO:0973.287.149-TEAMLUANVAN.COM u + 2 = −1 u + 2 = −5 Giải các h phương trình trên và trở lại bien x, y, ta có 4 h sau 1 1 1 Bài toán 2.3. Cháng minh rang phương trình dạng + = với n = pα1 . . . pαk , x y n 1 k trong đó p1; p2; . . . ; pk là các so nguyên to, có (2α1 + 1) (2α2 + 1) . . . (αk + 1) nghi m nguyên dương. Lài giai. Th t v y, phương trình đã cho tương đương với (x − n) (y − n) = n2 2α1 2αk ⇔ (x − n) (y − n) = p1 . . . pk . M t khác, p2α1 . . . p2αk nên ta có 1 k (2α1 + 1) (2α2 + 1) . . . (αk + 1) ước. V y, ta thu được đieu can cháng minh. Bài toán 2.4 (Poland MO, xem [4]). Tìm tat cả các nghi m nguyên (x, y) của phương trình x2 (y − 1) + y2 (x − 1) = 1. Lài giai. Phương trình đã cho tương đương với x2 y − x2 + y2 x − y2 = 1 + Đ t u = x + y v = xy, ⇔ xy (x + y) − (x + y) phương trình trở thành 2xy = 1. Như v y, ta có 4 h sau uv − u2 + 2v = 1 ⇔ (uv + 2v) − u2 − 4 = 5 ⇔ (u + 2) (v − u + 2) = 5. u + 2 = 1 v − u + 2 = 5; v − u + 2 = −5; u + 2 = 5 v − u + 2 = 1; v − u + 2 = −1. x + y = −1 xy = 2; x + y = 3 xy = 2; x + y = −3 xy = −10; x + y = −7 xy = −10. Giải ra được 4 nghi m (1; 2) , (2; 1) , (2; −5) , (−5; 2) .
  • 26. 22 Viết đề tài giá sinh viên – ZALO:0973.287.149-TEAMLUANVAN.COM 3 3 3 3 3 3 3 3 3 3 3 3 3 3 3 3 3 3 Bài toán 2.5 (xem [6]). Xác định tat cả các b® ba so nguyên dương (x, y, z) thỏa mãn phương trình với so nguyên to p > 3. x3 + y3 + z3 − 3xyz = p, Lài giai. Vì và nên hay p = x3 + y3 + z3 − 3xyz = (x + y + z) x2 + y2 + z2 − xy − yz − zx (x + y + z > 1) x + y + z = p và x2 + y2 + z2 − xy − yz − zx = 1 (x − y)2 + (y − z)2 + (z − x)2 = 2. Không mat tính tőng quát, ta có the giả thiet x ≥ y ≥ z ≥ 1. Neu x > y > z thì x − y ≥ 1, y − z ≥ 1, x − z ≥ 2. Do đó (x − y)2 + (y − z)2 + (z − x)2 ≥ 6 > 2. Như v y, ta phải xét các trường hợp dưới đây Trường hợp 1. Khi x = y thì y = z + 1, x = z + 1 và x + y + z = p. V y nên z = p − 2 , x = y = p + 1 khi p = 3k + 2. 3 3 Trường hợp 2. Khi x > y = z thì y = z, x = y + 1 và x + y + z = p. V y nên y = z = p − 1 , x = p + 2 khi p = 3k + 1. 3 3 Tóm lại, neu p = 3k + 1 thì phương trình có ba nghi m p − 1 , p − 1 , p + 2 , p − 1 , p + 2 , p − 1 , p + 2 , p − 1 , p − 1 . Neu p = 3k + 2 thì phương trình có ba nghi m p − 2 , p + 1 , p + 1 , p + 1 , p − 2 , p + 1 , p + 1 , p + 1 , p − 2 .
  • 27. 23 Viết đề tài giá sinh viên – ZALO:0973.287.149-TEAMLUANVAN.COM 9x2 + 9y2 + 1 + 9xy + 3x − 3y = 823. Bài toán 2.6 (xem [7]). Xác định tat cả các so nguyên n đe phương trình sau đây có nghi m nguyên dương Lài giai. Ta có x3 + y3 + z3 − 3xyz = n. Suy ra x3 + y3 + z3 − 3xyz = (x + y + z) x2 + y2 + z2 − xy − yz − zx . x3 + y3 và + z3 — 3xyz = (x + y + z) (x − y)2 + (y − z)2 + (z − x)2 2 x3 + y3 + z3 − 3xyz = (x + y + z)3 − 3 (x + y + z) (xy + yz + zx) . Theo cách viet thá nhat, phương trình đã cho có nghi m nguyên dương khi n = 3k + 1 và n = 3k + 2. Khi đó (k + 1, k, k) và (k + 1, k + 1, k), với k ≥ 1, là nghi m dương tương áng. Neu n chia het cho 3 thì tà cách bieu dien thá hai, x + y + z chia het cho 3 và như v y n = x3 + y3 + z3 − 3xyz chia het cho 9. Ngược lại, neu n = 9k với k ≥ 2, thì phương trình nh n nghi m (k − 1, k, k + 1) . Với n = 0, ta có nghi m nguyên dương x = y = z ∈ N∗ . Với n = 9 phương trình không có nghi m nguyên dương (x, y, z). Bài toán 2.7 (Russia MO, xem [4]). Tìm tat cả các nghi m nguyên dương của phương trình x3 − y3 = xy + 61. Lài giai. Nhân cả hai ve của phương trình với 27, ta thu được phương trình tương đương với (3x)3 + (−3y)3 + (−1)3 − 3(3x)(−3y)(−1) = 1642. Ta có a3 + b3 + c3 − 3abc = (a + b + c) a2 + b2 + c2 − ab − bc − ca . V y nên phương trình đã cho tương đương với (3x − 3y − 1) 9x2 + 9y2 + 1 + 9xy + 3x − 3y = 2.823. Vì 9x2 + 9y2 + 1 + 9xy + 3x − 3y > (3x − 3y − 1) và 823 là so nguyên to nên ta có 3x − 3y − 1 = 2 Giải h phương trình trên ta thu được nghi m là (6, 5).
  • 28. 24 Viết đề tài giá sinh viên – ZALO:0973.287.149-TEAMLUANVAN.COM 2.2 Phương pháp đong dư Bang cách xét so dư hai ve của phương trình, phương pháp đong dư thường dùng đe cháng minh phương trình không có nghi m nguyên ho c dùng đe hạn che các khả năng của bien. Tà đó, de dàng tìm được nghi m nguyên của phương trình. Dưới đây là m®t vài ví dụ minh hoa cho phương pháp này. Bài toán 2.8. Cháng minh rang phương trình (x + 1)2 + (x + 2)2 + · · · + (x + 2001)2 = y2 không có nghi m nguyên. Lài giai. Đ t x = z − 1001, phương trình đã cho trở thành (z − 1000)2 + · · · + (z − 1)2 + z2 + (z + 1)2 + · · · + (z + 1000)2 = y2 hay 2001z2 + 2 12 + 22 + · · · + 10002 = y2 . Suy ra, phương trình tương đương với 2000z2 + 2 1000.1001.2001 6 = y2 , 2001z2 + 1000.1001.667 = y2 . Ta thay ve trái ≡ 2 (mod 3) nên không the là so chính phuơng, trong khi ve phải là m®t so chính phương. V y nên phương trình đã cho không có nghi m nguyên. Bài toán 2.9 (Russia MO, xem [4]). Xác định tat cả các c p so nguyên to (p, q) thỏa mãn phương trình p3 − q5 = (p + q)2 . Lài giai. De dàng kiem tra p > q. Neu q = 3 thì p = 7 và ta có c p so nguyên to (7, 3). Neu q > 3, và do p, q là so nguyên to nên p ≡ 1 hay 2 (mod 3), và q ≡ 1 hay 2 (mod 3). Neu p ≡ q (mod 3) thì ve trái chia het cho 3, ve phải không chia het cho 3. Neu p /≡ q (mod 3), thì ve phải chia het cho 3, ve trái không chia het cho 3. Trong cả hai trường hợp trên, phương trình đeu vô nghi m. V y nên phương trình đã cho có nghi m duy nhat (p, q) = (7, 3).
  • 29. 25 Viết đề tài giá sinh viên – ZALO:0973.287.149-TEAMLUANVAN.COM Bài toán 2.10 (Balkan MO, xem [4]). Cháng minh rang phương trình x5 − y2 = 4 không có nghi m nguyên. Lài giai. Xét vành Z11. De dàng kiem tra (x5 )2 ≡ x10 ≡ 0 hay 1 (mod 11) với moi x ∈ Z. Do v y x5 ≡ 0 ho c ±1 (mod 11). Tà đây suy ra x5 −4 ≡ 6 ho c 7 ho c 8 (mod 11). Vì th ng dư b c hai modulo 11 chỉ có the là 0, 1, 3, 4, 5, 9 nên phương trình đã cho không the có nghi m nguyên. Bài toán 2.11. Tìm tat cả các nghi m nguyên của phương trình x3 + y3 = z6 + 3. Lài giai. Neu phương trình có nghi m trong Z thì nó cũng có nghi m trong Z7. Khi đó ton tại x, y, z ∈ Z7 đe x3 + y3 = z6 + 3. Trong Z7 có 0 3 = 0, 1 3 = 1, 2 3 = 1, 3 3 = −1, 4 3 = −1, 5 3 = −1, 6 3 = −1. V y nên x3 hay y3 chỉ có the là 0 ho c 1 ho c −1. Bang kiem tra trực tiep, ta thay x3 + y3 chỉ có the là 0, 1, 2, −1, −2. Nhưng z6 + 3 chỉ có the là 0 + 3 = 3 ho c 1 + 3 = 4. Đieu này cháng tỏ phương trình vô nghi m. 2.3 Phương pháp đánh giá Phương pháp đánh giá là dùng các bat đȁng thác phù hợp đe hạn che lại các khoảng đang xét. Tà đó, ta tìm được nghi m nguyên của phương trình m®t cách đơn giản hơn. Bài toán 2.12. Giải phương trình x3 + y3 = (x + y)2 với x, y ∈ Z. Lài giai. Xét x + y = 0. Khi đó x = k, y = −k với k ∈ Z đeu là nghi m. Xét x + y 0. De dàng ta có hay x2 − xy + y2 = x + y (x − y)2 + (x − 1)2 + (y − 1)2 = 2. Tà đây suy ra |x − 1|, |y − 1| ≤ 1. Như v y, x, y ∈ {0, 1, 2}. Kiem tra (0, 1), (1, 0), (1, 2), (2, 1), (2, 2) và (k, −k) với k ∈ Z là nghi m phương trình đã cho.
  • 30. 26 Viết đề tài giá sinh viên – ZALO:0973.287.149-TEAMLUANVAN.COM − ≥ Bài toán 2.13 (Russia MO, xem [4]). Giải phương trình x, y, z ∈ N∗ . 1 1 1 3 + + = , x y z 5 3 3 Lài giai. Không hạn che có the giả thiet 2 ≤ x ≤ y ≤ z. Khi đó và suy x 5 ra x ∈ {2, 3, 4, 5}. 1 1 Neu x = 2 thì + = y z 1 2 10 . De thay 11 ≤ y ≤ z. Khi đó, y ≥ 100 1 suy ra 10 y ∈ {11, 12, . . . , 20} và z = 10 + y 10 . Vì z nguyên dương nên (y − 10) | 100. Ta thu được các nghi m (2, 11, 110), (2, 12, 60), (2, 14, 35), (2, 15, 30), (2, 20, 20). Neu x = 3 thì 1 1 4 + = . y z 15 Làm tương tự như trường hợp trên ta có y ∈ {3, 4, 5, 6, 7}. De dàng kiem tra đe được nghi m (3, 4, 60), (3, 5, 15), (3, 6, 10). 1 1 7 Neu x = 4 thì y + z = 20 . Tương tự ta có y ∈ {4, 5}. De dàng kiem tra đe được nghi m (4, 4, 10). Neu x = 5 thì 1 1 2 + = y z 5 . Tương tự ta có nghi m (5, 5, 5). Bài toán 2.14. Giải phương trình nghi m nguyên dương x2 + y2 + z2 + 2xy + 2x(z − 1) + 2y(z + 1) = w2 . Lài giai. Ta có (x + y + z + 1)2 = x2 + y2 + z2 + 2xy + 2x(z + 1) + 2y(z + 1) + 2z + 1. (x + y + z − 1)2 = x2 + y2 + z2 + 2xy + 2x(z − 1) + 2y(z − 1) − 2z + 1. Suy ra Do đó (x + y + z − 1)2 < w2 < (x + y + z + 1)2 . x2 + y2 + z2 + 2xy + 2x(z − 1) + 2y(z + 1) = (x + y + z)2 . Suy ra x = y. Đ t x = y = m; z = n thì w = 2m + n, trong đó m; n là các so nguyên dương. V y nghi m phương trình đã cho là các b® so (m; m; n; 2m +n), với m; n là các so nguyên dương.
  • 31. 27 Viết đề tài giá sinh viên – ZALO:0973.287.149-TEAMLUANVAN.COM z 3 1 x y z x y x y 3 y 3 x x y 2 đ t y = tx, với t ∈ Z. V y x3 = x2 + 2t2 x2 hay x = 2t2 + 1, u = t Lài giai. Với z = −y ta có x3 = x2 + 2y2 . Khi x = 0 thì y = 0. Khi x /= 0 ta 2t2 + 1 . Bài toán 2.15. Giải phương trình nghi m nguyên dương 1 + 1 1 + 1 1 + 1 = 2. Lài giai. Không hạn che có the giả thiet x ≥ y ≥ z ≥ 1. Khi đó, ta có 1 + ≥ 2 và suy ra z ≤ 3. Xét các trường hợp sau đây: Trường hợp z = 1. Khi đó 1 + 1 1 + 1 = 1 vô nghi m. Trường hợp z = 2. De dàng chỉ ra phương trình 1 + 1 1 + 1 = 4 và suy ra 1 + 1 2 ≥ 4 . De dàng suy ra y < 7. Vì 1 + 1 > 1 nên y > 3. Ta de dàng nh n được nghi m (7, 6, 2), (9, 5, 2), (15, 4, 2). Trường hợp z = 3. De dàng có 1 + 1 1 + 1 = 3 và suy ra 3 = z ≤ y < 5. De dàng kiem tra đe được nghi m (8, 3, 3), (5, 4, 3). Tóm lại, ta nh n được các nghi m (7, 6, 2), (9, 5, 2), (15, 4, 2), (8, 3, 3) và (5, 4, 3). 2.4 Phương pháp tham so hóa M®t so phương trình nghi m nguyên f(x1; x2; . . . ; xn) = 0 có vô so nghi m và ta không the li t kê được het tat cả các nghi m đó. Khi đó, ta sě tìm nghi m này dưới dạng tham so như sau: x1 = g1(k1, k2, . . . , kl), x2 = g2(k1, k2, . . . , kl), . . . , xn = gn(k1, k2, . . . , kl), trong đó g1, g2, . . . , gn là hàm l bien, k1, . . . , kl ∈ Z. Điem mạnh của phương pháp này là có the dùng đe cháng minh phương trình nghi m nguyên có vô so nghi m. Sau đây ta sě xét m®t so bài toán minh hoa cho phương pháp này. Bài toán 2.16. Cháng minh rang, có vô hạn b® ba (x, y, z) ∈ Z3 thỏa mãn phương trình x3 + y3 + z3 = x2 + y2 + z2 . Với t ∈ Z, ta nh n được vô so nghi m 2t2 + 1, t 2t2 + 1 , −t 2t2 + 1 .
  • 32. 28 Viết đề tài giá sinh viên – ZALO:0973.287.149-TEAMLUANVAN.COM ∈ ≥ Bài toán 2.17. Cháng minh rang, phương trình x2 = y3 + z5 có vô so nghi m nguyên dương (x, y, z). Lài giai. Xét Khi đó y = t7 (t + 1)5 , z = t4 (t + 1)3 . x2 = t21 (t + 1)15 + t20 (t + 1)15 = t20 (t + 1)16 . Giải ra, ta thu được x = t10 (t + 1)8 . Do v y, phương trình đã cho có nhieu vô so nghi m nguyên dương x = t10 (t + 1)8 , y = t7 (t + 1)5 , z = t4 (t + 1)3 . Bài toán 2.18. Cháng minh rang, phương trình x2 + y2 = 13z có vô so nghi m nguyên dương (x, y, z). Lài giai. Vì 22 + 32 = 13 nên khi chon x = 2.13n , y = 3.13n , ta có 13z = 132n+1 . Suy ra z = 2n + 1. Do v y, phương trình có nhieu vô so nghi m nguyên dương x = 2.13n , y = 3.13n , z = 2n + 1, n ∈ N. Bài toán 2.19. Cháng minh rang, phương trình xn + yn = zn−1 có vô so nghi m nguyên dương (x, y, z) khi n ≥ 3. Lài giai. Đ t x = ty, t N. Khi đó yn (tn + 1) = zn−1 . Chon y = (tn + 1)n−2 ta có z = (tn + 1)n−1 và x = t (tn + 1)n−2 . Do v y, phương trình có nhieu vô so nghi m nguyên dương x = t (tn + 1)n−2 , 13n , y = (tn + 1)n−2 , z = (tn + 1)n−1 , t ∈ N∗. Bài toán 2.20. Cháng minh rang, phương trình 2x + 1 = xy có vô so nghi m nguyên dương (x, y). Lài giai. Trước tiên ta cháng minh 23n + 1 chia het cho 3n với moi n 0. Với n = 0, ket lu n là hien nhiên. Giả sả 23n + 1 chia het cho 3n . Với n + 1 ta bieu dien 23n+1 + 1 = 23n + 1 22.3n 23n + 1 . −
  • 33. 29 Viết đề tài giá sinh viên – ZALO:0973.287.149-TEAMLUANVAN.COM Xét thàa so 22.3n −23n +1 = 23n +1 2 −3.23n chia het cho 3. Do v y 23n+1 +1 chia Thàa so 23n + 1 chia het cho 3n theo giả thiet quy nạp. het cho 3n+1 . Phương trình đã cho có vô so nghi m nguyên dương với moi so nguyên n ≥ 0. 3n , 23n + 1 3n Bài toán 2.21. Cháng minh rang, với so nguyên n ≥ 2 phương trình xn + yn = zn+1 có vô so nghi m nguyên dương (x, y, z). Lài giai. Phương trình đã cho có vô so nghi m nguyên dương dạng (tn , t(tn + 1),tn + 1) với moi so nguyên t ≥ 1. Bài toán 2.22. Tìm các b® ba so nguyên dương (x, y, z) thỏa mãn 1 1 1 + = . x y z xy Lài giai. Ta có z = x + y . Giả sả d = (x, y). Đ t x = dm, y = dn. Khi đó dmn (m, n) = 1 và (mn, m + n) = 1. Do v y, z = xy m + n và suy ra m + n là ước của d. Đ t t = . Ta có x = tm(m + n), y = tn(m + n), z = tmn với m, n, t ∈ N∗ z và (m, n) = 1. Nh n xét 2.3. 1. Neu a, b, c là các so đôi m®t nguyên to cùng nhau và thỏa mãn đieu ki n 1 1 1 + = a b c thì a + b là m®t so chính phương. 1 1 1 2. Neu a, b, c là các so nguyên dương và thỏa mãn đieu ki n + a b a2 + b2 + c2 là m®t so chính phương. Th t v y, = thì c a2 + b2 + c2 = k2 h m2 (m + n)2 + n2 (m + n)2 + m2 n2 i = k2 h (m + n)4 − 2mn (m + n)2 + m2 n2 i = k2 h (m + n)2 − mn i2 Bài toán 2.23. Xác định tat cả các c p so nguyên (x, y) thỏa mãn đieu ki n x3 + y3 + 21xy = 343. Lài giai. Ta có x3 + y3 + (−7)3 − 3(−7)xy = 0.
  • 34. 30 Viết đề tài giá sinh viên – ZALO:0973.287.149-TEAMLUANVAN.COM Suy ra (x + y − 7) h (x − y)2 + (y + 7)2 + (x + 7)2 i = 0. Do v y, ta nh n được nghi m (−7, −7) và (a, −7 − a) với a ∈ Z. Bài toán 2.24. Xác định tat cả các c p so nguyên (x, y) thỏa mãn đieu ki n x3 + y3 + 6xy = 8. Lài giai. Ta có Suy ra x3 + y3 + (−2)3 − 3(−2)xy = 0. (x + y − 2) x2 + y2 + 22 − xy + 2x + 2y = 0. Do v y, ta nh n được nghi m (−2, −2) và (a, 2 − a) với a ∈ Z. 2.5 Phương pháp quy nạp toán hoc Ta xét m®t so quy tac sả dụng quy nạp toán hoc. - Nguyên lý quy nạp thá nhat. M nh đe P(n) là đúng với moi so tự nhiên n ≥ α neu (1) Bư c cơ s : M nh đe P(α) đúng. (2) Bư c quy nạp: Neu P(n) đúng thì P(n + 1) cũng đúng, trong đó n ≥ α. Như v y, sau khi kiem tra bước cơ sở và cháng minh tính đúng của m nh đe P (n + 1) dưới giả thiet m nh đe P (n) đúng, ta ket lu n P (n) đúng cho moi so tự nhiên n ≥ α. - Nguyên lý quy nạp thá hai. M nh đe P(n) là đúng với moi so tự nhiên n ≥ α neu (1) Bư c cơ s : M nh đe P(α) đúng. (2) Bư c quy nạp: M nh đe P (n + 1) là đúng khi các m nh đe P (α), P (α + 1), . . . , P(n) đeu đúng, trong đó n ≥ α, n ∈ N. Bài toán 2.25 (Belarussia MO, xem [4]). Cháng minh rang với moi so nguyên n ≥ 3, phương trình sau luôn có nghi m (x; y) trong đó x, y là các so nguyên dương lẻ 7x2 + y2 = 2n .
  • 35. 31 Viết đề tài giá sinh viên – ZALO:0973.287.149-TEAMLUANVAN.COM + Lài giai. Ta sě cháng minh, với moi n ≥ 3, ton tại so nguyên dương lẻ xn; yn thỏa mãn đieu ki n 7x2 + y2 = 2n . n n Với n = 3, de dàng tìm được nghi m x3 = y3 = 1. Giả sả ton tại so nguyên dương lẻ xn; yn thỏa mãn đieu ki n 7x2 + y2 = 2n . n n Ta phải cháng minh ton tại so nguyên dương lẻ xn+1; yn+1 thỏa mãn đieu ki n Th t v y, 2 n+1 2 n+1 = 2n+1 . xn ± yn 2 7xn ± yn 2 = 2(7x2 + y2 ) = 2n+1 . 2 2 n n Rõ ràng m®t trong hai so xn + yn 2 và |xn − yn| 2 là so lẻ vì xn + yn + |xn − yn| = x ho c 2 2 n xn + yn + |xn − yn| = y . 2 2 n M t khác, xn; yn là các so lẻ. Neu xn + yn 2 là lẻ thì xn − yn 2 chȁn. Khi đó là so lẻ. Do đó ta chon 7xn − yn 2 = 3xn + xn − yn 2 xn+1 = xn + yn ; y 2 n+1 = 7xn − yn . 2 Neu xn − yn 2 là lẻ thì xn + yn 2 chȁn. Khi đó là so lẻ. Do đó ta chon 7xn + yn 2 = 3xn + xn + yn 2 |xn − yn| ; y 2 n+1 = 7xn + yn . 2 7x + y 7
  • 36. 32 Viết đề tài giá sinh viên – ZALO:0973.287.149-TEAMLUANVAN.COM k+2 k+2 k+2 k k k k Bài toán 2.26 (xem [4]). Cháng minh rang với moi so nguyên dương n, phương trình sau luôn có nghi m (x; y) trong đó x, y là các so nguyên dương x2 + y2 + z2 = 59n . Lài giai. Ta sě sả dụng phương pháp quy nạp với b c s = 2; n0 = 1. De thay 12 + 32 + 72 = 59 và 142 + 392 + 422 = 592 nên (x1; y1; z1) = (1; 3; 7) và (x2; y2; z2) = (14; 39; 42) là 2 nghi m của phương trình. Giả sả với moi n = k, (k ≥ 1) phương trình có nghi m (xk; yk; zk). Tác là x2 + y2 + z2 = 59k . k k k Đ t xk+2 = 59xn, yk+2 = 59yn, zn+2 = 59zn, ∀k ≥ 1. Ta có x2 + y2 + z2 = 592 x2 + y2 + z2 = 59k+2 . Do đó (xk+2; yk+2; zk+2) xác định như trên là nghi m của phương trình. Bài toán 2.27. Cháng minh rang với moi so nguyên n ≥ 3, phương trình sau luôn có nghi m (x1; x2; . . . .; xn) là các so nguyên dương đôi m®t khác nhau 1 1 + x1 x2 1 1 1 + · · · + x 1 = 1. Lài giai. Với n = 3, ta có + + 2 3 6 = 1. Giả sả phương trình có nghi m đen n = k, (k ≥ 3), tác là 1 1 + x1 x2 1 + · · · + x = 1, với x1; x2; . . . .; xk là các so nguyên dương đôi m®t khác nhau. Suy ra Do đó 1 2x1 1 1 + 2x2 1 1 1 + · · · + 2x = 2 . 1 1 + 2 2x1 + 2x2 + · · · + 2x = 1, với 2; 2x1; 2x2; . . . .; 2xk là các so nguyên dương đôi m®t khác nhau. V y nên (2; 2x1; 2x2; . . . .; 2xk) là nghi m can tìm. n k k
  • 37. 33 Viết đề tài giá sinh viên – ZALO:0973.287.149-TEAMLUANVAN.COM 2.6 Phương pháp xuong thang Fermat đã dùng phương pháp xuong thang đe cháng minh phương trình x4 + y4 = z4 không có nghi m nguyên dương. Cơ sở của phương pháp xuong thang là tính sap thá tự tot của N (và Nk , tích Đe các của k phiên bản N): M®t t p con khác rong bat kỳ của N đeu có phan tả nhỏ nhat. Đe cháng minh m®t phương trình là vô nghi m, ta giả sả ngược lại rang t p R các nghi m nguyên (tự nhiên, nguyên dương) của phương trình khác rong. Ta đưa ra m®t thá tự tot trên R và giả sả α0 là nghi m nhỏ nhat (theo thá tự nêu trên). Neu bang cách nào đó ta xây dựng được nghi m a1 nhỏ hơn α0 thì chúng ta sě đi đen mâu thuan. Mâu thuan này cháng tỏ đieu giả sả là sai và như v y phương trình đã cho vô nghi m. Bài toán 2.28. Cháng minh rang phương trình x4 + y4 = z2 không có nghi m nguyên dương. Lài giai. Giả sả rang phương trình đã cho có nghi m nguyên dương. Giả sả (x, y) = d, tác là x = da, y = db, trong đó (a, b) = 1. Khi đó a4 + b4 = (z/d2 )2 . Giả sả z = d2 c, trong đó c ∈ Q, khi đó a4 + b4 = c2 (2.1) Vì c ∈ Q, c2 ∈ N nên c ∈ N+ . Trong tat cả các nghi m của phương trình (2.1), chon nghi m có c nhỏ nhat. Ta có (a2 )2 + (b2 )2 = c2 trong đó (a, b) = 1, suy ra (a2 , b2 ) = 1, tác là (a2 , b2 , c) là b® ba Pythagore nguyên thủy. Ton tại các so nguyên dương m, n sao cho a2 = m2 − n2 , b2 = 2mn, c = m2 + n2 , trong đó m, n khác tính chȁn lẻ, m > n và (m, n) = 1, nghĩa là a2 = m2 − n2 lẻ. Giả sả m chȁn, n lẻ. Khi đó n2 , a2 chia 4 dư 1, nghĩa là m2 = n2 + a2 chia 4 dư 2 mâu thuan. V y m lẻ, n chȁn, ngòai ra (a, n, m) l p thành b® Pythagore nguyên thủy, do đó ton tại p, q ∈ N+ sao cho a = p2 − q2 , n = 2pq, m = p2 + q2 , trong đó p, q khác tính chȁn lẻ, p > q và (p, q) = 1, ngòai ra b2 = 2mn, nghĩa là b2 = 4pq(p2 + q2 ), suy ra b = 2h, h ∈ N+ , khi đó h2 = pq(p2 + q2 ). (2.2) Giả sả rang ton tại so nguyên to r chia het pq, p2 + q2 . Vì r chia het pq nên không mat tőng quát, có the giả sả r chia het p, khi đó r chia het (p2 + q2 ) − p2 = q2 , suy ra r chia het q, mâu thuan vì (p, q) = 1. V y (pq, p2 + q2 ) = 1, như the, tà (2.2), theo định lý ??, ta có pq = s2 , p2 + q2 = t2 với s, t ∈ N+ . Vì pq = s2 , (p, q) = 1 nên p = u2 , q = v2 với u, v ∈ N+ , nghĩa là (u2 )2 + (v2 )2 = t2 hay u4 + v4 = t2 , trong đó c = m2 + n2 > m = p2 + q2 = t2 > t, mâu thuan với cách chon c. Như v y đieu giả sả ban đau là sai và ta có đieu phải cháng minh.
  • 38. 34 Viết đề tài giá sinh viên – ZALO:0973.287.149-TEAMLUANVAN.COM Bài toán 2.29. Tìm các nghi m nguyên của phương trình x3 − 3y3 = 9z3 . Lài giai. Hien nhiên 3|x. Đ t x = 3x1, với x1 nguyên. Thay vào phương trình ban đau roi chia cả hai ve cho 3, ta thu được 9x1 3 − y3 = 3z3 . Suy ra 3|y. Đ t y = 3y1, với y1 nguyên. Thay vào phương trình trên roi chia cả hai ve cho 3, ta được 3x1 3 + 9y1 3 = z3 . Suy ra 3|z. Đ t z = 3z1, với z1 nguyên. Thay vào phương trình trên roi chia cả hai ve cho 3 ta được x1 3 − 3y1 3 = 9z1 3 . Như v y neu (x, y, z) là nghi m của phương trình đã cho thì (x1, y1, z1) cũng là nghi m, trong đó, x = 3x1, y = 3y1, z = 3z1. L p lu n tương tự như trên thì (x2, y2, z2) cũng là nghi m, trong đó x1 = 3x2, y1 = 3y2, z1 = 3z2. Cá tiep tục như v y dan đen x, y, z đeu chia het cho 3k , với k là so tự nhiên tùy ý. Đieu này chỉ xảy ra khi x = y = z = 0. Đó là nghi m nguyên duy nhat của phương trình đã cho. Nh n xét 2.4. Trong ví dụ trên, neu yêu cau tìm các nghi m nguyên dương của phương trình x3 − 3y3 = 9z3 , ta có the dùng nguyên tac cực hạn đe cháng minh phương trình không có nghi m nguyên dương như sau. Giả sả (x0, y0, z0) là nghi m nguyên dương của phương trình, trong đó x0 là giá trị nguyên dương nhỏ nhat trong các giá trị. M t khác, x có the nh n được. L p lu n như trong các giải trên, ta thu được (x1, y1, z1) cũng là nghi m nguyên dương của phương trình. M t khác, x0 = 2x1, tác là x0 > x1. Đieu này trái với giả thiet x0 là so nguyên dương nhỏ nhat trong các giá trị nh n được của x. V y nên phương trình đã cho không có nghi m nguyên dương. Bài toán 2.30 (Korea MO, xem [4]). Tìm các nghi m nguyên của phương trình x2 + y2 + z2 = 2xyz. Lài giai. Do ve phải chia het cho 2 nên x2 + y2 + z2 chia het cho 2. Có 2 khả năng xảy ra.
  • 39. 35 Viết đề tài giá sinh viên – ZALO:0973.287.149-TEAMLUANVAN.COM Neu trong ba so x, y, z có m®t so chȁn, hai so lẻ. Giả sả x chȁn, y, z lẻ thì x2 + y2 + z2 chia cho 4 dư 2 còn 2xyz chia het cho 4. nên khă năng này loại. V y ba so x, y, z phải cùng chȁn. Đ t x = 2x1, y = 2y1, z = 2z1, với x1, y1, z1 là các so nguyên. Thay vào phương trình ban đau roi chia cả hai ve cho 4 ta có x1 2 + y1 2 + z1 2 = 4x1y1z1. L p lu n tuơng tự, ta thu được x1, y1, z1 cũng là các so chȁn. Cá tiep tục như v y dan đen x, y, z đeu chia het cho 2k , với k là so tự nhiên tùy ý. Đieu này chỉ xảy ra khi x = y = z = 0. Đó là nghi m nguyên duy nhat của phương trình đã cho. Bài toán 2.31. Tìm các nghi m nguyên của phương trình x4 + y4 + z4 = 9u4 . Lài giai. Neu u = 0 thì x = y = z = 0. Ta sě cháng minh không còn nghi m nào khác nghi m (0, 0, 0, 0). Giả sả x, y, z là các so nguyên thỏa mãn phương trình và u 0. Neu u không chia het cho 5 thì theo Định lý Fermat nhỏ ta có u4 ≡ 1 (mod 5) nên 9u4 ≡ 4 (mod 5) nhưng x4 , y4 , z4 ≡ 0 ho c 1 (mod 5) nên khả năng này không xảy ra. V y u phải chia het cho 5. Đ t u = 5u1, với u1 nguyên, ta có x4 + y4 + z4 ≡ 0 (mod 5). Theo Định lý Fermat nhỏ thì x, y, z chia het cho 5. Đ t x = 5x1, y = 5y1, z = 5z1, với x1, y1, z1 là các so nguyên. Thay vào phương trình ban đau roi chia cả hai ve cho 54 , ta có x1 4 + y1 4 + z1 4 = 9u1 4 . Như v y neu (x, y, z) là nghi m của phương trình đã cho thì (x1, y1, z1) cũng là nghi m. Trong đó, x = 5x1, y = 5y1, z = 5z1, u = 5u1. L p lu n tương tự như trên thì (x2, y2, z2, u2) cũng là nghi m, trong đó x1 = 5x2, y1 = 5y2, z1 = 5z2, u1 = 5u2. Cá tiep tục như v y dan đen x, y, z, u đeu chia het cho 5k , với k là so tự nhiên tùy ý. Đieu này chỉ xảy ra khi x = y = z = u = 0. Đó là nghi m nguyên duy nhat của phương trình đã cho. Bài toán 2.32. Tìm các nghi m nguyên dương của phương trình x2 − y2 = 2xyz.
  • 40. 36 Viết đề tài giá sinh viên – ZALO:0973.287.149-TEAMLUANVAN.COM xy zx Lài giai. Giả sả x0, y0, z0 là các so nguyên dương thỏa mãn phương trình, trong đó x0 là giá trị nguyên dương nhỏ nhat trong các giá trị M t khác, x có the nh n được. Đ t d = x0y0. Neu d = 1 thì x0 = y0 = 1, z0 = 0, (loại). V y d > 1. Goi p là ước nguyên to của d. Ta có (x0 + y0)(x0 − y0) = x0 2 − y0 2 = 2x0y0z0 ≡ 0 (mod p)). Suy ra, x0 ≡ y0 (mod p) ho c x0 ≡ −y0 (mod p). Vì x0y0 chia het cho p nên x0 ≡ y0 ≡ 0 (mod p). Đ t x0 = px1, y0 = py1, với x1, y1 là các so nguyên dương. Thay vào phương trình ban đau roi chia cả hai ve cho p2 , ta có x1 2 − y1 2 = 2x1y1z0. Suy ra, (x1, y1, z0) cũng là nghi m nguyên dương của phương trình. M t khác, x0 = px1, tác là x0 > x1. Đieu này trái với giả thiet x0 là so nguyên dương nhỏ nhat trong các giá trị nh n được của x. V y nên phương trình đã cho không có nghi m nguyên dương. Nh n xét 2.5. Đe sả dụng phương pháp xuong thang, người ta can đen nhǎng điem cực biên của t p hợp so đã cho. Bài toán 2.33. Tìm tat cả các so nguyên to p, q, r thỏa mãn phương trình pq + qp = r. Lài giai. Nh n xét rang trong 3 so nguyên to p, q, r toi thieu phải có m®t so nguyên to chȁn. So đó phải bang 2. Vì p, q là hai so nguyên to nên r /= 2. Không mat tính tőng quát, có the giả thiet q = 2. V y p2 +2p = r. Vì r là so nguyên to nên p phải là só lẻ. Neu p = / 3 thì p2 + 2p ≡ (±1)2 + (−1)p (mod 3) ≡ 1 − 1 (mod 3). V y r chia het cho 3, nhưng p2 + 2p = 3 chỉ thỏa mãn cho p = 1 : mâu thuan. Tà đây suy ra p = 3, r = 17. Bài toán 2.34. Tìm nghi m nguyên dương của phương trình x + y + z = xyz (2.3) Lài giai. Vì vai trò của x, y, z như nhau nên ta có the giả sả 1 ≤ x ≤ y ≤ z. Tà 1 1 (2.3) suy ra 1 = + 1 3 + ≤ , suy ra x2 ≤ 3 suy ra x = 1. Thay x = 1 vào (2.3), ta được 1 + y + z = yz hay (y − 1)(z − 1) = 2 = 1.2. yz x2
  • 41. 37 Viết đề tài giá sinh viên – ZALO:0973.287.149-TEAMLUANVAN.COM . z x y x x y z x z Do đó y = 2, z = 3 (vì y − 1 ≤ z − 1). V y nghi m của phương trình (2.3) là (1; 2;3) và các hoán vị của nó. Nh n xét 2.6. đây vì chỉ có hǎu hạn so nguyên dương và chúng có vai trò như nhau nên ta có the giả sả x là so nguyên dương bé nhat. Bài toán 2.35. Tìm tat cả các so tự nhiên n sao cho ton tại các so tự nhiên (x, y, z) có tính chat (x + y + z)2 chia het cho nxyz. (x + y + z)2 Lài giai. Đ t F (x; y; z) = . xyz Giả sả n là so nguyên dương nào đó sao cho ton tại b® so nguyên dương x, y, z đe n = F (x; y; z). Ta giả sả (x, y, z) là m®t b® so như v y, đong thời và z đạt cực tieu trong tat cả các b® thỏa mãn đieu ki n bài ra. Ta có n(xyz) = (x + y + z)2 = (x + y)2 + 2z(x + y) + z2 (2.4) Tà đó, suy ra z (x + y)2 . M t khác neu (x, y, z) thỏa mãn đieu ki n đòi hỏi thì F (x + y)2 ! x, y; (x + y)2 z = n, do b® so x, y; z thỏa mãn (2.4). (x + y)2 Như v y neu z > x + y thì z z. Tà đó, suy ra x + y ≥ z. Ta có > z, trái với giả thiet ve tính cực tieu của x y z n = + + yz xz xy 2 2 2 + + + x y z ≤ 1 + 1 + 1 + 1 + 2 + 2 + 2 7 3 ≤ + . V y n lớn nhat khi z = 1, suy ra x = y = z = 1, tác là n 7. 7 3 Giả sả ngược lại, 7 ≤ x + z . 7 3 7 3 Khi đó x ≥ 2, suy ra x + z ≤ + = 5, vô lý. 2 3 Tính toán trực tiep, ta được F (9; 9; 9) = 1, F (4, 4, 8) = 2, F (3; 3; 3) = 3, F(2; 2; 2) = 4, !
  • 42. 38 Viết đề tài giá sinh viên – ZALO:0973.287.149-TEAMLUANVAN.COM F(1; 4; 5) = 5, F(1; 2; 6) = 6, F(1; 1; 2) = 8, F(1; 1; 1) = 9. V y các giá trị n can tìm là 1; 2; 3; 4; 5; 6; 8; 9. Nh n xét 2.7. - Ta chỉ can tìm các so nguyên dương n đe dau đȁng thác xảy ra m c dù yêu cau bài toán là (x + y + z)2 chia het cho nxyz, vì neu (x + y + z)2 = k(nxyz) thì ta thay kn bởi n′. - Vì trong t p hợp các so nguyên dương bao giờ cũng ton tại so nguyên dương bé nhat nên ta có the giả sả z nhỏ nhat. Bài toán 2.36. Cho n là so tự nhiên, n ≥ 3. Tìm so tự nhiên k nhỏ nhat có tính chat sau: Với moi b® n so tự nhiên d1, d2, . . . , dn nguyên to cùng nhau sao cho tőng d1 + d2 + · · · + dn chia het cho moi di với 1 ≤ i ≤ n, ta có (d1 + d2 + · · · + dn)k chia het cho tích d1d2 . . . dn. Lài giai. Giả sả p là so nguyên to, p là ước của tích d1d2 . . . dn. Goi s là so lớn nhat sao cho ton tại j, trong đó 1 ≤ j ≤ n đe dj chia het cho ps . Vì p1 + p2 + · · · + pn chia het cho dj nên d1 + d2 + · · · + dn chia het cho ps , tà đó suy ra d1 + d2 + · · · + dn)n−1 chia het cho ps(n−2) . Vì d1, d2, . . . , dn không có ước chung khác 1 nên phải ton tại dl nào đó không chia het cho p. Như v y lũy thàa cao nhat của p chia het tích d1d2 . . . dn không vượt quá s(n − 2). Vì p được chon là ước nguyên to tùy ý của tích nên ta có (d1 + d2 + · · · + dn)n−2 .d1d2 . . . dn. Như v y so k nhỏ nhat can tìm thỏa mãn đieu ki n k ≤ n − 2. Ta cháng tỏ k = n − 2, tác là neu ton tại d1, d2, . . . , dn thỏa mãn giả thiet của bài toán sao cho (d1 + d2 + · · · + dn)n−3 không chia het cho tích d1d2 . . . dn. Ta lay d1 = 1, d2 = n − 1, di = n, 3 ≤ . i ≤ n. Khi đó d1 + d2 + · · · + dn = n(n − 1).di ∀1 ≤ i ≤ n. M t khác, d1d2 . . . dn = (n − 1) nn−2 . Do (n, n−1) = 1 nên lũy thàa bé hơn (n−2) của tőng d1 +d2 +· · ·+dn không the chia het cho tích d1, d2, . . . , dn. Bài toán 2.37. (IMO 2002) Cho n là so nguyên dương lớn hơn 1. Tat cả các ước nguyên dương của n là d1; d2; . . . ; dk với 1 = d1 < d2 < · · · < dk = n. Đ t d = d1d2 + d2d3 + · · · + dk−1dk. a) Cháng minh rang D < n2 .
  • 43. 39 Viết đề tài giá sinh viên – ZALO:0973.287.149-TEAMLUANVAN.COM n2 n2 n2 b) Xác định tat cả n sao cho D là m®t ước so của n2 . Lài giai. a) Nh n xét rang d1dk = d2dk−1 = · · · = dl.dk−l+1 = · · · = dkd1 = n. Do đó D < n2 ⇔ d1d2 + d2d3 + · · · + dk−1dk < n ⇔ d1d2 + d2d3 + · · · + dk−1dk < 1 ⇔ d1d2 + d2d3 + · · · + dk−1dk < 1 d1dk.d2dk−1 d2dk−1.d3dk−2 d2dk−1.dk.d1 ⇔ M t khác, thì 1 d1d2 1 + d2d3 + · · · + d 1 k−1 .dk < 1. (2.5) Ve trái (2.5) < 1 − d + 1 − d + · · · + 1 − 1 = 1 − 1 = 1. d1 2 d2 3 dk−1 dk dk Vì v y bat đȁng thác (2.5) đúng và D < n2 . b) Neu n = p là so nguyên to thì D = n là ước của n2 . Neu n là hợp so và goi p là ước so nguyên to nhỏ nhat của n thì dk = n, d2 = p, dk−1 = n và do đó D > p n2 p = dk−1dk. 2 n2 2 Theo ket quả câu a, ta có n > D > p nên D ko the là ước so của n được. Bài toán 2.38 (Hungari MO 2000). Tìm tat cả các so nguyên to p sao cho với so p đó, ton tại các so nguyên dương n, x, y thỏa mãn đieu ki n pn = x3 + y3 . Lài giai. Các so nguyên to tìm được là 2 và 3. Th t v y, ta có 21 = 13 + 13 và 32 = 12 + 23 . Xét p > 3. Giả sả ton tại các so nguyên dương n, x, y thỏa mãn đieu ki n pn = x3 + y3 . Khi đó, ta chon n, x, y sao cho so n nhỏ nhat trong tat cả các b® ba so như the ton tại. Vì p /= 2 nên (x; y) (1; 1). Vì v y x2 − xy + y2 = (x − y)2 + xy > 1 và x + y > 1. Do x2 − xy + y2 và x + y cùng chia het x3 + y3 nên suy ra chúng là b®i so của p, do đó p chia het (x + y)2 − (x2 − xy + y2 ) = 3xy. 2
  • 44. 40 Viết đề tài giá sinh viên – ZALO:0973.287.149-TEAMLUANVAN.COM 3 2 2x + 2y+ 3= 5, 3 2 2x 2y +3 = 5, Suy ra p chia het m®t trong ít nhat hai so x, y vì p không chia het 3. Hơn nǎa, p không the chia het chỉ m®t trong hai so x và y vì ta có p| (x + y). V y nên p chia het x và p chia het y. Tà đó, suy ra n > 3, chia hai ve của pn = x3 + y3 cho p3 , ta có pn−1 = x 3 y 3 Đieu này mâu thuan với cách chon n bé nhat như đã nói ở trên. 2.7 M t so phương pháp khác Tiep theo, xét m®t so bài toán ve phương trình nghi m nguyên có trong các kỳ Olympic, thi hoc sinh giỏi. Bài toán 2.39 (Rumania MO, xem [4]). Tìm tat cả các nghi m nguyên của phương trình sau x6 + 3x3 + 1 = y4 . Lài giai. Phương trình đã cho tương đương với (x3 + 1)2 + (x3 + 1) = y4 + 1, hay Do đó (2x3 + 3)2 − 4y4 = 5. (2x3 + 3 + 2y2 ).(2x3 + 3 − 2y2 ) = 5. Ta thu được các h phương trình sau 2x3 − 2y2 + 3 = 1, 2x3 − 2y2 + 3 = 5, 2x3 + 2y2 + 3 = 1, 2x3 − 2y2 + 3 = −1, 2x3 + 2y2 + 3 = −5, — − 2x3 + 2y2 + 3 = −1. Giải các h trên ta thu được nghi m của phương trình ban đau là (0, 1), (0, −1). Bài toán 2.40 (16th USA MO, xem [5]). Tìm tat cả các so nguyên x, y khác 0 của phương trình sau (x2 + y)(x + y2 ) = (x − y)3 . p p + .
  • 45. 41 Viết đề tài giá sinh viên – ZALO:0973.287.149-TEAMLUANVAN.COM Lài giai. Ta viet phương trình đã cho dưới dạng phương trình b c hai ȁn y 2y2 + (x2 − 3x)y + 3x2 + x = 0. Ta có ∆ = (x2 − 3x)2 − 8(3x+ x) = x(x + 1)2 (x − 8). Phương trình trên có nghi m nguyên khi và chỉ khi x(x + 1)2 (x − 8) là so chính phương. Đ t hay tương đương với x(x − 8) = z2 , (x − 4)2 − z2 = 16, (x − z − 4)(x + z − 4) = 16. De dàng tìm được x ∈ {−1, 8, 9}. Do đó, nghi m của phương trình ban đau là (−1, −1), (8, −10), (9, −6), (9, −21). Bài toán 2.41 (Armenia MO, xem [4]). Tìm tat cả các nghi m nguyên (x, y) của phương trình sau Lài giai. Ta có (x + 1)4 − (x − 1)4 = y3 . (x + 1)4 − (x − 1)4 = 8x3 + 8x. Giả sả (x, y) là m®t nghi m của phương trình và x ≥ 1. Khi đó (2x)3 < (x + 1)4 − (x − 1)4 < (2x + 1)3 . Do đó 2x < y < 2x + 1, (vô lí) . Như v y, neu (x, y) là nghi m của phương trình thì x phải là so nguyên không dương. Ta thay rang, neu (x, y) là nghi m của phương trình thì (−x, −y) cũng là nghi m. Do đó, −x cũng phải là so nguyên không dương. V y chỉ có the x = 0. V y nên phương trình đã cho có nghi m duy nhat là (0, 0). Bài toán 2.42 (29th IMO). Cho a, b là các so nguyên dương thỏa mãn đieu ki n ab + 1|a2 + b2 . Cháng tỏ rang a2 + b2 là m®t so chính phương. ab + 1
  • 46. 42 Viết đề tài giá sinh viên – ZALO:0973.287.149-TEAMLUANVAN.COM Lài giai. Goi (a, b) là c p so nguyên dương thỏa mãn đieu ki n bài toán thì (a, b) là nghi m của phương trình a2 − kab + b2 = k. De thay a = 0 ho c b = 0 thì k là so chính phương. Neu a 0 và b 0 thì a và b cùng dau. Th t v y, neu ab < 0 thì a2 − kab + b2 > k. Giả sả a > 0, b > 0 và k > 0. Neu a = b thì (2 − k)a2 = k > 0. Suy ra k = 1. Neu a > b > 0. Goi b là so nguyên dương nhỏ nhat sao cho (a, b) là nghi m của phương trình. Rõ ràng (b, kb − a) cũng là nghi m. Theo trên, neu kb = a thì k là so chính phương. M t khác kb − a > 0 (vì (kb − a) và b cùng dau) nên kb − a < b. Th t v y, kb − b < b ⇔ k < a + b b ⇔ a2 + b2 < 1 + ab a + 1. b Ta cháng minh bat đȁng thác cuoi cùng luôn đúng. Th t v y, a2 + b2 1 + ab a2 + ab < < 1 + ab a2 + ab = ab a + 1. b Như v y, ta đã cháng minh được (b, kb − a) là nghi m của phương trình ban đau nhưng kb − a < b. Đieu này mâu thuan với giả sả b là so nguyên dương nhỏ nhat sao cho (a, b) là nghi m. V y k là so chính phương. Bài toán 2.43 (Olympic Toán Hà N®i mở r®ng (HOMC), 2009). Cháng minh phương trình (x + 1)2 + (x + 2)2 + · · · + (x + 99)2 = yz không có nghi m nguyên (x, y, z) với z > 1. Lài giai. Ta có tương đương với yz = (x + 1)2 + (x + 2)2 + · · · + (x + 99)2 , Do đó yz = 99x2 + 2(1 + 2 + · · · + 99)x + (12 + 22 + · · · + 992 ). yz = 99x2 + 2.99.100 .x + 99.100.199 = 33(3x2 + 300x + 50.199). 2 6 Vì ve phải chia het cho 3 nên 3|y. Suy ra, 32 |yz . Nhưng ve phải không chia het cho 9. V y nên phương trình đã cho không có nghi m nguyên với z > 1.
  • 47. 43 Viết đề tài giá sinh viên – ZALO:0973.287.149-TEAMLUANVAN.COM 2 √ √ Bài toán 2.44 (Bucharest MO, xem [4]). Tìm tat cả các c p so nguyên (x, y) của phương trình x3 − 4xy + y3 = −1. Lài giai. Nhân cả hai ve của phương trình với 27 và công cả hai ve với 64, ta được Sả dụng ket quả 27x3 + 27y3 + 43 − 4.27xy = 37. a3 + b3 + c3 − 3abc = (a + b + c)(a2 + b2 + c2 − ab − bc − ca), phương trình tương đương với (3x + 3y + 4)(9x2 + 9y2 + 16 − 9xy − 12x − 12y) = 37. Vì 37 là so nguyên to và (9x2 + 9y2 + 16 − 9xy − 12x − 12y) = 1 [(3x − 3y)2 + (3x − 4)2 + (3y − 4)2 ] ≥ 0. nên 3x + 3y + 4 = 1 ho c 3x + 3y + 4 = 37. Neu 3x + 3y + 4 = 1 thì 9x2 + 9y2 + 16 − 9xy − 12x − 12y = 37. Suy ra nghi m là (−1, 0) và (0, −1). Neu 3x + 3y + 4 = 37 thì 9x2 + 9y2 + 16 − 9xy − 12x − 12y = 1. Suy ra (3x − 3y)2 + (3x − 4)2 + (3y − 4)2 = 2. Đieu này không the xảy ra vì x, y là các so nguyên khác nhau nên |3x − 3y| ≥ 3. V y nghi m của phương trình ban đau là (−1, 0) và (0, −1). Bài toán 2.45 (Hungary MO, xem [4]). Tìm tat cả các nghi m nguyên dương (x, y, z) của phương trình x3 + 3y3 + 9z3 − 3xyz = 0. Lài giai. De thay (0, 0, 0) là m®t nghi m của phương trình. Giả sả (x1, y1, z1) là m®t nghi m khác của phương trình trong đó x1 là so nguyên dương nhỏ nhat trong các giá trị nh n được của x. Neu m®t trong các so x1, y1, z1 bang 0. Do 3 3 và 3 9 là các so vô tỉ nên hai so còn lại cũng bang 0. Ta có the giả sả x1, y1, z1 > 0. Rõ ràng 3|x1. Đ t x1 = 3x2, với x1 nguyên. Thay vào phương trình roi chia cả hai ve cho 3 ta có 9x2 3 + y1 3 + 3z1 3 − 3x2y1z1 = 0.
  • 48. 44 Viết đề tài giá sinh viên – ZALO:0973.287.149-TEAMLUANVAN.COM Suy ra 3|y‘1. Đ t y1 = 3y2, với y2 nguyên. Thay vào phương trình trên roi chia cả hai ve cho 3 ta có 3x2 3 + 9y2 3 + z1 3 − 3x2y2z1 = 0. Như v y 3|z1. Đ t z1 = 3z2, với z2 nguyên. Thay vào phương trình trên roi chia cả hai ve cho 3 ta có x2 3 + 3y2 3 + 9z2 3 − 3x2y2z2 = 0. Do đó (x2, y2, z2) cũng là nghi m của phương trình ban đau. M t khác, x1 = 3x2, tác là x1 > x2. Đieu này trái với giả thiet x1 là so nguyên dương nhỏ nhat trong các giá trị nh n được của x. V y nên phương trình đã cho không có nghi m nguyên dương. Bài toán 2.46 (5th USA MO, xem [6]). Tìm tat cả các nghi m nguyên (a, b, c) của phương trình a2 + b2 + c2 = a2 b2 . Lài giai. Ta sě cháng minh a, b, c đeu là các so chȁn. Ta có the giả sả a, b, c là các so nguyên không âm. Ta có (2n)2 ≡ 0 (mod 4)và (2n + 1)2 ≡ 1 (mod 4). Trường hợp 1. Khi a, b, c đeu là so lẻ. a2 + b2 + c2 ≡ 3 (mod 4)nhưnga2 b2 ≡ 1 (mod 4). Trường hợp 2. Hai trong ba so a, b, c là so lẻ. a2 + b2 + c2 ≡ 2 (mod 4)nhưnga2 b2 ≡ 0ho c1 (mod 4). Trường hợp 3. Hai trong ba so a, b, c là so chȁn. a2 + b2 + c2 ≡ 1 (mod 4)nhưnga2 b2 ≡ 0 (mod 4). V y cả ba so a, b, c đeu chȁn. Đ t a = 2a1, b = 2b1, c = 2c1, với a1, b1, c1 là các so nguyên. Thay vào phương trình roi chia cả hai ve cho 4 ta có a1 2 + b1 2 + c1 2 = 4a1 2 b1 2 . L p lu n tương tự, ta có 4a1 2 b1 2 ≡ 0 (mod 4) và moi so a1 2 , b1 2 , c1 2 ≡ 0 ho c 1 (mod 4). Suy ra a1 2 ≡ b1 2 ≡ c1 2 ≡ 0 (mod 4), áng với a1, b1, c1 là các so chȁn.
  • 49. 45 Viết đề tài giá sinh viên – ZALO:0973.287.149-TEAMLUANVAN.COM Đ t a1 = 2a2, b1 = 2b2, c1 = 2c2, với a2, b2, c2 là các so nguyên. Thay vào phương trình roi chia cả hai ve cho 4 ta có a2 2 + b2 2 + c2 2 = 16a2 2 b2 2 . Cá tiep tục như v y, dan đen a, b, c đeu chia het cho 2k , với k là so tự nhiên tùy ý. Đieu này chỉ xảy ra khi a = b = c = 0. Đó chính là nghi m nguyên duy nhat của phương trình đã cho. Bài toán 2.47 (8th USA MO). Tìm tat cả các nghi m nguyên không âm (x1, x2, . . . , x14) (không ke các hoán vị của nó) của phương trình x1 4 + x2 4 + · · · + x14 4 = 15999. Lài giai. - Với n là so chȁn, đ t n = 2k, với k nguyên thì n4 = 16k4 ≡ 0 (mod 16). - Với n là so lẻ thì n4 − 1 = (n − 1) (n + 1) n2 + 1 . Vì (n − 1), (n + 1) là hai so chȁn liên tiep và n2 + 1 cũng là so chȁn nên n4 − 1 = (n − 1) . (n + 1) . n2 + 1 ≡ 0 (mod 16). Do đó ho c Suy ra nên M t khác, ta có n4 ≡ 0 (mod 16) n4 ≡ 1 (mod 16). x1 4 + x2 4 + · · · + x144 ≡ r (mod 16) 0 ≤ r ≤ 14. 1599 = 16000 − 1 ≡ 15 (mod 16). V y nên phương trình đã cho không có nghi m. Bài toán 2.48 (37th IMO). Tìm tat cả các nghi m nguyên dương (a, b) của phương trình ab2 = ba . Lài giai. Ta sě cháng minh phương trình chỉ có các nghi m là (1, 1), (16, 2), (27, 3).
  • 50. 46 Viết đề tài giá sinh viên – ZALO:0973.287.149-TEAMLUANVAN.COM Goi (a, b) là m®t nghi m của phương trình. Đ t (a, b) = d thì a = du, b = dv, trong đó (u, v) = 1. Phương trình đã cho tương đương với (du)dv 2 = (dv)u . Ta xét các trường hợp sau. Trường hợp 1. Neu dv2 = u thì u = v. M t khác (u, v) = 1 nên u = v = 1, d = 1. Do đó nghi m của phương trình là (a, b) = (1, 1). Trường hợp 2. Neu dv2 > u thì ta viet được phương trình dưới dạng ddv2 −u udv2 = vu . Rõ ràng udv2 |vu . Vì (u, v) = 1 nên u = 1. Khi đó, phương trình trở thành ddv2 −1 = v. Neu d = 1 thì v = 1. Đieu này trái với giả thiet dv2 > u. Neu d ≥ 2 thì ddv2−1 geq22v2−1 ≥ 22v−1 > v, v = 1, 2, 3, . . . Do đó, trong trường hợp này phương trình cũng không có nghi m. Trường hợp 3. Neu dv2 < u thì d < u. Ta viet được phương trình dưới dạng udv2 = du−dv2 vu . Rõ ràng vu |udv2 . Vì (u, v) = 1 nên v = 1. Khi đó, phương trình trở thành ud = du−d .
  • 51. 47 Viết đề tài giá sinh viên – ZALO:0973.287.149-TEAMLUANVAN.COM 2018 Chương 3. Các dạng toán liên quan đen phương trình và h phương trình Diophant Trong nhieu bài toán thực te, người ta can tìm các nghi m nguyên dương (chá không đơn thuan là nghi m nguyên) của h phương trình đại so dạng Ax = b, trong đó A ∈ Rm×n là ma tr n nguyên và rank(A) = m, b ∈ Rm là véctơ nguyên. Khi đó, người ta thường sả dụng các phương pháp như đã trình bày trong Chương 2 đe tìm nghi m nguyên dương của h phương trình tương áng. 3.1 M t so dạng toán ve đa thfíc nguyên Bài toán 3.1 (Thi chon ĐTQG, Thanh Hóa 2017). Tìm tat cả các đa thác P (x) với h so là các so nguyên thỏa mãn đieu ki n n(n−1) − 1 chia het cho P (n), với moi so nguyên dương n. Lài giai. Giả sả P (x) là m®t đa thác thỏa mãn bài toán. Xét các trường hợp: - Neu deg P (x) = 0 thì P (x) ≡ a0, a0 ∈ Z. M t khác V y P (x) ≡ ±1 2(2−1)2018 − 1 .P (2) = a0 ⇔ a0 = ±1. - Neu deg P (x) ≥ 1. Đ t P (x) = amxm + am−1xm−1 + · · · + a1x + a0 với m ∈ N∗ , ai ∈ Z, ∀i = 0; m. - Neu am > 0 thì ton tại so nguyên dương N sao cho P (x) > 0, ∀x > N. Với n ∈ N∗ , n > N, xét so nguyên to p là ước của P (n). (1) . .p n(n−1)2018 − 1 Tà giả thiet suy ra
  • 52. 48 Viết đề tài giá sinh viên – ZALO:0973.287.149-TEAMLUANVAN.COM p M t khác P (n + p) = am(n + p)m + am−1(n + p)m−1 + · · · + a1 (n + p) + a0 = P (n) + pQ (n) , suy ra P (n + p) .p (với Q (n ) ∈ Z). Vì (n + p)(n+p−1) 2018 − 1 .P (n + p) nên (n + p)(n+p−1) 2018 ≡ 1 (mod p) nên n(n+p−1) 2018 ≡ 1 (mod p), do đó (n, p) = 1. Theo Định lý Euler, ta có np−1 ≡ 1 (mod p). Khi đó, tà (n + p − 1)2018 = n2018 + (p − 1) A (với A ∈ N∗ ), suy ra n(n+p−1)2018 ≡ nn2018 +(p−1)A ≡ nn2018 np−1 A ≡ nn2018 (mod p), do n(n+p−1) 2018 ≡ 1 (mod p) suy ra nn2018 ≡ 1 (mod p) hay nn2018 − 1 .p. (2) Tà (1) , (2) và sả dụng tính chat (am − 1; an − 1) = a(m;n) − 1, suy ra nn2018 − 1; n(n−1) 2018 − 1 .p nên Do đó n(n2018 ;(n−1)2018 ) − 1 .p. (do n2018 ; (n − 1)2018 = 1) (n − 1) .p, (3) Xét so nguyên to q, q > N thỏa mãn đieu ki n P (q + 1) .p. Theo (3) ta có (q + 1 − 1) .p nên q.p. Vì p, q là các so nguyên to nên q = p. Tà đó, suy ra P (q + 1) = pkp với moi so nguyên to p > N, kp là so nguyên dương phụ thu®c p. Goi vp (n) là so mũ đúng của ước nguyên to p trong n, nghĩa là n.pk nhưng n / . p k+1. Sả dụng tính chat, áng với x, y là các so nguyên, n nguyên dương và p là so nguyên to lẻ sao cho p |(x − y) và x, y không chia het cho p thì vp (xn − yn ) = vp (x − y) + vp (n) . Suy ra v (p + 1)p 2018 − 1 = v ((p + 1) − 1) + vp p2018 = 2019 p
  • 53. 49 Viết đề tài giá sinh viên – ZALO:0973.287.149-TEAMLUANVAN.COM Σ Y Σ Y Σ = (−1) C 2016 k 2016 k k+1 2016 2017 2016 nên kp ≤ 2019 với moi so nguyên to p > N. Do dãy (kp) có vô hạn phan tả (vì có vô hạn so nguyên to p > N) nên ton tại m®t dãy con thỏa mãn đieu ki n P (q + 1) = pk với vô so so nguyên to p. Suy ra P (x) = (x − 1)k với k ∈ N∗ , 1 ≤ k ≤ 2019. - Neu am < 0, bang cách đ t Q (x) = −P (x) và làm tương tự ta có Q (x) = (x − 1)k , suy ra P (x) = −(x − 1)k với k ∈ N∗, 1 ≤ k ≤ 2019. Thả lại, ta thay các đa thác P (x) = (x − 1)k và P (x) = −(x − 1)k với k ∈ N∗ , 1 ≤ k ≤ 2019 không thỏa mãn yêu cau bài toán tại n = 1. V y tat cả đa thác can tìm là P (x) ≡ ±1. Bài toán 3.2 (Thi chon ĐTQG, Đà Nȁng 2017). Tìm so nguyên dương n nhỏ nhat sao cho ton tại đa thác f (x) b c n có h so nguyên thỏa mãn đieu ki nf (0) = 0, f (1) = 1 và với moi m ∈ N∗ , f (m) (f (m) − 1) là b®i của 2017. Lài giai. Ta sě cháng minh n ≥ 2016 thỏa mãn. Giả sả 1 ≤ n ≤ 2015 và ton tại đa thác f (x) thỏa mãn bài toán. Áp dụng công thác n®i suy Lagrange Suy ra 2015 f (x) = f (k) k=0 2015 i=0,i= / x − i . k k − i 2015 f (2016) = f (k) k=0 2015 i=0,i= / k 2016 − i k − i 2015 k+1 k 2016 k=0 f (k). Ta cháng minh Ck ≡ (−1) (mod 2017) , k = 0, 1, 2, . . . , 2015 bang phương pháp quy nạp. Với k = 0 thì C0 ≡ 1 (mod 2017). Giả sả đúng đen k, tác là thì k 2016 ≡ (−1) (mod 2017) Ck+1 = Ck+1 − Ck ≡ (−1) (mod 2017) . C
  • 54. 50 Viết đề tài giá sinh viên – ZALO:0973.287.149-TEAMLUANVAN.COM Σ Σ Σ Σ Σ V y f (k) = − 2015 k=0 f (k) (mod 2017) nên Nhưng 2016 k=0 f (k) = 1 + 2016 k=2 2016 f (k) ≡ 0 (mod 2017) . k=0 f (k) và Do đó 2016 k=0 f (k) ≡ 0; 1 (mod 2017) ; k = 2, . . . , 2016. f (k) / ≡0 (mod 2017) (mâu thuan với nh n xét trên). V y n ≥ 2016. Với n = 2016, xét đa thác f (x) = x2016 thỏa mãn yêu cau bài toán. Do đó n nhỏ mhat bang 2016. Bài toán 3.3 (Thailand MO 2014). Tìm tat cả các đa thác P (x) với h so là các so nguyên thỏa mãn đieu ki n P (n) |2557n + 213.2014, ∀n ∈ N∗. Lài giai. De thay P (n) = 1, ∀n ∈ N∗ và P (n) = −1, ∀n ∈ N∗ là nhǎng đa thác thỏa mãn yêu cau bài toán. Giả sả đa thác P (x) thỏa mãn bài toán và ton tại n0 ∈ N∗ : |P (n0)| ≥ 2. Goi p là m®t ước nguyên to của P (n0). Ta có P (n0) |2557n0 + 213.2014 và P (n0 + p) |2557n0+p + 213.2014. Do đó p |(P (n0 + p) − P (n0)) |2557n0 (2557p − 1). M t khác, vì p |P (n0) |2557n0 + 213.2014 nên p / ∈ {2, 3, 19, 53, 71, 2557} . Do đó p |(2557p − 1). M t khác theo Định lý Fermat p |(2557p − 2557) thì p |2556 suy ra p ∈ {2, 3, 71}, vô lý. V y chỉ có P (n) = 1, ∀n ∈ N∗ và P (n) = −1, ∀n ∈ N∗ là hai đa thác thỏa mãn yêu cau bài toán. 3.2 M t so dạng toán lư ng giác liên quan Trong phan này ta xét m®t so dạng toán lượng giác liên quan can đen ky thu t giải phương trình và h Diophant.
  • 55. 51 Viết đề tài giá sinh viên – ZALO:0973.287.149-TEAMLUANVAN.COM 2 2x + y = 12k + 1 x + y = 6l + 1 Bài toán 3.4. Xác định so nghi m nguyên của phương trình lượng giác cos π x = 0 trong khoảng (−2015; 2015). Lài giai. Phương trình đã cho tương đương với hay π π 2 x = 2 + kπ (k ∈ Z) x = 2k + 1. Do x ∈ (−2015; 2015) , nên ta có −2015 < 2k + 1 < 2015, tác là −1008 < k < 1007. V y trong khoảng (−1008; 1007) có 2016 so nguyên. V y nên phương trình đã cho có 2016 nghi m nguyên x trong khoảng (−2015; 2015). Bài toán 3.5. Tìm các nghi m nguyên (x, y) của h phương trình lượng giác sin π(2x + y) = 1 6 2 π(x + y) 1 cos 3 = 2 Biet rang x, y thu®c khoảng (−6; 10). Lài giai. H phương trình đã cho tương đương với 2x + y = 12m + 5 (k, m, l, h ∈ Z). x + y = 6h − 1 Trường hợp 1. Xét h phương trình 2x + y = 12k + 1 x + y = 6l + 1.
  • 56. 52 Viết đề tài giá sinh viên – ZALO:0973.287.149-TEAMLUANVAN.COM H này tương đương với Ket hợp với đieu ki n x = 12k − 6l y = 12l − 12k + 1. x, y ∈ (−6; 10) ta thu được h bat phương trình −6 < 12k − 6l < 10 −7 < 12l − 12k < 9. Tà đó suy ra −19 < 12k < 29, −1 ≤ k ≤ 2. - Với k = −1, thay vào h bat phương trình trên ta không thu được giá trị nguyên nào của l. - Với k = 0, thay vào h bat phương trình trên ta thu được l = 0. Do đó (x, y) = (0, 1). - Với k = 1, thay vào h bat phương trình trên ta thu được l = 1. Do đó (x, y) = (6, 1).’ - Với k = 2, thay vào h bat phương trình trên ta không thu được giá trị nguyên nào của l. Trường hợp 2. Xét h phương trình 2x + y = 12k + 1 H này tương đương với x + y = 6h − 1. Ket hợp với đieu ki n x = 12k − 6h + 2 y = 12h − 12k − 3. x, y ∈ (−6; 10) ta thu được h bat phương trình −8 < 12k − 6h < 8 −3 < 12h − 12k < 13.
  • 57. 53 Viết đề tài giá sinh viên – ZALO:0973.287.149-TEAMLUANVAN.COM Tà đó suy ra −19 < 12k < 29, −1 ≤ k ≤ 2. - Với k = −1, thay vào h bat phương trình trên ta thu được h = −1. Do đó (x, y) = (−4, −3). - Với k = 0, thay vào h bat phương trình trên ta thu được h = 0, h = 1. Do đó (x, y) = (2, −3), (x, y) = (−4, 9). - Với k = 1, thay vào h bat phương trình trên ta thu được h = 1, h = 2. Do đó (x, y) = (8, −3), (x, y) = (2, 9). - Với k = 2, thay vào h bat phương trình trên, ta thu được h = 3. Do đó (x, y) = (8, 9). Trường hợp 3. Xét h phương trình 2x + y = 12m + 5 H này tương đương với x + y = 6l + 1. Ket hợp với đieu ki n x = 12m − 6l + 4 y = 12l − 12m − 3. x, y ∈ (−6; 10) ta thu được h bat phương trình −10 < 12m − 6l < 6 −3 < 12l − 12m < 13. Tà đó suy ra −23 < 12m < 25, −1 ≤ m ≤ 2. - Với m = −1, thay vào h bat phương trình trên ta thu được l = −1. Do đó (x, y) = (−2, −3). - Với m = 0, thay vào h bat phương trình trên ta thu được l = 0, l = 1. Do đó (x, y) = (4, −3), (x, y) = (−2, 9).
  • 58. 54 Viết đề tài giá sinh viên – ZALO:0973.287.149-TEAMLUANVAN.COM - Với m = 1, thay vào h bat phương trình trên, ta thu được l = 2. Do đó (x, y) = (4, 9). - Với m = 2, thay vào h bat phương trình trên, ta không thu được giá trị nguyên nào của l. Trường hợp 4. Xét h phương trình 2x + y = 12m + 5 H này tương đương với x + y = 6l − 1. Ket hợp với đieu ki n x = 12m − 6h + 6 y = 12h − 12m − 7. x, y ∈ (−6; 10) ta thu được h bat phương trình −12 < 12m − 6h < 4 1 < 12h − 12m < 17. Tà đó suy ra −23 < 12m < 25, −1 ≤ m ≤ 2. - Với m = −1, thay vào h bat phương trình trên, ta không thu được giá trị nguyên nào của h. - Với m = 0, thay vào h bat phương trình trên, ta thu được h = 1. Do đó (x, y) = (0, 5). - Với m = 1, thay vào h bat phương trình trên, ta thu được h = 2. Do đó (x, y) = (6, 5). - Với m = 2, thay vào h bat phương trình trên ta không thu được giá trị nguyên nào của h. V y h phương trình đã cho ban đau có 14 nghi m nguyên (x, y) bao gom (0, 5) , (6, 5) , (−2, −3) , (4, −3) , (−2, 9) , (4, 9) , (−4, −3) , (2, −3) , (−4, 9) , (8, −3) , (2, 9) , (8, 9) , (0, 1) , (6, 1)
  • 59. 55 Viết đề tài giá sinh viên – ZALO:0973.287.149-TEAMLUANVAN.COM sin 2015x > 1 6 6 3 3 445 3 445 3 4 hay với x, y thu®c khoảng (−6; 10). Tóm lại, đe khảo sát phương trình, h phương trình, bat phương trình, h bat phương trình lượng giác có đieu ki n, ta thực hi n các bước sau: Bước 1. Tìm nghi m tőng quát (x1, x2, . . . , xn) của phương trình (h phương trình, bat phương trình, h bat phương trình) lượng giác. Bước 2. Tà đieu ki n của phương trình (h phương trình, bat phương trình, h bat phương trình) lượng giác, ta hạn che đieu ki n của các tham so trong nghi m tőng quát (x1, x2, . . . , xn). Bài toán 3.6. Giải h bat phương trình 2 1 cos 445x ≤ 2 . Lài giai. H bat phương trình đã cho tương đương với π 5π + k2π < 2015x < + k2π π 4π + m2π ≤ 445x ≤ + m2π x = π 1 + 2k < x < π 5 + 2k = x 2015 6 2015 6 2 x = π 1 + 2m ≤ x ≤ π 4 + 2m = x . - Trường hợp 1: x ∈ [x3; x2) neu x1 < x3 < x2 ≤ x4. + x1 < x3 tương đương với do đó 3 a = 403m − 89k > −59 4 a ≥ −59. + x3 < x2 tương đương với 1 a = 403m − 89k < −30 12 1 (k, m ∈ Z) 3
  • 60. 56 Viết đề tài giá sinh viên – ZALO:0973.287.149-TEAMLUANVAN.COM 1 445 3 2015 6 do đó + x2 ≤ x4 tương đương với a ≤ −31. 7 do đó a = 403m − 89k ≥ −231 12 a ≥ −231. V y nên x ∈ [x3; x2) neu 403m − 89k = a trong đó a ∈ Z, −59 ≤ a ≤ −31. Ta có nên 403 89 C = p4 4 q4 = [4; 1, 1, 8, 2, 2] 163 = [4; 1, 1, 8, 2] = 36 Khi đó, phương trình có m®t nghi m riêng là Suy ra phương trình p4 = 163, q4 = 36. 403m − 89k = 1 (m1; k1) = (36; 163) . 403m − 89k = a trong đó a ∈ Z, −59 ≤ a ≤ −31 có nghi m tőng quát là m = 36a + 89t k = 163a + 403t (t ∈ Z) . S = π 1 + 72a + 178t ; π 5 + 326a + 806t , trong đó a, t ∈ Z, −59 ≤ a ≤ −31. - Trường hợp 2: x ∈ (x3; x4] neu x1 ≤ x3 và x4 < x2. + x1 ≤ x3 tương đương với 3 a = 403m − 89k ≥ −59 4 V y trong trường hợp này, ta có t p nghi m của h bat phương trình đã cho là
  • 61. 57 Viết đề tài giá sinh viên – ZALO:0973.287.149-TEAMLUANVAN.COM 2 2015 6 2015 6 sin(2x + y) = 1 1 do đó + x4 < x2 tương đương với a ≥ −59. 7 Do đó a = 403m − 89k < −231 12 . a ≤ −232. Như v y trường hợp 2 không xảy ra. - Trường hợp 3: x ∈ (x1; x2) neu x3 ≤ x1 và x2 ≤ x4. + x3 ≤ x1 tương đương với Do đó 3 a = 403m − 89k ≤ −59 4 . a ≤ −60. + x2 ≤ x4, theo ket quả của trường hợp 1 thì tương đương với a ≥ −231. V y trong trường hợp này, ta có t p nghi m của h bat phương trình đã cho là S = π 1 + 326b + 806l ; π 5 + 326b + 806l , trong đó b, l ∈ Z, −231 ≤ b ≤ −60. Ket lu n: T p nghi m của h bat phương trình ban đau là S = S1 ∪ S2. Bài toán 3.7. Giải h phương trình 2 với đieu ki n x − y ≥ 10π. cos(x + y) = 2 Lài giai. H phương trình đã cho tương đương với
  • 62. 58 Viết đề tài giá sinh viên – ZALO:0973.287.149-TEAMLUANVAN.COM − 2 2x + y = π + k2π 2x + y = 5π + m2π x + y = π + l2π 2x + y = π + k2π x = − π + (a + t)2π π 6 6 (k, m, l, h ∈ Z). 3 x + y = π + h2π 3 - Trường hợp 1: Xét h phương trình 6 có nghi m là x + y = 3 + l2π x = − π + (k − l)2π 6 π y = 2 + (2l − k)2π. Ket hợp đieu ki n x − y ≥ 10π ta thu được bat phương trình 16 2k − 3l ≥ 3 . Bat phương trình này tương đương với trong đó a ∈ Z, a ≥ 6. Tà đó, ta có 2k − 3l = a V y nên k = 2a + 3t l = a + 2t (t ∈ Z). 6 y = π + t2π
  • 63. 59 Viết đề tài giá sinh viên – ZALO:0973.287.149-TEAMLUANVAN.COM 3 2x + y = π + k2π x = π + (b + t)2π 2x + y = 5π + m2π π trong đó a, t ∈ Z, a ≥ 6. - Trường hợp 2: Xét h phương trình 6 có nghi m là x + y = − 3 + h2π x = π + (k − h)2π 2 5π y = − 6 + (2h − k)2π. Ket hợp đieu ki n x − y ≥ 10π, ta thu được bat phương trình 13 2k − 3h ≥ 3 . Bat phương trình này tương đương với trong đó b ∈ Z, b ≥ 5. Tà đó, ta có 2k − 3h = b, V y nên k = 2b + 3t h = b + 2t (t ∈ Z). 2 trong đó b, t ∈ Z, b ≥ 5. 5π y = − 6 + t2π - Trường hợp 3: Xét h phương trình 6 x + y = π + l2π
  • 64. 60 Viết đề tài giá sinh viên – ZALO:0973.287.149-TEAMLUANVAN.COM x = π + (c + t)2π 2x + y = 5π + m2π π π có nghi m là x = π + (m − l)2π 2 π y = − 6 + (2l − m)2π. Ket hợp đieu ki n x − y ≥ 10π, ta thu được bat phương trình 14 2m − 3l ≥ 3 . Bat phương trình này tương đương với trong đó c ∈ Z, c ≥ 5. Tà đó, ta có 2m − 3l = c, V y nên m = 2c + 3t l = c + 2t (t ∈ Z). 2 trong đó c, t ∈ Z, c ≥ 5. y = − 6 + t2π - Trường hợp 4: Xét h phương trình 6 có nghi m là x + y = − 3 + h2π x = 7π + (m − h)2π 6 3π y = − 2 + (2h − m)2π. Ket hợp đieu ki n x − y ≥ 10π, ta thu được bat phương trình 11 2m − 3h ≥ 3 .
  • 65. 61 Viết đề tài giá sinh viên – ZALO:0973.287.149-TEAMLUANVAN.COM x = 7π + (d + t)2π sin(2x + y) = 1 π Bat phương trình này tương đương với 2m − 3h = d, trong đó d ∈ Z, d ≥ 4. Tà đó, ta có V y nên m = 2d + 3t h = d + 2t (t ∈ Z). 6 trong đó d, t ∈ Z, d ≥ 4. 3π y = − 2 + t2π Tóm lại, ta có 4 ho nghi m (x; y) thỏa mãn giả thiet bài ra, bao gom π x = − + (a + t)2π x = π + (b + t)2π 6 π y = + t2π, 2 2 5π y = − 6 + t2π, x = π + (c + t)2π x = 7π + (d + t)2π 2 y = − 6 + t2π, 6 3π y = − 2 + t2π, trong đó a, b, c, d, t ∈ Z, a ≥ 6, b ≥ 5, c ≥ 5, d ≥ 4. Bài toán 3.8. Giải h phương trình 2 cos(x + y) = 1 2 trên đoạn [−6π; 6π] và thỏa mãn đieu ki n x − y ≥ 10π. Lài giai. H đã cho có 4 ho nghi m (x; y) thỏa mãn đieu ki n x − y ≥ 10π, bao gom
  • 66. 62 Viết đề tài giá sinh viên – ZALO:0973.287.149-TEAMLUANVAN.COM π x = − 6 + (a + t)2π 6 ≤ a ≤ 6 + 1 π x = − + (a + t)2π x = π + (b + t)2π 6 π y = + t2π, 2 2 5π y = − 6 + t2π, x = + (c + t)2π x = + (d + t)2π π 7π 2 6 π y = − 6 + t2π, 3π y = − 2 + t2π trong đó a, b, c, d, t ∈ Z, a ≥ π 6, b ≥ 5, c ≥ 5, d ≥ 4. - Trường hợp 1: y = 2 + t2π (a, t ∈ Z, a ≥ 6). Với đieu ki n x, y ∈ [−6π; 6π] ta có π −6π ≤ − 6 + (a + t)2π ≤ 6π −6π ≤ π + t2π ≤ 6π 2 a, t ∈ Z, a ≥ 6. H này tương đương với 1 1 12 − 3 ≤ a + t ≤ 12 + 3 1 1 suy ra − 4 − 3 ≤ t ≤ − 4 + 3 a, t ∈ Z, a ≥ 6 3 a ∈ Z
  • 67. 63 Viết đề tài giá sinh viên – ZALO:0973.287.149-TEAMLUANVAN.COM 2 − 4 − 3 ≤ b + t ≤ − 4 + 3 - Trường hợp 2: (b, t ∈ Z, b ≥ 5). nên a = 6. Với a = 6, t = −3, ta thu được (x; y) = 35π ; − 11π . 6 2 x = π + (b + t)2π 5π y = − 6 + t2π Với đieu ki n x, y ∈ [−6π; 6π] , ta có π −6π ≤ 2 + (b + t)2π ≤ 6π −6π ≤ − 5π + t2π ≤ 6π 6 H này tương đương với b, t ∈ Z, b ≥ 5. 1 1 5 5 12 − 3 ≤ t ≤ 12 + 3 suy ra b, t ∈ Z, b ≥ 5 5 ≤ b ≤ 5 + 1 suy ra b = 5. Với b = 5, ta thu được 5 12 3 b ∈ Z, 1 — 3 ≤ t ≤ − 4 − 2 nên t ∈ ∅. t ∈ Z
  • 68. 64 Viết đề tài giá sinh viên – ZALO:0973.287.149-TEAMLUANVAN.COM 2 − 4 − 3 ≤ c + t ≤ − 4 + 3 V y trong trường hợp này ta không thu được nghi m (x; y) thỏa mãn yêu cau của bài toán. x = π + (c + t)2π π y = − 6 + t2π Với đieu ki n x, y ∈ [−6π; 6π] ta có π −6π ≤ 2 + (c + t)2π ≤ 6π −6π ≤ − π + t2π ≤ 6π 6 c, t ∈ Z, c ≥ 5. H này tương đương với 1 1 1 1 12 − 3 ≤ t ≤ 12 + 3 suy ra c, t ∈ Z, c ≥ 5 5 ≤ c ≤ 5 + 2 Với c = 5 ta thu được c ∈ Z 3 nên c = 5. 1 12 1 — 3 ≤ t ≤ − 4 − 2 nên t ∈ ∅. t ∈ Z V y trường hợp này ta không thu được nghi m (x; y) thỏa mãn yêu cau của bài toán. - Trường hợp 3: (c, t ∈ Z, c ≥ 5).
  • 69. 65 Viết đề tài giá sinh viên – ZALO:0973.287.149-TEAMLUANVAN.COM d ∈ Z 6 6 2 6 2 6 2 - Trường hợp 4: (d, t ∈ Z, d ≥ 4). x = 7π + (d + t)2π 3π y = − 2 + t2π Với đieu ki n x, y ∈ [−6π; 6π] ta có 7π −6π ≤ 6 + (d + t)2π ≤ 6π −6π ≤ − 3π + t2π ≤ 6π 2 d, t ∈ Z, d ≥ 4. H này tương đương với 7 7 − 12 − 3 ≤ d + t ≤ − 12 + 3 3 − 3 ≤ t ≤ 3 + 3 4 4 suy ra d, t ∈ Z, d ≥ 4 4 ≤ d ≤ 4 + 2 Với d = 4, t = −2, ta t hu được 3 nên d = 4. (x; y) = 31π ; − 11π . Tóm lại, bài toán đã cho có hai nghi m (x; y) bao gom 35π ; − 11π , 31π ; − 11π .
  • 70. 66 Viết đề tài giá sinh viên – ZALO:0973.287.149-TEAMLUANVAN.COM 3.3 M t so dạng toán thi Olympic liên quan Bài toán 3.9 (Đe thi chon đ®i tuyen quoc gia dự thi IMO 2006). Hãy tìm tat cả các c p so tự nhiên (n; k) với n là so nguyên không âm và k là so nguyên lớn hơn 1 sao cho so A = 172006n + 4.172n + 7.195n có the phân tích được thành tích của k so nguyên dương liên tiep. Lài giai. Trước het, ta thay rang tích của 4 so tự nhiên liên tiep phải chia het cho 8 vì trong 4 so đó có 1 so chia het cho 4 và m®t so chia 4 dư 2. Tà A = 172006n + 4.172n + 7.195n , suy ra - Neu n là so chȁn, ta có 172006n ≡ 1 (mod 8); 4.172n ≡ 4.1 (mod 8); 195n ≡ 7.35n ≡ 7.310 ≡ 7 (mod 8). V y nên A ≡ 12 ≡ 4 (mod 8), tác là A không chia het cho 8. - Neu n là so lẻ, cũng tương tự 172006n ≡ 1 (mod 8); 4.172n ≡ 4.1 (mod 8); 195n ≡ 7.35n ≡ 7.35 ≡ 7.3 (mod 8). Suy ra A ≡ 10 ≡ 2 (mod 8), tác là A cũng không chia het cho 8. Tác là, trong moi trường hợp luôn có A không chia het cho 8. Suy ra neu k thỏa mãn đe bài thì k < 4, suy ra k ∈ {2, 3}. Xét tàng trường hợp cụ the. - Neu k = 2 thì ton tại x tự nhiên sao cho A = x(x + 1). + Neu n = 0 thì A = 12, x = 3, thỏa mãn đe bài. + Neu n > 0 thì rõ ràng 171003n > 4.172n + 7.195n . Ta thay A = x(x + 1) = 172006n + 4.172n + 7.195n > 172006n . Suy ra x > 171003n . Nhưng x(x + 1) > 172006n + 171003n > A, mâu thuan. Do đó, trong trường hợp này không có n thỏa mãn đe bài. - Neu k = 3 thì ton tại x tự nhiên sao cho A = x(x − 1)(x + 1), x ≥ 1. De thay x phải là so chȁn (vì neu ngược lại thì A chia het cho 8, mâu thuan).
  • 71. 67 Viết đề tài giá sinh viên – ZALO:0973.287.149-TEAMLUANVAN.COM n 2 n n−1 n−1 n−1 2 n n n−1 Ta thay A ≡ 12.(−1)n ≡ 2.(−1)n (mod 5), trong khi x(x − 1)(x + 1) = x(x2 − 1) ≡ {0; ±1} (mod 5), mâu thuan. Do đó, trong trường hợp này không có n thỏa mãn đe bài. V y tat cả các c p so thỏa mãn đe bài là (n; k) = 0; 2). Bài toán 3.10 (Thi HSG Hà N®i, vòng 2, 2012). Cho dãy so (un) xác định bởi a1 = 20; a2 = 30; an+2 = 3an+1 − an (n ∈ N∗). (3.1) Tìm tat cả các so nguyên dương n sao cho 5an+1an + 1 là so chính phương. Lài giai. Áp dụng ket quả (3.1) với dãy (an), ta được Suy ra an+1an−1 − a2 = a1a3 − a2 = 500. Xét với n ≥ 4, ta có a2 + 500 = an+1an−1. (an + an+1)2 = a2 + 2anan+1 + a2 . M t khác n n+1 a2 = 9a2 − 6anan−1 + a2 . Suy ra n+1 n n−1 (an + an+1)2 = a2 + 2anan+1 + 9a2 − 6anan−1 + a2 = 2anan+1 + 3an(3an − an−1) + a2 + an(an − 3an−1) = 5anan+1 + a2 — anan−2 Do đó = 5anan+1 + a2 — (an−1 + 500) = 5anan+1 − 500. (an + an+1)2 = 5anan+1 − 500 < 5anan+1 + 1. Tà dãy (an) tăng và n ≥ 4, ta có an + an+1 ≥ 180 + 470 = 650. Suy ra (an + an+1 + 1)2 = (an + an+1)2 + 2(an + an+1) + 1 > (an + an+1)2 + 501 = 5anan+1 + 1.
  • 72. 68 Viết đề tài giá sinh viên – ZALO:0973.287.149-TEAMLUANVAN.COM ·· · V y (an + an+1)2 < 5anan+1 + 1 < (an + an+1 + 1)2 nên 5anan+1 + 1 không chính phương. Bang phép thả trực tiep với n = 1, 2, 3, ta thu được n = 3 là giá trị duy nhat can tìm. Bài toán 3.11 (VMO 1991 - Bảng B). Cho dãy so (an) xác định bởi a1 = 1.2.3, u2 = 2.3.4, . . . , un = n(n + 1)(n + 2). Đ t Sn = a1 + a2 + · · · + an. Cháng minh rang 4Sn + 1 là so chính phương. Lài giai. Ta có an = (n + 1) n2 + 2n = (n + 1) h (n + 1)2 − 1 i = (n + 1)3 − (n + 1). Suy ra Sn = a1 + a2 + · · · + an = 23 + 33 + · · · + (n + 1)3 − [2 + 3 + · · · + (n + 1)] = 13 + 23 + 33 + · · · + (n + 1)3 − [1 + 2 + 3 + · · · + (n + 1)] . Bang phương pháp quy nạp, ta cháng minh được 2 2 13 + 23 + 33 + + (n + 1)3 = (n + 1) (n + 2) 4 (n + 1)(n + 2) Suy ra 1 + 2 + 3 + · · · + (n + 1) = 2 . Tà đó, ta có Sn = n(n + 1)(n + 2)(n + 3) . 4 4Sn + 1 = n(n + 1)(n + 2)(n + 3) + 1 = n2 + 3n n2 + 3n + 2 + 1 = n2 + 3n 2 + 2 n2 + 3n + 1 = n2 + 3n + 1 2 . M t khác n ∈ N∗ nên n2 + 3n + 1 là so nguyên dương. V y 4Sn + 1 là so chính phương. Nh n xét 3.8. Ta có the bien đői như sau Sn = 1.2.3 + 2.3.4 + · · · + n.(n + 1).(n + 2),
  • 73. 69 Viết đề tài giá sinh viên – ZALO:0973.287.149-TEAMLUANVAN.COM 4 n 3 n n suy ra 4Sn + 1 = 1.2.3.4 + 2.3.4.(5 − 1) + · · · + + n.(n + 1).(n + 2)[(n + 3) − (n − 1)] + 1 = n.(n + 1).(n + 2).(n + 3) + 1. Bài toán 3.12 (Romania TST, 2002). Cho dãy so (un) xác định bởi u0 = 1, u1 = 1 un+2 = 14un+1 − un, n ≥ 1. Cháng minh rang với moi so tự nhiên n thì 2un − 1 là m®t so chính phương. Lài giai. t1,2 = 7 ± 4 Xét phương trình đ c trưng tương áng t2 = 14t − 1 có hai nghi m 3. Do đó, so hạng tőng quát của dãy là u = a 2 + √ 3 2n + b 2 − √ 3 2n . Theo giả thiet u0 = u1 = 1, ta tìm được 1 1 Suy ra a = 4 2 + √ , b = 2 − √ 3 Do đó u = 1 2 + √ 3 2n−1 + 2 − √ 3 2n−1 . √ 2un − 1 = 3 + 1 2n−1 — √ 3 − 1 2n 2n−1 2 Xét hai dãy nguyên dương (An), (Bn) mà √ 3 ± 1 2n−1 = A √ 3 ± Bn. Do đó √ 3 + 1 2n−1 − √ 3 − 1 2n−1 = 2B . Tà đó ta có 2un − 1 = Bn 2 2n−1 là m®t so chính phương. V y ta có đieu phải cháng minh. ( √ n 4 . .
  • 74. 70 Viết đề tài giá sinh viên – ZALO:0973.287.149-TEAMLUANVAN.COM u − 1 2 n Bài toán 3.13 (Thi HSG thành pho Can Thơ, 2012 - 2013). Cho dãy so nguyên (un) xác định bởi u1 = 1; u2 = 2 un = 4un−1 − un−2, ∀n ≥ 2, n ∈ N. a) Cháng minh rang un 2 + u2 − 4unun−1 = −3 với n ≥ 2, n ∈ N. 2 b) Cháng minh n 3 n−1 là so chính phương với moi n nguyên dương. Lài giai. a) Xét phương trình đ c trưng tương áng λ2 − 4λ + 1 = 0 có hai nghi m λ1 = 2 − √ 3; λ2 = 2 + √ 3. Khi đó un = A.λn + B.λn . 1 2 Với u1 = 1; u2 = 2 ta tìm được A = √ 3 + 2 ; B = 2 − √ 3 . 2 2 Suy ra Khi đó u = 1 2 − √ 3 n−1 + 2 + √ 3 n−1 ; n ≥ 1. u2 + u2 = 1 2 − √ 3 2(n−1) + 2 + √ 3 2(n−1) + V y n n−1 4unun−1 4 + 2 − √ 3 2(n−2) + 2 + √ 3 2(n−2) + 4 = 2 − √ 3 2n−3 + 2 + √ 3 2n−3 + 1. = 2 − √ 3 n−1 + 2 + √ 3 n−1 × × 2 − √ 3 n−2 + 2 + √ 3 n−2 = 2 − √ 3 2n−3 + 2 + √ 3 2n−3 + 4. u2 + u2 — 4unun−1 = −3. n u2 − 1 n−1 b) Cháng minh n 3 là so chính phương.
  • 75. 71 Viết đề tài giá sinh viên – ZALO:0973.287.149-TEAMLUANVAN.COM n n . . n n — − ≡ − ≡ ( n n−1 n 2u2 − u1 = 4 − 1 = 3.3 , ∀n ≥ 2 k Tà câu a) ta có 4u2 + u2 — 4unun−1 = 3u2 − 3. Suy ra hay (2un − un−1)2 = 3u2 − 3 u2 − 1 3 (2un − un−1)2 9 Ta sě cháng minh rang 2un − un−1.3, ∀n ≥ 2 2un−1 − un.3, ∀n ≥ 2. Th t v y, với n = 2 thì . 2u1 − u2 = 0.3, ∀n ≥ 2. Giả sả 2uk − uk−1.3, ∀k ≥ 2 2uk−1 − uk.3, ∀k ≥ 2 . Suy ra 2uk+1 − uk = 2(4uk − uk−1) − uk = 6uk + uk − 2uk−1 . 3; 2uk − uk+1 = 2uk − (4uk − uk−1) = −2uk + uk−1 . 3. Nói riêng, ta có 2un − un−1.3, ∀n ≥ 1. Suy ra 2un − un−1 = 3k, k ∈ Z. V y u2 − 1 3 = k2 . Suy ra u 2 − 1 3 là so chính phương. Bài toán 3.14 (IMO 1964). a) Tìm tat cả các so nguyên dương n sao cho 2n − 1 chia het cho 7. b) Cháng minh rang không có so tự nhiên n nào đe 2n + 1 chia het cho 7. Lài giai. a) Vì n là so nguyên dương nên ta xét các trường hợp của n như sau: • Với n = 3k, k ∈ Z ta có 2n − 1 = 23 − 1 ≡ 1k − 1 ≡ 0 (mod 7). Do đó, với n là b®i của 3 thỏa yêu cau bài toán. • Với n = 3k + r, k ∈ Z, r = 1, 2 ta có 2n 1 = 23k .2r 1 2r 1 1 (mod 7), r = 1 3 (mod 7), r = 2. Tà đó, suy ra n = 3k, k ∈ Z ta luôn có 2n − 1 chia het cho 7. b) Theo trên ta có 2n ≡ 1, 2, 4 (mod 7) với moi so tự nhiên n. Do đó 2n + 1 /≡ (mod 7) với moi so nguyên dương n. . = .